Vous êtes sur la page 1sur 141

Welcome to Fin10

Congratulations on taking the first step towards become better


financially literate.
First things first If you are reading this because you think it will help you become a Financial
Planner / Advisor and provide you with the expertise to invest your money directly, then you
might as well stop reading right now.
Here are some of the things that you will NOT learn from this course:

This course is not a Certification course and is not something that you should put on
your resume (Though if you are looking for a career in the financial services industry, it
can form as a good primer for the basics)

It will not teach you to become a professional Financial Planner

It will not teach you to become an expert Stock Market Investor overnight

It will not teach you any get rich quick schemes

Here is what you will get:

It will teach you to become financially aware and manage your money

It will teach you the basics of money including savings, expenses, inflation, asset
classes, portfolio management, diversification, and investments

It will ensure that when you are talking to a professional advisor, you understand
exactly what they are suggesting + What questions to ask them

It will ensure that when you are buying financial products (like insurance or mutual
funds), you know how to select the right products

Fin10 from www.finqa.in | info@finqa.in |+91 96500 65242

All Rights Reserved

Why did we start Fin10?


Fin10 is an initiative by Finqa, a fee based financial advisory firm, which helps individuals
with planning their personal finances and investments for the long term.
While conducting workshops for our clients, we discovered that a lot of individuals were
ignorant of the basics of personal finance. We realized that its because of the fact that
managing money is something that is not taught either in schools or colleges.
Most of what we learn is from our parents, friends and family or via magazine / newspaper
articles. In our haste to make money, most of us dont really think about how to make their
existing money work harder.
Another reason is the rampant mis-selling of financial products, Insurance being one of them.
The primary purpose of insurance is to provide for your family in the event of a mishap
However, they are sold as investment products with the promise of a large return later in life.
What most people dont realize is that the annualized return on them is just about 6-7%
(which happens to be less than annual inflation) + the investment cover of Rs 5-10 lakhs is not
sufficient after an accident.
A lot of folks also have their money saved in savings or Fixed Deposit accounts. After tax
returns from both these are again way below inflation so your money is actually losing value
instead of growing over a period of time.
So thats when we thought, why not have a book or guide instead of individually explaining
the basics to everyone. We suggest you take out some time to go through the chapters and
see if these make sense to you.
If you have any questions, please fee free to contact us at:
info@finqa.in
+91 96500 65242 or visit our website at www.finqa.in
Cheers and all the best.
Ankur Kapur ,CFA ,CFP
ankur@finqa.in

Fin10 from www.finqa.in | info@finqa.in |+91 96500 65242

All Rights Reserved

Whats in this e-book?


100 lessons on a variety of topics as outlined below
Basics of Personal Finance
# 1: What do you want in life, financially?
# 2: What are some common financial mistakes?
# 3: What are some common financial myths?
# 4: How to make your personal finance budget?
# 5: What is your Net Worth?
# 6: How do you measure return?
# 7: The Eighth wonder of the world Compounding!
# 8: How does inflation impact your return?
# 9: Do you have an emergency fund?
# 10: How are financial products Mis-Sold?
Investment Types
# 11: What are the various forms of investment?
# 12: Types of investment vehicles: Public Provident Fund
# 13: Types of investment vehicles: National Savings Certificate (NSC)
# 14: Types of investment vehicles: Post Office Schemes and Deposits Post Office Monthly Income
Scheme (POMIS)
# 15: Types of investment vehicles: Government Securities
# 16: Types of investment vehicles: Corporate and Infrastructure Bonds
# 17: Types of investment vehicles: Bank Deposits
# 18: Types of investment vehicles: Real Estate?
# 19: Types of investment vehicles: Equity?
# 20: Types of investment vehicles: Mutual Funds?
Investment Strategy
# 21: Asset allocation defines your return
# 22: Why should one have a diversified portfolio?
# 23: What is your risk profile?

Fin10 from www.finqa.in | info@finqa.in |+91 96500 65242

All Rights Reserved

# 24: Understanding your risk profile


# 25:What should my portfolio be like?
# 26:How to create a savings fund of Rs 12+ crores?
# 27:Benefit from the Power of Compounding!
Mutual Funds
# 28: Why should one invest in Mutual Funds?
# 29: What are the different types of Mutual Funds?
# 30: What are Equity Mutual Funds?
# 31: What are Debt Mutual Funds?
# 32: What are Hybrid Funds?
# 33: What are ETFs? International funds? Arbitrage funds?
# 34: How can I invest in Gold?
# 35: What is a Systematic Investment Plan (SIP)?
# 36: What should one keep in mind while choosing a good Mutual Fund?
# 37: How to make extra return on your portfolio?
# 38: How to invest your lump sum money into Equity market?
Equity / Stocks
# 39: Myths of Investing directly in the stock markets
# 40: What is a Demat account and how does it work?
# 41: Income and tax slab
Taxation
# 42: Understand various income heads
# 43: What is salary and what does salary include?
# 44: Types of tax benefits on Investment products
# 45: Taxation of Capital Gains
# 46: How can I do safe investment for a short span?
# 47: Set off and Carry forward of Losses
# 48: Should I select growth or dividend option in case of a mutual fund?

Fin10 from www.finqa.in | info@finqa.in |+91 96500 65242

All Rights Reserved

# 49: Tax Saving Infrastructure Bonds vs. Infrastructure Bonds


# 50: How can a NRI open a bank account in India?
# 51: NRI equity investments and taxation in India?
# 52: NRI Mutual Fund investments and taxation in India?
# 53: What are the Income Tax rules on sale of a house?
# 54: 7 incomes you shouldnt forget to declare
Insurance
# 55: Steps in insurance planning
# 56: What are the different types of insurance products?
# 57: What is bonus in case of a Life-insurance?
# 58: Types of Life Insurance Policies
# 59: Riders on a Life Insurance Policy
# 60: Types of Non-Life Insurance Products
# 61: How do you calculate your life insurance needs?
# 62: What type of Insurance do you need?
Planning
# 63: Retirement planning is not for the old
# 64: How to determine your retirement expenses? Part I
# 65: How to determine your retirement investments? Part II
# 66: What is the amount you should be saving now for your retirement?
# 67: What are the kinds of retirement products?
# 68: What are the various mandatory retirement saving products?
# 69: What is a NPS under voluntary retirement saving option?
# 70: What are pension plans offered by insurance companies?
# 71: What are pension plans offered by mutual funds?
# 72: If you are already retired, what are your options for creating pension during retirement?
# 73: So many retirement products but which one should I buy?
# 74: When and why to rebalance your portfolio?

Fin10 from www.finqa.in | info@finqa.in |+91 96500 65242

All Rights Reserved

# 75: How can Real estate help you during your retirement?
# 76: Alternative to pension plans?
# 77: 10 things to do before you Retire
# 78: What should I do with my Bonus?
# 79: Should I buy a flat or rent one?
# 80: Why you dont want to be Amitabh Bachchan?
# 81: How can you not become victim of mis-selling?
# 82: Do I need a professional to help me with my finances?
# 83: Can I take care of my financial matters my family and myself in the future?
# 84: Higher education is expensive, why not plan for it!
# 85: True returns for the Investors
# 86: 5 steps to the perfect financial plan
How to do stuff?
# 87: Which should you use Credit or Debit Cards?
# 88: 6 things you should NOT try and save money on
# 89: Should You Hire A Financial Planner Or Wealth Manager?
# 90: When and why to rebalance your Portfolio?
# 91: Why your credit score is very important ?
# 92: Which Loans should you pay off first?
# 93: The 3 starters to Managing Debt Effectively
# 94: How to avoid the cycle of Bad Debt?
# 95: Things you should know about PPF
# 96: From whom to buy mutual funds
# 97: How to Open Post Office Monthly Income Scheme
# 98: How to buy National Savings Certificate
# 99: Want to invest in bonds but dont know where to start?
# 100: How to buy life insurance?
Lets get started

Fin10 from www.finqa.in | info@finqa.in |+91 96500 65242

All Rights Reserved

# 1: What do you want in life, financially?


Key Message: To be aware of why you are earning money and what you need to plan your
finances for.
Everybody wants to be rich and famous but very few of us have actually sat and thought about how rich do we
really want to be and what will keep us secure in life. Something along the lines of what makes you happy think
about it and the answer might surprise you.
Having money in your hand every month does not guarantee you the lifestyle you anticipate throughout your life.
Circumstances and needs always keep changing. Todays sound financial situation does not necessarily foretell an
equally rosy future. A loss of income, even temporary, can eat into your savings or lead to debt. An uninsured loss
can wipe out your accumulated wealth. Insufficient savings can force a reduced lifestyle during retirement.
Frequent or unplanned borrowings can leave negative money i.e., debts for future. Also, poor tax planning can
result in paying higher taxes than what you are liable to pay. All this, combined with changes in your life cycle
needs and/or external economic changes can make you and your future generations financially vulnerable.
You need to plan and manage your current income (the money you earn today) and your future income (money
you can expect to earn in future) according to your needs. These needs are your goals, are your dreams. `
People who write their goals are much more likely to achieve them. Sit down by yourself or with loved ones and
start to imagine your future. Consider what drives you in your life and how that has changed over time. While we
cant tell you what you should want in life, the below list of questions should provide you a fair idea of how you
should think about your future.
What milestones do you see in the future? Start a family, Send kids to college, Buy a new home,

Retire, etc
If you could do anything, time and money aside, what would that be? Spend more time with your family,

A second home, Start your own business

Fin10 from www.finqa.in | info@finqa.in |+91 96500 65242

All Rights Reserved

What would you like to add in your life? More time, More money, More social life, etc
What would you like to reduce in your life? Debt, Job stress, etc
Having a vision for the future and planning for that vision are as important as money in achieving a fulfilling life.
Envisioning your dreams and putting them down on paper is the first step in making them a reality. The next step is
to prioritize the goals that are most important to you and to establish milestone for reaching them.
Reflect on your life, your dreams, and your goals. Unless you know what you want to achieve, you will never be
able to reach where you want to be. Start planning financially for your dreams.
ACTION:
Make a list of your goals and an estimate of how much they cost today. Then, use this goal calculator (
http://finqa.in/investing-to-meet-your-goals-and-aspirations/ ) for calculating the future value of your goals.
This will give you an estimate of how much money will be required at various points in your life.

# 2: What are some common financial mistakes?


Key Message: Before you start planning your finances it may be relevant to know what are
some of the financial mistakes most people make.
Money saved is money earned. Majority of young people have goals of how they can spend
their salary and not save. Whether its a need to buy expensive phones/watches as a social
status symbol, or buy expensive cars on long term EMIs or eating out regularly A section of
these people who are living beyond their means are heading towards financial disaster. Here
are some common financial mistakes.
1. Spending frivolously
It may not seem like much at the time, but even small expenses when added up can lead to a
big hole in your pocket. Upgrading cell phones, buying a bigger car and regular movie tickets all
amount to a huge unnecessary expense. A first-day-first-show movie with popcorn for two
costs Rs.1000 every Friday, effectively means an outflow of Rs. 4,000 a month. Usually the
movie watching is followed by a dinner which costs Rs 2,000, an additional outflow of Rs 8,000
a month. Recreation is necessary too but going overboard can be controlled. Instead of going
out all four weeks you can go once a month. Saving this additional amount every month can
allow you Rs 1.08 lacs for something bigger over a year.
2. Mindless use of credit cards

Fin10 from www.finqa.in | info@finqa.in |+91 96500 65242

All Rights Reserved

Using credit card means spending borrowed money. Credit cards are a convenient way to
borrow money, but they come with double-digit interest rates. If the payments on your credit
card are not made on time, the money you will have to pay back will be much more than what
you spent on your card and that too within months. People are most susceptible to fall for this
mistake. Paying your credit card bills always in full and on time, will help you get the best of this
handy tool.
3. Caring little for regular savings
Contributing regularly to savings account, either for emergencies or for investment, is one of
the best wealth building habits, often ignored by many. Notwithstanding the nominal returns
on savings, building a healthy corpus by saving regularly is essential for sustenance and growth.
4. Investing based on half knowledge or imitation
Wise investment decisions should be based on ones risk profile, needs and priorities, and
financial goals. As these differ from individual to individual, investment strategies also should
be different. Just because an investment strategy has worked for someone (even for best of
friends), does not mean it would work for you. Investment decisions should be made only after
thoroughly knowing your investment needs and matching them with the options available to
you.
Here are a couple more which people make:
Not having an emergency fund

Not spending enough time budgeting and planning expenses


Not having enough insurance (And not aware of how much their company provides )
Procrastinating (I will start investing tomorrow / when the market is low )
Not asking enough / the right questions when buying financial products
Paying late fees and penalties
Buying too many LIC moneyback policies
Keeping surplus money in a savings bank account
Keeping a large corpus of your net worth in Fixed Deposits
Investing to save Tax (usually at the end of the year before filing taxes)

Action:
Make a list of mistakes which you make and try and think why you have been making them.

Fin10 from www.finqa.in | info@finqa.in |+91 96500 65242

All Rights Reserved

# 3: What are some common financial myths?


Key Message: How many myths have you fallen prey to? I have a lot of money (or very little
money); I need not plan
Whether you have a lot or little money, you cannot achieve your financial goals consistently
without a plan. Planning for your finances not only helps you achieve your goals, it also helps
you monitor and review your progress with your finances.
I am far too busy to spend my time on planning
Anything new seems to be tedious at first. But it is worth investing some of your time on
acquiring this skill as it would enable you to worry less about and enjoy more of your hardearned money. You should ideally spend time planning finances or overseeing from time to
time, as no one would understand your needs and goals better than you do.
If I earn enough now, I would have enough money during retirement
The only way to retire rich is to plan and manage your finances well today, no matter how much
or less you earn today. There are many cases of millionaires retiring paupers and of modest
earners retiring rich. The difference lies in the way they manage their cash, cash flows,
investment, debt and insurance.
Investing in mutual funds is risk-free and guarantees huge returns
No investment is risk-free, it can be either high risk or low risk and the risk is generally
proportional to the returns it offers. Mutual funds are managed by professional wealth
managers or portfolio managers, whose main task is to create an investment portfolio by wisely
balancing the risks and returns. Thus, the objective of the fund manager becomes to minimize
risks and maximize returns for the investors. But the risks cannot be totally eliminated and the
returns can never be promised.
Investing in stock markets is very risky and best avoided
Stocks are categorized as high riskhigh returns investment channel. They have over the years
proved to be one of the most profitable investments. Stocks have a significant role of growth to
play in a diversified investment portfolio. But some look at stock investment as a get-rich-quick
formula and invest blindly, without proper knowledge. Such people should certainly avoid
stock market investments, at least until they gather some know-how about it.
It is okay to pay the minimum amount mentioned in my credit card statement every month
Paying only the minimum amount means interest being charged on the balance amount, which
you anyway must pay sooner or later, with much more interest later. The sooner you repay all
Fin10 from www.finqa.in | info@finqa.in |+91 96500 65242

All Rights Reserved

the money spent on credit card, the more beneficial the card will be for you. Paying only the
minimum amount every month is one bad habit that many young people get themselves into
leading to mounting debts on them. It is best to avoid it. If you have got into this bad habit, the
sooner you get out of it, the better it is for you and your finances.

Action:
Think of some of the notions you have about money and your finances and list them below. We will
help you with if its a fact or myth and how you need to act accordingly.

# 4: How to make your personal finance budget?


Key Message: To find out where your money goes, and how much of a monthly deficit / surplus
you have.
Step 1: Identify your goals/dreams
What are your financial goals? Are you planning to go on a vacation? Are you planning to buy a
house/car? Do you have debts you need to pay off? Budgeting involves tough choices, but
having a goal will make budgeting a little less painful.
Step 2: Unravel the mystery of where all your money goes.
Do you check your bank account at the end of the month and wonder where all the money
went? Before you can manage your money, you have to know how youre spending it. Use an
excel file to track and categorize your expenses for one month. Get in the habit of recording
your expenditures once a day.
Its useful to separate your expenses into three categories:
1. Fixed Needs Necessary expenses that stay the same from month to month, e.g., rent,
phone bill.
2. Variable Needs Necessary expenses that may vary from month to month, e.g., petrol
expense, food.
3. Discretionary Needs Nonessential expenses, e.g., movies, eating out.
If you have a monthly savings goal (and you should!), include it as an expense. It is much easier
to save money if youve planned for it in your budget. And its important, too: if you run into
unforeseen expenses, youll want to be able to pay them without going into debt. And even if
nothing goes wrong, having some savings will help you follow your dreams in the future.
Step 3: Identify your sources of income.
Fin10 from www.finqa.in | info@finqa.in |+91 96500 65242

All Rights Reserved

Where does your money come from? List the sources of your income (e.g., work, rent, pension
plan) and the amount that comes in from each source each month. It is always a preferred
option to look at after tax income that will be available.
Step 4: Add it all up.
When you compare your income and expenses, do you have a monthly surplus, or will you need
to lower your standard of living.
If you already have a surplus in your budget, congratulations! You can invest in your future.
On the other hand, if your expenses exceed your income, step 5 will help you make some
adjustments.

Step 5: Make adjustments if needed.


If youre over budget, you need a strategy for controlling costs. Balance your budget, starting
with the discretionary expenses identified in step 2.
When you added up your monthly expenses, did you notice any surprisingly large
numbers? Did you spend Rs 5,000 at restaurants or on yet another new outfit? Did you spend
more on electronics than food?
Begin with such discretionary expenses that you may be overindulging in. For each type of
discretionary expenses, decide on a reasonable monthly limit that will help you balance your
budget. Set a cap on your discretionary expenses expenses and see if youve balanced your
budget.

Fin10 from www.finqa.in | info@finqa.in |+91 96500 65242

All Rights Reserved

If you cant trim enough from your discretionary expenses in order to balance your budget,
you will need to reduce your variable needs expenditures in the short term and perhaps your
fixed needs expenditures in the long term. This may mean taking the bus instead of driving and
finding less expensive house to rent next year.

Action:
Use our Savings Calculator (http://finqa.in/savings-calculator/) to identify your income and
expenses and determine your surplus / deficit. Create a budget accordingly if required. You should
be saving at least 10 20% of your income for growth investments

# 5: What is your Net Worth?


Key Message: Learn how to calculate your net worth.
No matter how much (or less) you know about investments, stock markets, credit cards and
insurance, if you do not know your money well, you are most likely to fail with in planning
financially. The first and perhaps the most important step in financial planning is to know your
money i.e., your financial position well and then be able to manage it wisely.
What makes up your financial position or Net Worth?
Your income and expenses: What you earn and how much you spend determines how much
money you have on hand after meeting your needs (and some wants). This balance is an
important indicator of your financial position. It can be used as a guideline to plan your
finances. A healthy positive balance every month indicates a trend towards a good financial
position and a zero or negative balance most of the months corresponds to a weak financial
trend (especially if you are not building assets on the way).
A healthy positive balance from your income will allow you to build funds for meeting your
financial goals. Below mentioned is a sample of what you need to have:
Sample outcome of cash flow analysis
Incomes
Net Salary Income (Salary)

Rs. 13,20,000 p.a.

Partners Net Salary Income (Salary)

Rs. 10,20,000 p.a.

Total Income:

Rs. 23,40,000 p.a.

Fin10 from www.finqa.in | info@finqa.in |+91 96500 65242

All Rights Reserved

Expenses
Living Expenses

Rs. 9,00,000 p.a.

Total Expenses:

Rs. 9,00,000 p.a.

Committed Savings
Regular Savings (MF SIP)
into Mutual Fund Savings/SIP (Mutual funds)
Total Committed Savings:

Rs. 1,20,000 p.a.

Rs. 1,20,000 p.a.

Repayments
Regular Repayment (Home loan)
into Housing Loan
Regular Repayment
into Car Loan

Rs. 7,80,000 p.a.

Rs. 84,000 p.a.

Total Repayments:

Rs. 8,64,000 p.a.

Total Income:

Rs. 23,40,000 p.a.

Total Expenses:

Rs. 18,84,000 p.a.

Net Cash Flow:

Rs. 4,56,000 p.a.

Your assets and liabilities: The amount of assetsitems of valueyou hold, is a precise
indicator of your current and future financial position. Assets tend to add to your income
(either now or in future) e.g., investmentsin gold/silver, deposits, stocks, mutual funds,
art/antique, land etc. Or they help reduce expenses, as in case of owning a houseit saves you
taxes and rent. Thus, assets help to strengthen your financial position. On the other hand,
liabilities weaken your financial position.
Debtsomething that you owe is a liability. And so is an old vehicle that needs a lot of fuel and
repairs, for the work it is doing.
More assets and lesser liabilities would help you better your financial position and strengthen
your money.
Fin10 from www.finqa.in | info@finqa.in |+91 96500 65242

All Rights Reserved

Sample Statement of Position or Net Worth


Investments

Current Valuation

Cash in Hand (Bank balance)

Rs. 3,00,000

Fixed Deposit (Bank FD)

Rs. 9,00,000

Mutual Fund Savings/SIP (Mutual funds)

Rs. 5,00,000

Public Provident Fund (PPF)

Rs. 10,00,000

Employee Provident Scheme (EPF)

Rs. 15,00,000

Partners Employee Provident Scheme (EPF)

Rs. 8,75,000

Total Investments:

Rs. 50,75,000

Other Assets

Current Valuation

Residential Property

Rs. 1,00,00,000

Car / Two Wheeler

Rs. 4,50,000

Total Other Assets:

Rs. 1,04,50,000

Liabilities

Current Valuation

Credit Card (Citi)

Rs. 45,000

Housing Loan

Rs. 65,00,000

Car Loan

Rs. 3,45,000

Total Liabilities:

Rs. 68,90,000

Total Assets:

Rs. 1,55,25,000

Net Worth:

Rs. 86,35,000

Just like any disease is best treated when detected in an early stage, similarly with regard to
your financial health recognizing early signs can help you take right steps and prevent great
disasters for yourself.

Fin10 from www.finqa.in | info@finqa.in |+91 96500 65242

All Rights Reserved

Action:
Calculate your Net Worth via our Net Worth Calculator ( http://finqa.in/networthcalculator/)
Focus on having a larger Net Worth than a liquid bank balance (Its not funny how many Indians
are obsessed with having a large bank balance which does pretty much nothing for their
money.

# 6: How do you measure return?


Key Message: There are various ways you can calculate investment return. Today you will
learn broad classes of how you can evaluate performance of your investment.
Return on investment is a basic computation made to assess how an investment is performing.
Return can be measured in three ways:
1.) Absolute Returns
Comparing the amount of inflows and outflows on an investment in absolute terms. In
mathematical terms an absolute return can be calculated as:
Absolute return = (Ending value Beginning value) Beginning value X 100
2.) Annualized Return
Computing an annualized rate of return by comparing inflows and outflows.
Annualized return = (Ending value Beginning value) Beginning value X 100 X
(1/Holding period of the investment)
Annualized return can also be calculated in the form of Compounded Annual Growth Rate
(CAGR). We will develop detailed understanding of CAGR in tomorrows lesson.
3.) Total Return
Total Return is the return computed by comparing all forms of return earned on the
investment. For example, dividend payment, interest payment, bonus payment. Thus total
return is the annualized return after including all benefits on the investment.
Total return = (Ending value Beginning value + additional benefits) Beginning value X 100 X
(1/Holding period of the investment)
Fin10 from www.finqa.in | info@finqa.in |+91 96500 65242

All Rights Reserved

Comparing the above 3 types of returns and why they make a difference.
Lets say you invested Rs 1 lac and it became Rs 1,16,664 lacs (8% p.a.) in 2 years. We will use
this example to calculate returns as per the above mentioned methods.
I.

Absolute return = (Ending value Beginning value) Beginning value X 100

(116664 100000) 100000 X 100

=
II.

16.6 %
Annualized return = (Ending value Beginning value) Beginning value X 100 X

(1/Holding period of the investment)


=

(116664 100000) 100000 X 100 X (1/2)

=
8.3% p.a.
III.
Total return = (Ending value Beginning value + additional benefits) Beginning value X
100 X (1/Holding period of the investment)
Lets say a dividend was also paid in second year, Rs 5000.
=

(116664 100000 + 5000) 100000 X 100 X (1/2)

=
10.8% p.a.
The rate of return that you should ideally be looking at during investments is the Annualised
rate of return and your aim at the minimum should be to beat inflation. E.g. in the scenario
above, if the inflation was 7% in the last 2 years, then your net increase in value of money was
actually 1.3%. However, if you consider it with Absolute or total returns, then the figures can
appear higher than they really are.
This is also a tactic most insurance agents use to fool people Their pitch will usually go like
Invest Rs 500,000 this year and at the end of 15 years, you will get Rs 13 lakhs a return of
157% Most people will think in absolute terms while the real annualised returns may be much
lower.

Action:
Whenever you are buying a financial product, ask the seller for the expected annualised rates of
return instead of the absolute returns. Use our Real Rate of Return (http://finqa.in/real-rate-returninvestment/) calculator to calculate the actual rate of return on your existing investments

Fin10 from www.finqa.in | info@finqa.in |+91 96500 65242

All Rights Reserved

# 7: The Eighth wonder of the world The power


of Compounding !
Key message: The incredible power of Compounding.
Compound interest is the eighth wonder of the world. He who understands it, earns it he who
doesnt pays it. Albert Einstein
As time progresses some things just get better..Red wine gets an enhanced taste while aged
cheese is so much tastier. Women tend to look more beautiful and generally men tend to get
wittier.
Dont you agree?
Now whats with Money?? It grows.. grows with time
And why?? Because it gets compounded.
So what exactly is compound interest ? Its earning interest for interest already earned. In other
words, your interest on savings also earns interest. Suppose you deposit some money in your
account today, there will be an increase in the value of it after a year. And, thats because it has
earned interest.
The journey to wealth can begin with even keeping aside the smallest amount each day.
Dont believe?? The picture below says a thousand words.

Fin10 from www.finqa.in | info@finqa.in |+91 96500 65242

All Rights Reserved

Rs. 15000/- invested per month for 20 years will turn into more than Rs 1.13 crores.
On the contrary, if you left it in a savings account, it would be worth something in the region of
Rs 36 lakhs.
And the best part about enjoying the power of compounding is:
You dont have to be rich to see its benefit; but you can become rich in the process
You can start from today; so why not start now?
The more time you give it, the fatter balance you can enjoy with, later.
Compounded annual growth rate (CAGR) can be calculated by using the following formula:

= ((Ending value/Beginning value) ^ (1/holding period of investment) 1) X 100


Lets say Rs 1 lac became Rs 1,16,664 lacs (8% p.a.) in 2 years. CAGR would be:
= (1,16,664/100000)^(1/2)-1 X 100
= 8% p.a.
CAGR is the most powerful tool to assess performance of your investments. Now an LIC or bank
agent cannot fool you by saying if you invest Rs 5 lacs today, after 15 years you will get ~13 lacs,
a return of 157%. Because you would know that CAGR for this investment would be mere 6.5%
p.a. which does not even meet inflation rate. This sort of investment will end up destroying
your wealth rather than generating any positive returns for you.

Fin10 from www.finqa.in | info@finqa.in |+91 96500 65242

All Rights Reserved

However, its not as easy as it sounds else everyone would have been a millionaire by the time
they retired. The good news is that with a bit of planning, you can reap the benefit of
compounding. As a first step, you will need to plan your expenses in such a way that you save
20% of your income every year. Secondly, your savings need to be invested in such a way that
they provide you with consistent return.

Action Item:
Review your existing investments and see if they are utilizing the power of compounding.

# 8: How does inflation impact your return?


Key message: What is inflation and how it impacts your savings and earnings.
Everyone knows that inflation eats into savings and increases costs, but do you know by how
much?
Inflation is a rise in general levels of price of everyday goods, over a period of time. If your
household expenses were Rs 20,000 in 2003, you have to spend double the amount now!!

Year

Inflation

2003

Expense
20,000

2004

3.78%

20,756

2005

5.57%

21,912

2006

6.53%

23,343

2007

5.51%

24,629

2008

9.70%

27,018

2009

14.97%

31,063

2010

9.47%

34,004

2011

6.49%

36,211

2012

11.17%

40,256

Fin10 from www.finqa.in | info@finqa.in |+91 96500 65242

All Rights Reserved

2013

9.13%

43,932

Due to inflation, a steady income alone is not enough to help you reach your financial goals. For
example, the current cost of a college admission may be Rs. two lakhs. But after 5 years, the
cost would typically be higher. While saving for a goal, therefore, it is important to estimate the
future value of the goal because that is the amount that has to be accumulated.
The future value of a goal = Current Value x (1+ Rate of Inflation) ^ (Years to Goal)
If the rate at which the cost increases is taken at 10% then the cost of the college admission
after 5 years would be: Rs.200000 x (1+10%) ^ 5= Rs.3,22,102. This is the value of the goal
which needs to be achieved by saving and investment.
So now you know that your focus on saving and keeping your money in your savings account or
a fixed deposit is not sufficient to meet your goal. This is the reason provident contribution and
gratuity will not take care of your retirement needs. You typically have to keep your savings in a
diversified pool of assets that will help you provide income and growth.

Action
Consider the following strategies for inflation-proofing your portfolio:

Start investing as soon as you can to take advantage of the power of compounding.

Seriously consider investments with a track record of beating inflation.


Talk to your financial advisor about an investment plan tailored to your personal needs.

# 9: Do you have an emergency fund?


Key message: Do you have sufficient funds to meet your financial needs if you were to leave
your job? Know how much to save for your contingency purposes.
Imagine you won a bumper prize for an all paid expense trip with family to a place of your
choice. Or Imagine seeing an email from your dream company selecting you for a job & offering
a salary 7 times your current one. Imagine you wake up to find yourself a whole lot richer after
hitting a jackpot of Rs. 50 crores.

Fin10 from www.finqa.in | info@finqa.in |+91 96500 65242

All Rights Reserved

So all smiles and all glory, who said money cant buy happiness?
Now, imagine if you were to lose your job all of a sudden tomorrow or you the financial
backbone of the family met with a fatal accident. Just as much as you would like to dream of all
the good things you also need to plan for unforeseen circumstances.
So, amidst brouhaha over inflation, job insecurity, unexpected emergencies and no hikes in
salary there is nothing that can guarantee you the event reversal but yes definitely something
that can help you traverse these moments more confidently setting an emergency fund. Just
as the name implies, emergency fund is the amount of money that you save solely to help you
sail smoothly during emergencies. An emergency fund forms the core of a smart financial
strategy. At the time of despair the last thing that you would want to worry about is regarding
how to arrange for the money. It is then that you can rely on the money that you have kept
aside every month.
A better way to maintain this would be to keep separate accounts for smaller emergencies (like
unexpected expense on car repairs or any home repairs) and for greater emergencies (like
losing a job or any natural disaster). An emergency fund must be kept liquid i.e. you must be
able to actually convert it to cash in your hand so that you can access it immediately.
Though there is no fixed value as in how much one should set apart as emergency fund, it is
generally believed that you should have enough to sustain yourself for at least 3-6 months in
your current lifestyle.
And just because an emergency is truly an emergency that comes uninvited or without notice;
sooner you start out better for you.
The Emergency fund is to be used only for absolute emergencies and not for vacationing or
indulging into your desires.

# 10: How are financial products Mis-sold ?


Key Message: We will cover broad areas how various financial products are mis-sold.
The way a bank or insurance company sells in India is much like a trap with companies,
regulators, agents and staff colluding to defraud you of the hard earned money. The process is
usually as follows: you have a bank that you have banked for years. As soon as there is any
Fin10 from www.finqa.in | info@finqa.in |+91 96500 65242

All Rights Reserved

credit (salary, bonus etc.) in your account, the trigger is generated and sent to your account
relationship manager. As soon as a trigger is sent to your Relationship Manager, he would call
you to fix an appointment. This manager pitches a product to you. The pitch is verbal. The
merits of buying this product are explained with great enthusiasm. However, the same merits
will not appear in the product brochure. The excuse will be that the brochure is not updated.
Words like guaranteed, surety, historical returns and mutual fund with free insurance are
commonly used. Next day your RM will follow up again and probably even pressurize to decide
quickly. You will get convinced because after all you are dealing with your own bank. You would
have bought something for which you either have no knowledge or half-baked knowledge.
One of the two things will happen after few months. Either you will find out that the product
you bought is not the one that was described to you. Or you wait for few years to see that
market is performing well but the product has not delivered any return. When you approach
your bank with your concerns, you will find a different manager who would be your new
relationship manager. He will either say talk to the relevant department or you should return
back the policy and invest in a better scheme with the same company. You will land up sending
emails to 100 different departments or get sold for the new product with new features.
These scams are very common irrespective whether you are dealing with a government agency
or a private company. All these relationship managers are sales agents and do not have any
knowledge of the product. They are assigned targets and they are simply chasing those targets.
Neither do they have any understanding of your financial objectives nor do they care.
A rampant practice among banks is to cross-sell insurance and mutual funds of their respective
insurance and asset management companies to customers who approach them for a loan.
Customers taking a home loan are sold mortgage redemption insurance or term insurance
policies of their insurance subsidiaries. Similarly, when a customer wants to get a locker in a
bank branch, he is made to invest in a fixed deposit. Bank personnel selling mutual funds or
insurance plans are usually not even certified by AMFI and IRDA to sell the respective products.
How can you avoid becoming a victim of mis-selling?

You must have your own basic understanding of the financial product.

You must understand how the financial product helps you achieve your financial objective.
Do not accept facts as suggested by the agent, investigate.
Any time is a good time to start investments so take your time to perform due-diligence.
Please read offer document before signing it.
Make sure the agent is registered with relevant regulator, ask for licence details.
Ideally deal with a fee-based SEBI registered investment advisor.

Fin10 from www.finqa.in | info@finqa.in |+91 96500 65242

All Rights Reserved

# 11: What are the various forms of investment?


Key Message: Know the difference between an investment product and Asset Class.
You must have come across various occasions when a sales agent would push for a financial
product. There would be numerous benefits quoted by the sales agent regarding the product.
But what this agent forgets is the basic aspect of any investment instrument. The returns of any
financial product depends upon its underlying. For example, a public provident fund (PPF) is a
debt instrument and is directly dependent on the interest rate in the economy. The underlying
remains the same for a fixed deposit, government security or a corporate bond. Therefore, the
return of any financial product depends upon the underlying and not on the claim made by an
agent.
Every investment option can be described in terms of its risk and return characteristics.
Traditionally the asset classes were broadly equity and bond. However, as investment options
have extended beyond capital market products, these basic categories have also expanded to
include commodities, real estate and currency. The risk and return features of each asset class
are distinctive. Therefore, the performance for each asset class may vary from time to time.
Following is the list of generally used asset classes and their risk-return attributes:
Debt
Debt instrument provide fixed return in the form of coupon/ interest income.

Debt instrument have the scope for capital appreciation when interest rates fall, but may
be subject to interest rate risk when interest rates rise.
Risk and return characteristics of Debt instrument are relatively lower than equity and

hence, suitable for an investor seeking regular income flows with minimal risk.
There are variety of assets that are categorized as debt. For example, public provident fund
(PPF), National Savings Certificate (NSC), Provident Fund, fixed deposit, corporate bonds,
government bonds and treasury bills.
Equity
A stock represents ownership in a company.

Empirical study suggests that this asset class provides higher returns if invested for long
run.
Volatility is higher in this asset class than cash and bonds as an asset class.
Equity can be broadly classified as large cap, mid cap and small cap. We will build our deep
understanding in the later lessons on equity classification.
Remember, Life Insurance Corporation (LIC) also invests in equity. Bonus amount in case of
an insurance product would also depend upon how the underlying portfolio of equity has
performed.
Fin10 from www.finqa.in | info@finqa.in |+91 96500 65242

All Rights Reserved

Real estate
Real estate involves investment in land or building (commercial as well as residential).
Real estate is also considered as a growth asset that has the potential of providing higher
returns if invested for long run.
Real estate investment includes commercial real estate, residential real estate and real
estate investment trusts (REITS).
Gold
Physical gold is preferred by Indian families as a secured and stable investment and is also
highly liquid.
Gold is generally used as a hedge against inflation.
Gold category would include investment in physical gold, gold fund, e-gold or gold
exchange traded fund.
There are varieties of other investment options such as derivatives, hedge funds and private
equity but these are not suitable for retail investor. In case you are interested in knowing about

these products, you may write to us and accordingly we can share material on these products.
In the next few lessons, we will develop deeper understanding of these asset classes.

# 12: Types of investment vehicles: Public


Provident Fund
Key Message: Public Provident Fund is probably one of the ways that the previous generation
used to save for their retirement. For todays generation PPF should be just a part of ones
portfolio.
Objective The objective of the PPF is to provide a saving option to those individuals who may
not be covered by the provident funds of their employers or may be self-employed. The return
is decided by Government of India and is directly dependent on the prevailing interest rate in
the economy. Therefore, PPF can be categorized in the debt asset class.
Basic features
PPF is a 15-year deposit account that can be opened with a designated bank or a post
office.
A person can hold only one PPF account in his or her own name or in the name of a minor
child to whom he or she is a guardian.

Fin10 from www.finqa.in | info@finqa.in |+91 96500 65242

All Rights Reserved

Account can be opened by an individual for himself/herself, and or on behalf of a minor of


whom he/she is a guardian. HUFs and NRIs are not allowed to open PPF accounts. If a
resident subsequently becomes an NRI during the prescribed term, he/she may continue to
subscribe to the fund till its maturity on a non-repatriation basis.
Joint account cannot be opened, however nomination facility is available.

Maturity
Regular deposits have to be made for a period of 15 years; penalties apply for skipping the
deposit. The account matures after expiry of 15 years from the end of financial year in
which the account was opened.
One withdrawal in a financial year is permissible from seventh financial year from the year

of opening the account. Maximum withdrawal can be 50% of balance at the end of the
fourth year or the immediate preceding year, which ever is lower.
The account can be closed in the 16th financial year or continued with or without
additional subscription, for further blocks of 5 years. However, the continuation can be

with or without contribution. Once an account is continued without contribution for more
than a year, the option cannot be changed.
Investment Limits

Minimum amount that needs to be deposited in this account is Rs.500 and maximum
amount that can be deposited in a financial year in this account is Rs.1,00,000.

Maximum Number of Deposits


A maximum of 12 deposits can be made in a single year. Interest is calculated on the
lowest balance available in the account between 5th of the month and the last day of the
month. However, the total interest in the year is added back to PPF only at year end.
Interest is cumulated and not paid out.
Taxation
Contribution to PPF is eligible for deduction under sec 80C of Income tax Act 1961. Interest
is completely tax free. Unlike other instruments which are eligible for tax deduction under
Section 80C, PPF enjoys an exempt-exempt-exempt (EEE) status, where withdrawals are
also not taxed.
A PPF account is not subject to attachment (seizure of the account by Court order) under
any order or decree of a court. However income tax authorities can attach PPF accounts to
recover tax dues.
PPF is quite attractive because of exempt-exempt-exempt (EEE) status. However, you must bear
in mind that PPF rate of interest is decided by the Government and is directly linked with the
interest rate environment in the country. As an investment asset, a PPF should be treated in par
with the income assets

Fin10 from www.finqa.in | info@finqa.in |+91 96500 65242

All Rights Reserved

# 13: Types of investment vehicles: National


Savings Certificate (NSC)
Key Message: My father had NSCs, I should also have these certificates. But wait, do you even
know what NSCs are and how do they operate.
Features
NSCs are like bonds, issued by the government for a specific period, and pay interest. They
can be bought from a post office and are usually held until maturity.
NRI, HUF, Companies, trusts, societies, or any other institutions are not allowed to
purchase the NSCs.

Certificates are available in denominations of Rs. 100, 500, 1000, 5000, and 10000.
Minimum investment is Rs. 500 without any maximum limit.
It can be bought by an individual or jointly by two adults. Nomination is possible. NSCs can
also be bought in the name of minors.

Maturity
The certificates issued under NSC (VIII issue) Second Amendment Rules, 2011, will be for a
maturity period of five years, commencing from the date of issue of the certificate.

Premature encashment is allowed only in case of death of the holder, forfeiture by a


pledge, or under orders of court of law.

Pre-mature encashment is permitted after a period of 3 years from the date of purchase at
a discounted interest rate. The certificates are also accepted as collateral for taking a loan.
NSCs are not transferrable.

Taxation
NSC enjoys tax benefit under section 80C of Income Tax Act, 1961.
Interest on NSC VIII is 8.5% p.a. compounded half yearly, which works out to be 8.68% p.a.
Accrued interest is taxable, but is deemed to be reinvested and therefore eligible for
Section 80C benefits.
As an investment asset, a NSC should be treated in par with the income assets (Debt asset
class). NSCs are not very attractive investment now because the interest you earn on NSC is
taxable. Lets say you operate at 30% tax slab, the effective after-tax return on NSC would be ~6
p.a. Is this return good enough to beat inflation @8%, probably not. In fact, if you are keen to
invest in a government instrument, PPF is quite attractive because of exempt-exempt-exempt
(EEE) status. But even in case of PPF, rate of interest is decided by the Government and is
directly linked with the interest rate environment in the country.

Fin10 from www.finqa.in | info@finqa.in |+91 96500 65242

All Rights Reserved

# 14: Types of investment vehicles: Post Office


Schemes and Deposits Post Office Monthly
Income Scheme (POMIS)
Key Message: Post office schemes are usually not talked a lot in youth, but no harm in
understanding these options, right!!
Investment in post office scheme is primarily to provide regular monthly income to the
depositors. This scheme has a term of 5 years (reduced from 6 years w.e.f 1 December, 2011).
Minimum amount of investment is Rs.1500, and maximum amount in case of single account is
Rs. 4.5 lakhs, and in case of joint account is Rs. 9 lakhs. Interest rate is 8.4% p.a. payable
monthly with no bonus on maturity. The earlier facility of bonus of 5% payable on maturity
stands withdrawn from 1 December, 2011.
Premature withdrawal of the invested amount is allowed after 1 year of opening the account. If
the account is closed between 1 and 3 years of opening, 2% of the deposited amount is
deducted as penalty. If it is closed after 3 years of opening, 1% of the deposited amount is
charged as penalty.
Post Office Time Deposits (POTD) These are similar to fixed deposits of commercial Banks.
Interest rates are calculated half-yearly and withdrawals are permitted after six months. No
interest is paid on closure of accounts before one year, and thereafter the amount of deposit
shall be repaid with interest @2% below the corresponding time deposit rate for the completed
number of years. Accounts can be pledged as a security for availing a loan. The POTD is offered
for various durations as mentioned in the below table:

Fin10 from www.finqa.in | info@finqa.in |+91 96500 65242

All Rights Reserved

These rates are subject to changes annually as indicated by the government.

Duration

Interest rates (p.a.) w.e.f 1 April 1, 2013*

One Year

8.2%

Two Year

8.2%

Three Year

8.3%

Five Year

8.4%

Post office schemes are traditional channels of safe investment option that can provide you
current income (debt asset class). These schemes do not offer any tax advantage and interest is
completely taxable.

# 15: Types of investment vehicles: Government


Securities
Key Message: Investment in Government securities is not very popular among retail clients due
to tedious process involved in the opening of the account. However, if you decide to invest in GSec, you should have a good understanding on these securities operate.
Government securities (G-secs) are issued by the RBI on behalf of the government and are
categorized in debt asset class. G-secs represent the borrowing of the government, mostly to
meet the deficit. Deficit is the gap between the governments income and expenditure. G-secs
are issued through auctions that are announced by RBI from time to time. Banks, mutual funds,
insurance companies, provident fund trusts and such institutional investors are large and
regular buyers of G-secs.
With a view to encouraging retail participation, the government has reserved 5% of the auction
amount in every auction for non-competitive buyers, including retail investors. In order to buy
G-secs retail investors have to open a constituent SGL account with their bank or any other
holder of SGL (Securities General Ledger) accounts. The CSGL account is held as parts of the
accounts of the offering bank, in which the G-secs are held as electronic entries in demat form.
Investors can also hold G-secs in their normal demat account, after buying them.

Fin10 from www.finqa.in | info@finqa.in |+91 96500 65242

All Rights Reserved

The minimum investment amount is Rs.10,000. Investors can apply for buying G-secs through
their SGL-holding bank and make the payment through their bank. The price at which they will
be buying the G-secs will be determined at the end of the auction. Interest is paid out on prespecified dates into the designated bank account of the investor. Interest is not subject to TDS
but is fully taxable. Redemption proceeds are also paid into the bank account.
Though there is a retail debt market segment in which all issued G-secs can be traded, there is
no liquidity for small lots of G-secs. The institutional market where the trading lot is Rs.5 crore
is quite active. Retail investors may have to hold the G-secs to maturity.
G-secs are benchmark securities in the bond market, and tend to offer a lower interest rate
compared to other borrowers for the same tenor. This is because there is no credit risk or risk
of default in a G-sec.
Although the option of investing directly into a G-Sec is available to you, but understanding the
bond market may be a challenge. Instead Gilt fund maybe a better option. We will share more
details about Gilt funds in mutual funds section.

# 16: Types of investment vehicles: Corporate and


Infrastructure Bonds
Key Message: Lot of people invest into corporate and infrastructure bonds without knowing the
risk attached to investing into these bonds.
Corporate bonds are debt instruments issued by private and public sector companies. They are
issued for tenors ranging from two years to 15 years. The more popular tenors are 5-year and
7-year bonds. Publicly issued bonds tend to have a face value of Rs.10,000.
Bonds of all non-government issuers come under the regulatory purview of SEBI. They have to
be compulsorily credit-rated and issued in the demat form. The coupon interest depends on the
credit rating. Bonds with the highest credit rating of AAA, for example, are considered to have
the highest level of safety with respect to repayment of principal and periodic interest. Such
bonds tend to pay a lower rate of interest than those that have a lower credit rating such as
BBB.
Bonds issued by companies in the financial sector tend to carry a higher coupon interest rate,
compared to those issued by companies in the manufacturing sector. This is also due to the
Fin10 from www.finqa.in | info@finqa.in |+91 96500 65242

All Rights Reserved

perception of higher risk, as companies in the finance sector tend to borrow more (as a
proportion of their equity capital) compared to companies in the manufacturing sector.
Apart from a regular fixed-interest-paying bond, the other types of bonds issued are: zero
coupon bonds, floating rate bonds and bonds with put or call options. Convertible bonds, allow
investors to convert the bond fully or partly into equity shares, in a pre-determined
proportion. Corporate bonds offered to retail investors tend to feature various options, to
make it attractive to investors across tenors and frequency of interest payments. Apart from
credit risk, retail investors also bear liquidity risk while buying these bonds. The interest on
these bonds is completely taxable.
There is lot of analysis which needs to be performed in selecting the right corporate bonds for
investment. You must invest into corporate bonds directly only if you understand how the debt
market operates and what the interest rate scenario in the economy is.
Infrastructure Bonds
The government announces from time to time, a list of infrastructure bonds, investment in
which is eligible for deduction under Section 80C of the Income tax Act. Bonds issued by
financial institutions like the Industrial Development Bank of India (IDBI), India Infrastructure
Finance Company Ltd. (IIFCL) and National Bank for Agriculture and Rural Development
(NABARD) are eligible for such deduction. The bonds are structured and issued by these
institutions as interest paying bonds, zero coupon bonds or any other structure they prefer. The
terms of the issue such as tenor, rate of interest and minimum investment may differ across the
bonds. What is common is that these bonds have a minimum lock-in period (which could be
three years, or five years) and they cannot be transferred or pledged. Investment up to Rs
20,000 in these bonds is eligible for income tax deduction under Section 80 CCF of the IncomeTax Act. This is over and above the Rs 1,00,000 deduction available under Section 80C. The
interest earned is added to the income and taxed according to the investors income tax
bracket. No tax is deducted at source if the annual interest is less than Rs 2,500.
You must take the benefit of extra tax savings in an infrastructure bonds. If you operate at the
highest tax bracket and if you plan to invest Rs 20,000 in an infrastructure bonds, you
effectively invest Rs 14,000. This is because Rs 6000 are potential tax savings which you would
have

paid

as

taxes

had

you

not

invested

Fin10 from www.finqa.in | info@finqa.in |+91 96500 65242

into

infrastructure

All Rights Reserved

bonds.

# 17: Types of investment vehicles: Bank Deposits


Key Message: Everyone knows fixed deposits with banks, but today we will explain why a fixed
deposit may not be the best option always.
A bank fixed deposit (FD) also called as a term or time deposit, as it is a deposit account with a
bank for a fixed period of time. It entitles the investor to pre-determined interest payments and
return of the deposited sum on maturity. Fixed bank deposits offer higher returns than savings
accounts as the money is available for use by the bank for a longer period of time. Fixed
deposits are preferred by investors who like their money to remain with their bank and do not
have an immediate need for it.
A fixed deposit is created by opening an FD account with the bank which in turn issues an FD
receipt. Interest on an FD can be paid into the depositors savings bank account at a predefined frequency, or accumulated and paid at the end of the term. On maturity, the lump sum
deposit amount is returned to the investor. Investors can also choose to renew the deposit on
the maturity date. The minimum deposit amount varies across banks. The duration of deposits
can range from 14 days to 10 years though FDs longer than 5 years are not very common.
Interest Rates on FDs Interest rates depend on the duration of deposit, amount deposited and
policies of the bank. In general, longer term deposits pay a higher rate than shorter term
deposits. Banks also offer special rates to senior citizens, defined as those who are over 60
years of age. Interest rates also vary from bank to bank. The interest rate paid by a bank
depends on its need for funds. Interest rates do not remain unchanged. Deposit rates offered
by banks for various tenors change over time, depending on the economic cycle, their need for
funds and demand for credit (loans) from banks. Having said that, an interest rate committed to
be payable for a tenor, until maturity, does not change even if market interest rates change.
New rates usually apply only for fresh deposits. Banks may also prescribe a minimum lock-in
period during which funds cannot be withdrawn from the FD account. They may levy a penalty
on depositors for pre-mature withdrawals.

Investment in specified (under Section 80C of the Income Tax Act) 5-year bank FDs are eligible
for tax deductions up to a maximum amount of Rs.1 lakh, along with other investment options
listed under the same section. These deposits are subject to a lock-in period of 5 years and have
to be added back to the taxable income in the year of redemption.

Fin10 from www.finqa.in | info@finqa.in |+91 96500 65242

All Rights Reserved

If you operate at the highest tax bracket (30%) and say you invest into a fixed deposit offering
9% p.a. Your after tax return would be 9% X (1-30%) = ~6.3%. Given the average inflation rate in
our country at 8.5%, do you think you will ever be able to grow your money?

The only situation you should invest into a fixed deposit is when you expect interest
rates to fall down so that you can lock-in your return at a higher interest rate.

# 18: Types of investment vehicles: Real Estate


Key Message: Its a myth that real estate prices can never decline.
Real estate investments may be structured as income generating or growth oriented
investment. Income generating investments focus on rental income; while growth oriented
investments seek to benefit from value appreciation over time. Real estate growth is aligned to
economic cycles, as real estate growth has a high dependence on money supply and credit
availability. Rental incomes are a hedge against inflation, as rentals rise with inflation.
Over valuation in bullish markets is a common feature, as prices rise rapidly and hence, is a big
risk. Therefore, the sector tends to suffer steep corrections when the bubble bursts. There are
even more concerns for under-constructed property. Project delays are a reality and very few
are completed on schedule. The new trend in real estate is also to offer freebies. From cars to
modular kitchens, all are being offered when you book an apartment in a project. Dont fall for
these lures. All freebies are already factored into the price of the apartment. The same goes for
the attractive schemes on offer. The truth is that developers urgently need the cash and many
of the projects have not even got approvals from the authorities. This puts a question mark on
these projects. Another such lure is the attractive financing schemes that builders offer by
tying up with banks and housing finance companies. In most cases, you will get a better deal by
approaching the bank directly.
Lot of people believe that real estate prices will never go down. This is just a myth, real estate
prices also go down but the decline is gradual. Therefore real estate should be considered as
any other asset class and typically should not be more than 20% of your investment portfolio.
Instead of investing a large amount into a real estate project, a better way of getting real estate
exposure is through Real Estate Investment Trusts (REIT). REITs are companies that buy and
manage rent-producing assets such as offices and retail outlets. They raise money from issuing
units, which are listed and publicly traded. You can invest in a minimum of two units. The REIT
distributes the rental income earned, after expenses, as dividends. At least 90 per cent of the
profits have to be distributed, giving a regular income for investors. Professional managers will
handle operational issues such as building maintenance and managing tenants. Property held
Fin10 from www.finqa.in | info@finqa.in |+91 96500 65242

All Rights Reserved

by REITs may also be sold and reinvested in other assets. Any gains can go to unit holders. If you
are a safe player, REITs with their diversified portfolio will fit your bill better than land or
mega-projects.
Recently, the Government of India has provided tax benefits to boost real estate investment
trusts (REITs). Given the ticket size, not many can invest in large properties but through REITs,
one can invest smaller sums, which will be added to the larger pool. The one advantage that
REITs have is that the underlying asset, the property, also appreciates in value apart from
providing rental income. REITs could be an option for those who want to diversify their
portfolios into real estate.

# 19: Types of investment vehicles: Equity?


Key Message: What is equity investment? What are the various ways of getting equity
exposure?
Equity (stocks) is one of the principal asset classes. In general, you can think of equity as
ownership in any asset after all debts associated with that asset are paid off. For example, a car
or house with no outstanding debt is considered the owners equity because he or she can
readily sell the item for cash. Stocks are equity because they represent ownership in a
company.
People who invest into equity market without fundamental knowledge of investing claim that
stock market as a place where you always lose money. If this was true then we would not have
seen billionaires such as Warren Buffet, Rakesh Jhunjhunwala, Peter Lynch, Thomas Rowe Price,
John Templeton etc. They all were patient investors and did not get perturbed by market
volatility. Once they identified value in a stock and invested, they stick to it as long as there
was no fundamental negative change even if the particular stock went out-of-favour in the
market. In fact, they used volatility as an opportunity to buy more as they believed that sooner
or later, the market would recognise the inherent value and the stock would bounce back.
Equity is an important component of ones portfolio. Equity is a long term investment, ideally
the investment horizon should be atleast 7 years. Lot of people claim that investing into an FD
is risk-free and equity is too risky. Think about it, with an inflation of more than 8%, an after tax
FD return of ~6% isnt risk free after all.

Fin10 from www.finqa.in | info@finqa.in |+91 96500 65242

All Rights Reserved

Return as of June 2014

Returns

NSE nifty

1 year

6.8%

3 year

RESIDEX index

Gold

Fixed Deposit

0.5%

-18.5%

9.5%

0.9%

16.8%

5.4%

9.2%

5 year

16.3%

8.6%

11.6%

9.5%

10 year

12.9%

16.3%

14.6%

8.0%

(NHB) Delhi

There are various ways you can get equity exposure in your investment portfolio:
Direct Equity Applying stock valuation techniques successfully takes years of practice. Do not
try to invest directly into equity market unless you have years of experience. Ideally work with a
financial advisor and in parallel build your knowledge with respect to selected stocks. There is
usually a brokerage charge between 0.25%-0.75% on each transaction.
Mutual funds is a pool of money from numerous investors who wish to save or make money
just like you. Investing in a mutual fund can be a lot easier than buying and selling individual
stocks on your own. Investors can sell their units when they want. Each funds investments are
chosen and monitored by qualified professionals who use this money to create a portfolio. In
return fund manager charges a maximum of 2.5% of the investment amount as fee (in case of
equity funds) for managing the funds.
Exchange Traded Funds (ETFs) invest in stocks that comprise an index. The proportion in which
it will allocate money may be the same as individual stocks weight in the index. For example, a
Nifty ETF will invest in 50 stocks comprising the Nifty, most likely in accordance with the weight
of individual stocks in the index. Since the selection and weight is decided by the index itself,
there is no active manager to manage your investments, hence management fees of ETFs is
very low. If you are a first-time investor and not accustomed to the market, we would
recommend you to take the passive route, that is, invest in ETFs.
New Pension Scheme (NPS) allows you to choose maximum of 50% allocation to equity. NPS is
a very good scheme to save for your retirement especially if the scheme is sponsored by your
employer. We will discuss details of NPS in retirement section.
ULIPs Unit Linked Insurance Policy (ULIP) is a product offered by insurance companies that
unlike a pure insurance policy gives investors the benefits of both insurance and investment
under a single integrated plan. Lot of people think that insurance linked investment plans are
safe. Reality is that your premium on your insurance policy is also invested into equity market.
Ulips have been the most sold, as also the most mis-sold, financial products in the past 5-6
Fin10 from www.finqa.in | info@finqa.in |+91 96500 65242

All Rights Reserved

years. ULIPs may not be the best option to get equity exposure since they charge exorbitant
management fees (>12% for first 3-5 premium payments).
Depending upon your risk profile and your objectives equity allocation may be decided. There
are various ways of getting equity exposure as mentioned above. For the first time investor, a
combination of ETFs, Mutual funds and NPS may be a preferred option. However, if you are an
experienced investor, a combination of direct equity and mutual funds can grow your wealth
even better.

# 20: Types of investment vehicles: Mutual Funds?


Key Message: Different people have different notion about mutual funds. This email will address some of
the basics of mutual fund investment.

Quite simply, a mutual fund is a mediator that brings together a group of people and invests
their money in stocks, bonds and other securities.
Each investor owns shares, which represent a portion of the holdings of the fund. Thus, a
mutual fund is one of the most viable investment options for the common man as it offers an
opportunity to invest in a diversified, professionally managed basket of securities at a relatively
low cost. Investing in a mutual fund offers you a gamut of benefits:
Small investments: With mutual fund investments, your money can be spread in small bits
across varied companies. This way you reap the benefits of a diversified portfolio with small
investments.
Professionally managed: The pool of money collected by a mutual fund is managed by
professionals who possess considerable expertise, resources and experience. Through analysis
of markets and economy, they help pick favourable investment opportunities.
Spreading risk: A mutual fund usually spreads the money in companies across a wide spectrum
of industries. This not only diversifies the risk, but also helps take advantage of the position it
holds.
Transparency and interactivity: Mutual funds clearly present their investment strategy to their
investors and regularly provide them with information on the value of their investments. Also, a
complete portfolio disclosure of the investments made by various schemes along with the
proportion invested in each asset type is provided.
Liquidity: Closed ended funds can be bought and sold at their market value as they have their
units listed at the stock exchange. In addition to this, units can be directly redeemed to the
mutual fund as and when they announce the repurchase.

Fin10 from www.finqa.in | info@finqa.in |+91 96500 65242

All Rights Reserved

Choice: A wide variety of schemes allow investors to pick up those which suit their risk / return
profile.
Regulations: All the mutual funds are registered with SEBI. They function within the provisions
of strict regulation created to protect the interests of the investor.
Imagine you had an option of investing into good company shares and that too managed by
someone who is an expert in picking shares. Similarly, mutual funds provide you a platform to
start with small investment and that investment is managed by experts who guide as well as
execute the transactions for you.

# 21: Asset allocation defines your return


Key Message: If you invest in the best equity mutual fund and equity market as a whole crashes,
your selected product cannot beat the market. Therefore right asset class allocation defines
your return and not the product selection.
Asset allocation is essentially an investment strategy to stabilize risks and returns by choosing
investment instruments according to your financial goals, risk tolerance and time horizon. Asset
classes have different levels of risk and return variability. Each asset class may perform
differently over time. Successful asset allocation requires finding the proper mix of assets to
balance reward with an acceptable level of risk.
Asset allocation is critical for long-term investing and financial planning as it can help absorb
the impact of market fluctuations and balance your tolerance for risk. A downside of a specific
asset class is usually neutralised by an upside of another asset class. This way you can enjoy the
upside and de-risk the downside to a great extent.
Prudent asset allocation can help you ride out the ups and downs of long-term market
performance. No single asset class will outperform another consistently and no single asset
allocation strategy may be right for everyone. Some investments may be up while others may
be down helping minimize the overall potential impact of market decline and enable you to
reach your retirement goals smoothly.
Prudent asset allocation can help you balance your appetite for risk within your timeframe and
financial goals. This requires assessing, adjusting and tracking your investments regularly.
Assess your portfolio Assess your portfolio allocation regularly to make sure it matches your
risk tolerance, time horizon and financial goals and needs.

Fin10 from www.finqa.in | info@finqa.in |+91 96500 65242

All Rights Reserved

Adjust your allocation Adjust your allocation mix and re-align it to your financial goals based
on your risk tolerance and investment horizon.
Track your investments Revisit your asset allocation regularly to make sure your investments
are aligned with your financial goals, since your investment timeframes and risk tolerance may
change over time.
A quarterly financial check-up for the short term and a three-year long-term horizon check to
make sure your investments are aligned with your risk tolerance and long-term financial goals is
usually recommended. However, you need to review your portfolio when you anticipate a
major life-event.

# 22: Why should one have a diversified portfolio?


Key message: What are the various classes of investment and how does diversifying across
various asset classes help?
A portfolio is made up of several investment options across asset classes. The construction of
the portfolio involves allocating money to various asset classes. It is this decision which
determines how much of the assets need to be distributed over the following asset classes with
different characteristics. It is very difficult to determine in a year which particular asset class
would be the best performing one. Investing in only one class of asset could prove to be risky.
Distributing your investment among various asset classes reduces the overall risk in terms of
the variability of returns for a given level of expected return. Therefore, having a mixture of
asset classes is more likely to meet the investors expectations in terms of amount of risk and
possible returns.
Asset classes like equity and real estate provide long term growth but come with short term
volatility. Asset classes like deposits, bonds and other fixed income securities provide income.
The allocation between growth and income depends on the investor needs, and therefore,
portfolio return is driven by the financial goals of the investor.

Assets

Risk

Return

Primary objective

Corporate/Government bonds

Low

Low

Income

Fixed deposit

Low

Low

Income

Gold

High

High

Growth

Fin10 from www.finqa.in | info@finqa.in |+91 96500 65242

All Rights Reserved

Equity

High

High

Growth

Real Estate

High

High

Growth

As you can see, higher the risk higher is the return. In order to grow your portfolio after
considering the inflation, you must allocate some portion to growth assets such as gold, equity
and real estate. However, how much to allocate across various assets would depend upon your
objective.
When a need can be expressed in terms of the sum of money required and the time frame in
which it would be needed, we call it a financial goal. When a financial goal is set, its monetary
value and the future date on which the money will be required is first defined. This goal
definition indicates the amount of investment value that needs to be generated on a future
date. It is normal to include assumptions for expected inflation rate while defining a future goal.
Then the return that the portfolio should generate to achieve the targeted sum can be
ascertained, after understanding how much the investor can save for the goal.
Lets take two scenarios:
1. House down payment in 1 year for Rs 5,00,000. Current bank balance Rs 125,000. What
should be the amount of monthly savings?
This can be calculated through excel:

Rs 29,409 should be saved on a monthly basis. Please note that this amount should be saved in
a safe investment since the time horizon is just 12 months. Therefore, ideally a combination of
debt mutual fund and fixed deposit is most suitable.
2. Marriage of daughter after 25 years, current cost Rs 40,00,000. How much needs to be
saved on a monthly basis?
This can be calculated through excel:
Lets say the inflation is 8% for next 25 years, therefore the future cost of marriage in 25 years
would be = (1.08) ^ 25 X 40,00,000 = Rs 2,73,93,900. Looks like an exorbitant amount, right?
But how much should be the monthly savings for next 25 years?

Fin10 from www.finqa.in | info@finqa.in |+91 96500 65242

All Rights Reserved

Rs 20,646 should be saved on a monthly basis to achieve Rs 2.74 crores after 25 years @10 %
per year. Please note that this amount should be saved in combination of various assets
including growth assets. Therefore, ideally a combination of debt mutual fund, equity mutual
funds, real estate and gold fund is most suitable.

In order to decide your asset allocation, understanding of your goal, time horizon and the
expected return is very important. Before you even decide where you want to invest, knowing
why you are investing is even more critical.

# 23: What is your risk profile ?


Key Message: Everyone has an attitude towards life. This attitude is also reflected on how an individual
manages his/her investments. Today you will learn how to assess your risk profile.

It is human nature to want the highest return possible. However, return is just one of the
factors you need to consider when selecting an investment portfolio. Equally important is how
comfortable you are with fluctuation in market values, your requirements for regular income
versus capital growth and your investment time frame.
Risk profiling is a process for finding the optimal level of investment risk for an
individual considering the risk required, risk capacity and risk tolerance, where,

Risk required is the risk associated with the return required to achieve an individuals goals
from the financial resources available,

Risk capacity is the level of financial risk an individual can afford to take, and
Risk tolerance is the level of risk an individual is comfortable with.
This risk profiler helps in determining your tolerance to risk and how that relates to particular
investments risk profile is a summary of your current situation, which is likely to change over
time. You should periodically review your profile to ensure it remains consistent with your
circumstances. Please note that risk profile should only be used as a guide and not a substitute
for a detailed financial plan.
Click here to calculate your Risk Profile.( http://finqa.in/risk-profile/ )

Fin10 from www.finqa.in | info@finqa.in |+91 96500 65242

All Rights Reserved

# 24: Understanding your risk profile


Key Message: Learn what your Risk Profile tells you about yourself.
To assess your risk personality, we asked you to answer few questions in the previous lesson.
The analysis it provides can help you understand what sort of risk you are willing to take, what
sort of investments to stay away from and why you make the decisions you do when it comes
to finances. While no psychometric test can completely describe you, this questionnaire gives
you an insight into how comfortable you are with risk. Broadly there are five risk categories:
Conservative investors are investors who want stability and are more concerned with
protecting their current investments than increasing the real value of their investments. A
conservative investor is generally seeking to preserve capital and as a trade-off is usually
prepared to accept lower investment returns.
Moderately conservative investors are investors who want to protect their capital and achieve
some real increase in the value of their investments. This investor is usually seeking a
diversified investment portfolio with exposure to a broad range of investment sectors.
Moderate investors are long-term investors who want reasonable but relatively stable growth.
Some fluctuations are tolerable, but investors want less risk than that attributable to a fully
equity based investment.
Moderately aggressive investors are long-term investors who want good real growth in their
capital. A fair amount of risk is acceptable. They are generally willing to trade some risk for
greater long-term returns and typically will have a longer investment objective.
Aggressive investors are long-term investors who want high capital growth. Substantial year-toyear fluctuations in value are acceptable in exchange for a potentially high long-term return. An
aggressive investor is comfortable accepting high volatility in their capital value, with the risk of
short to medium-term periods of negative returns. They are willing to trade higher risk for
greater long-term returns and typically will have a long investment objective.
Please note that your risk profile cannot completely describe how you will or should feel about
any particular financial matter. Your choice on the level of risk to take in your financial matters
should also take into account:

Your timeframes how much time do you have until your bigger goals? Longer time
frames allow you to take greater levels of risk because the fluctuations even out over time.
Life Stage various seasons in life have an impact on the level of risk that is appropriate.
When there are others dependant on you, the level of risk taken will need to be lower.
Partners risk profile where a partner is involved the level of risk should reflect both
partners risk tolerances rather than just one.

Fin10 from www.finqa.in | info@finqa.in |+91 96500 65242

All Rights Reserved

Your risk personality assessment should be viewed as information for you to include in your
decisions on financial matters, not as a constraint on what you should do.

Action item:
While it maybe tempting to invest in riskier investments for higher returns, its better to invest
based on your risk profile.
Know your appetite for risk http://finqa.in/risk-profile/.

# 25: What should my portfolio be like ?


Key Message: We combine our understanding of all the investment options and asset classes to
create a portfolio.
Investors are classified broadly into categories based on the risk and return profiles. Indicative
portfolios for these profiles may be constructed as follows:

Moderately

Conservative

Short term bills

20%

15%

10%

7.5%

5%

Long term debt

60%

50%

40%

25%

10%

Gold

10%

10%

10%

7.5%

5%

Commodities

0%

0%

0%

5%

10%

Large cap equity

10%

20%

30%

30%

30%

Mid cap equity

0%

5%

10%

15%

20%

Real Estate

0%

0%

0%

10%

20%

Total

100%

100%

100%

100%

100%

Conservative

Moderate

Moderately

Asset

Aggressive

Aggressive

These are broad guidelines for long term investors. However, if your investment horizon is less
than three years, you must choose income assets.
Fin10 from www.finqa.in | info@finqa.in |+91 96500 65242

All Rights Reserved

Asset classes like equity and real estate provide long term growth but come with short term
volatility. Asset classes like deposits, bonds and other fixed income securities provide income.
The allocation between growth and income depends on the investor needs, and therefore,
portfolio return is driven by the financial goals of the investor.

Assets

Primary objective

Short term bills

Income

Long term bonds

Income

Gold

Growth

Commodities

Growth

Large cap equity

Growth

Mid cap equity

Growth

Real Estate

Growth

We will briefly discuss what qualifies under debt, gold, equity and real estate.
Debt Most of the people think that investing into an FD is the only risk free investment. Well,
all debt oriented assets are driven by same factors i.e. inflation and interest rate. Therefore
PPF, NSC, Post Office schemes and Fixed deposit fall under the same bracket. One other
product that people are fond of is Insurance linked investments especially LIC. Please note that
LIC generates 6-6.25% p.a. after tax return. Therefore, even investing in LIC would qualify as
debt investment.
Gold As an investment, one can purchase it as coins, bars, jewellery, or through mutual funds
known as gold ETFs (Exchange Traded Funds). However, the best way to invest in gold is using
gold exchange traded funds. This route of investment in gold decreases the cost and taxation,
and gives safety against theft. The liquidity and divisibility is good.
Real Estate Never invest in real estate with less than 8 years of time horizon. There are no
short term gains in real estate. Do not get lured by various builders coming up with guaranteed
schemes. Study the project and assess past record of the builder. Last but not the least make
sure the project is located at an excellent location.
Equity Time horizon for equity investment is more than 8 years. Performing various analysis
and researching good stocks is a skilful task. If you do not have the right skill, you can hire a
financial advisor who can help you make the right decision.
Fin10 from www.finqa.in | info@finqa.in |+91 96500 65242

All Rights Reserved

# 26: How to create a savings fund of Rs 12+


crores ?
Satya, 25 years, engineer earns Rs 50,000 per month after taxes. He is wondering what he should be
saving on a monthly basis to create a huge corpus. This email will help people like Satya answer a
pertinent question HOW MUCH SHOULD I SAVE?
Yes, there are legal and safe ways to get rich over a period of time.
If your salary grows at 10% a year, you save 20% of your income, and your savings grow at 10% per
annum, you can have a corpus of Rs 12 crores when you retire after 35 years (See illustration below to
see how your money will grow over time).
However, its not as easy as it sounds else everyone would have been a millionaire by the time they
retired. The good news is that with a bit of planning, it is achievable.
So what are the key challenges?
Saving 20% of your salary year on year
Achieving an average annual growth of 10% per annum.
You will need to plan your expenses in such a way that you can continue to save 20% of your income
every year.

Secondly, your savings need to be invested in such a way that they provide you with a return of atleast
10% per annum.
The key is to start early as the power of compounding also helps you. (A person who starts saving 5
years later ends up losing almost 2 crores)

# 27: Benefit from the Power of Compounding!


As time progresses some things just get better..
Red wine gets an enhanced taste while aged cheese is so much tastier.
Women tend to look more beautiful and generally men tend to get wittier.

Fin10 from www.finqa.in | info@finqa.in |+91 96500 65242

All Rights Reserved

Dont you agree?


Now whats with Money?? It grows.. grows with time
And why?? Because it gets compounded.
So whats compound interest exactly? Its earning interest for interest already earned. In other words,
your interest on savings also earns interest. Suppose you deposit some money in your account today,
there will be an increase in the value of it after a year. And, thats because it has earned interest.
The journey to wealth can begin with even keeping aside the smallest amount each day.
Dont believe?? The picture below says a thousand words.

Rs. 15000/- invested per month for 20 years will turn into more than Rs 1.13 crores.
On the contrary, if you left it in a savings account, it would be worth something in the region of Rs 36
lakhs.
And the best part about enjoying the power of compounding is:

You dont have to be rich to see its benefit; but you can become rich in the process
You can start from today; so why not start now?
The more time you give it, the fatter balance you can enjoy with, later.

Fin10 from www.finqa.in | info@finqa.in |+91 96500 65242

All Rights Reserved

# 28: Why should one invest in Mutual Funds ?


Key Message: Know why you should invest into mutual funds and why mutual funds are a safe
investment option.
Investors spread out and minimize their risk up to a certain extent by purchasing units in a
mutual fund instead of buying individual stocks or bonds. By investing in a large number of
assets, the shortcomings of any particular investment are minimized by gains in others.

Economies of scale: Mutual funds buy and sell large amounts of securities at a time. This

helps reduce transaction costs and bring down the average cost of the unit for investors.
Professional management: Mutual funds are managed by thorough professionals. Most
investors either dont have the time or the expertise to manage their own portfolio. Hence,

mutual funds are a relatively less expensive way to make and monitor their investments.
Liquidity: Investors always have the choice to easily liquidate their holdings as and when
they want.
Simplicity: Investing in a mutual fund is considered to be easier as compared to other
available instruments in the market. The minimum investment is also extremely small,
where an SIP can be initiated at just Rs.50 per month basis.

When it comes to mutual funds, putting all your eggs in a single basket is never a wise option.
This is due to the market volatility and the risks that come with it.
But you can always minimise them by distributing your investments among various financial
instruments, industries and other categories. This not only buffers the impact of a market
downturn, but also allows for more potential rewards by offering a broader exposure to various
stocks and sectors.

# 29: What are the different types of Mutual


Funds ?
Key Message: Before you even plan to invest into mutual funds, you must know what the broad
categories of mutual funds are?
Every investor has a different investment objective. Some go for stability and opt for safer
securities such as bonds or government securities. Those who have a higher risk appetite and
yearn for higher returns may want to choose risk-bearing securities such as equities. Hence,
mutual funds come with different schemes, each with a different investment objective.
Fin10 from www.finqa.in | info@finqa.in |+91 96500 65242

All Rights Reserved

There are hundreds of mutual fund schemes to choose from. Hence, they have been
categorized as mentioned below.
1. By structure: Closed-Ended, Open-Ended Funds, Interval funds.
2. By nature: Equity, Debt, Balance or Hybrid.
3. By investment objective: Growth Schemes, Income Schemes, Balanced Schemes, Index
Funds.
1. Types of mutual funds by structure
Close ended fund/scheme: A close ended fund or scheme has a predetermined maturity period
(eg. 5-7 years). The fund is open for subscription during the launch of the scheme for a specified
period of time. Investors can invest in the scheme at the time of the initial public issue and
thereafter they can buy or sell the units on the stock exchanges where they are listed. In order
to provide an exit route to the investors, some close ended funds give an option of selling back
the units to the mutual fund through periodic repurchase at NAV related prices or they are
listed in secondary market.
Open ended fund/scheme: The most common type of mutual fund available for investment is
an open-ended mutual fund. Investors can choose to invest or transact in these schemes as per
their convenience. In an open-ended mutual fund, there is no limit to the number of investors,
shares, or overall size of the fund, unless the fund manager decides to close the fund to new
investors in order to keep it manageable. The value or share price of an open-ended mutual
fund is determined at the market close every day and is called the Net Asset Value (NAV).
Interval schemes: Interval schemes combine the features of open-ended and close-ended
schemes. The units may be traded on the stock exchange or may be open for sale or
redemption during pre-determined intervals at NAV related prices. FMPs or Fixed maturity
plans are examples of these types of schemes.
2. Types of mutual funds by nature
Equity mutual funds: These funds invest maximum part of their corpus into equity holdings.
The structure of the fund may vary for different schemes and the fund managers outlook on
different stocks. The Equity funds are sub-classified depending upon their investment objective,
as follows:
Diversified equity funds
Mid-cap funds
Small cap funds
Sector specific funds
Tax savings funds (ELSS)
Equity investments rank high on the risk-return grid and hence, are ideal for a longer time
frame.

Debt mutual funds: These funds invest in debt instruments to ensure low risk and provide a
stable income to the investors. Government authorities, private companies, banks and financial
Fin10 from www.finqa.in | info@finqa.in |+91 96500 65242

All Rights Reserved

institutions are some of the major issuers of debt papers. Debt funds can be further classified
as:
Gilt funds
Income funds
MIPs
Short term plans
Liquid funds
Balanced funds: They invest in both equities and fixed income securities which are in line with
pre-defined investment objective of the scheme. The equity portion provides growth while debt
provides stability in returns. This way, investors get to taste the best of both worlds.

Types of mutual funds by investment objective


Growth schemes: Also known as equity schemes, these schemes aim at providing capital
appreciation over medium to long term. These schemes normally invest a major portion of their
fund in equities and are willing to withstand short-term decline in value for possible future
appreciation.
Income schemes: Also known as debt schemes, they generally invest in fixed income securities
such as bonds and corporate debentures. These schemes aim at providing regular and steady
income to investors. However, capital appreciation in such schemes may be limited.
Index schemes: These schemes attempt to reproduce the performance of a particular index
such as the BSE Sensex or the NSE 50. Their portfolios will consist of only those stocks that
constitute the index. The percentage of each stock to the total holding will be identical to the
stocks index weight age. And hence, the returns from such schemes would be more or less
equivalent to those of the Index.

# 30: What are Equity Mutual Funds?


Key Message: Everybody claims that equity investment is risky but first know different types of
equity mutual funds.
Equity funds invest in equity instruments such as shares. Most equity funds are created with
the objective of generating long term growth and capital appreciation. The investing horizon for
equity products is also longer, given that equity as an asset class may be volatile, in the short
term. Stocks are classified on the basis of market cap and industry.
Classification of equity funds is based on the type of stocks they invest in. Hence, equity funds
may be diversified funds, large cap funds; mid and small cap funds, sector funds, and thematic
funds, depending upon the sectors and the market segment that they invest in.

Fin10 from www.finqa.in | info@finqa.in |+91 96500 65242

All Rights Reserved

Large-cap funds: Large-cap funds are, typically, the least risky funds. These companies are
among the least volatile companies as they are mostly in mature businesses. You must allocate
highest to this category of investment.
Mid- and small-cap funds: These funds are riskier than large-cap funds. They invest in smallsized companies that are in their growing stages. Since these companies are in their growing
stages, they can get volatile in an uncertain market. These are high-risk companies; they
typically rise more than large-cap funds in rising markets, but fall more than large-cap
companies in falling markets.
Diversified equity funds invest across various sectors, sizes and industries, with the objective of
beating a broad equity market index. These funds feature lower risk as the benefit of
diversification kicks in and are suitable for investors with long investment
horizons. Underperformance of one sector or stock may be made up for by the outperformance of any one or more of the other sectors or stocks.
Thematic Equity Funds invest in multiple sectors and stocks pertaining to a specific theme.
Themes are chosen by the fund managers who believe these will do well over a given period of
time based on their understanding of macro trends and developments. Funds may be based on
the themes of infrastructure growth, commodity cycles, public sector companies, multi-national
companies, rural sector growth, businesses driven by consumption patterns and serviceoriented sectors. These funds run a higher concentration risk, as compared to a diversified
equity fund but are diversified within a particular theme. Such fund offer a higher return if the
specific theme they focus on does better than the overall market.
Sector funds are available for sectors such as information technology, banking, pharma and
FMCG. We know that sector performances tend to be cyclical. The return from investing in a
sector is never the same across time. For example, Auto sector, does well, when the economy is
doing well and more cars, trucks and bikes are bought. Such funds typically feature, high risk,
and are unsuitable for a longer horizon. Investments in sector funds have to be timed well.
Value funds identify under-priced stocks that are not in focus but have the potential to do well
in the long run. Value strategy plays out over the long run and is potentially a low risk
proposition.
Growth funds tend to focus on stocks, which show the potential for a higher earnings growth
compared to their peers. These funds are aggressive. It is therefore, obvious that growth funds
tend to do well, when the equity market is rising, when the companies are doing well, and their
earnings are high.
Index funds are passively managed, where the fund manager does not take a call on stocks or
the weights of the stocks in the portfolio, but simply replicates a chosen index. Replicating an
index means, holding all the same stocks, in exactly the same weightage as in the index. Index
funds could track the broader indices, such as Nifty and Sensex, or could track the sectorspecific indices such as BSE IT (Technology) or Bankex (Banking). First time equity investors,
who do not like to take a risk on the fund with respect to the benchmark, are typically
recommended index funds. Index funds will always deliver a return equal to the return on the
Fin10 from www.finqa.in | info@finqa.in |+91 96500 65242

All Rights Reserved

benchmark. A slight difference in return could be attributed to the expense ratio which is
charged by the mutual fund. However, expense ratio on index funds is considerably lower that
is 0.75% as maximum, as compared to, 2.50% for actively managed equity funds.
Dividend yield funds invest in stocks that have a high dividend yield. Dividend yield is
computed as a ratio of dividend pay-out in rupee terms to the current market price. Dividend
received represents regular income from equity. Hence, such funds are also known as equity
income funds. Stocks with high dividend yield tend to be less volatile compared to the regular
equity growth stocks as investors keep their focus on the regular dividend income coming from
such stocks.
Special situations funds invest in stocks of companies undergoing special situations, such as, a
turnaround, merger and acquisition, takeover, and the like. The idea is to buy these companies
when they are under-valued, and make profits when the share price of such companies rises on
the occurrence of the special event. Dividend yield and special situation-based investing, is
conservative and may require patient, long investment horizon.
Equity linked savings schemes (ELSS) is a special category of diversified equity funds designated
as ELSS, at the time of launch. Investment in ELSS to the extent of Rs. 1 lakh in a year enjoys a
tax deduction under Section 80C of the Income Tax Act. Investors can buy the units to claim tax
deduction at any time of the financial year. An ELSS can be offered as an open-ended scheme,
in which case, a fund house can have only one such scheme. Funds can also offer ELSS as a
closed-end scheme. Investment in both the open and closed end ELSS is subject to a 3-year
lock-in. The lock-in period will apply from the date of purchase of units. During the lock-in
period investors cannot sell, redeem, pledge, transfer, or in any manner alter their holding in
the fund. An ELSS schemes investment portfolio is quite similar to a diversified equity fund.
Rajiv Gandhi Equity Savings Scheme (RGESS) offers a rebate to first time retail investors with
annual income below Rs 10 lakhs. 50% of the amount invested (excluding brokerage, securities
transaction tax, service tax, stamp duty and all taxes appearing in the contract note) can be
claimed as a deduction from taxable income in a single financial year. Although any amount can
be invested in such scheme, the benefit is only available up to Rs. 50,000. Thus, the deduction is
limited to 50% of Rs 50,000, i.e., Rs 25,000. Once an RGESS deduction is claimed in a financial
year, no further RGESS deduction can be claimed by that investor in any future years. Mutual
funds announce specific schemes that are eligible for the RGESS deduction.

# 31: What are Debt Mutual Funds?


Debt Funds invest in debt securities, may invest in money market securities or in longer term
debt securities, or a combination of the two. The primary investment objective of liquid and
debt funds is regular income generation. However, since the longer term debt markets offer the

Fin10 from www.finqa.in | info@finqa.in |+91 96500 65242

All Rights Reserved

scope for capital growth, debt funds are offered along the yield curve, spanning very short term
to long term products.
Short Term Debt Funds
Money Market or Liquid Fund is a mutual fund for very short term investment. The primary
source of return is interest income. Liquid fund is a very short-term fund and seeks to provide
safety of principal and superior liquidity. It does this by keeping interest rate and credit risk low
by investing in very liquid, short maturity fixed income securities of highest credit quality.
It is suitable for retail investors who:

Are looking for very safe investment option and,

Have less than 3-6 months investment horizon.


Ultra short-term plan also known as treasury management funds, or cash management funds.
They invest in money market and other short term securities of maturity up to 365 days. The
objective is to generate a steady return, mostly coming from accrual of interest income, with

minimal NAV volatility.


It is suitable for retail investors who:

Are looking for very safe investment option and,


Have less than 1 year investment horizon.

Short Term Plan combines short term debt securities with a small allocation to longer term
debt securities. Short term plans earn interest from short term securities and interest and
capital gains from long term securities. The volatility in returns will depend upon the extent of
long-term debt securities in the portfolio. Short term funds may provide a higher level of
return than liquid funds and ultra-short term funds, but will be exposed to higher risks.
It is suitable for retail investors who:

Are comfortable taking some level of risk for an extra return and,

Have investment horizon between 1-3 years.


Long Term Debt Funds
Market interest rates and value of a bond are inversely related, any fall in the interest rates
causes a gain in a bond portfolio and vice versa. Therefore in a falling interest rate scenario,
when investors in most fixed income products face a reduced rate of interest income, long term
debt funds post higher returns. The extent of change in market prices of debt securities is

linked to the average tenor of the portfolio Higher the tenor, greater the impact of changes in
interest rates.
It is suitable for retail investors who:

Are comfortable taking some level of risk for an extra return and,

Have atleast 3 year investment horizon.


Fin10 from www.finqa.in | info@finqa.in |+91 96500 65242

All Rights Reserved

Apart from the above mentioned broad categories of debt funds, there are other debt funds to
meet specific needs of investors:
Credit opportunity funds a new category that has emerged among the debt funds in India
recently. The fund focuses on accrual income from corporate bonds of tenor 3 to 5 years. The
investors earn a high return owing to high return.
It is suitable for retail investors who:

Are looking for regular income from a debt fund. And,


Have at least 3 year investment horizon.

Gilt Funds invest in government securities of medium and long-term maturities. There is no risk
of default and liquidity is considerably higher in case of government securities. However, Prices
of government securities are very sensitive to interest rate changes. Long term gilt funds have a
longer maturity and therefore, higher interest rate risk as compared to short term gilt funds.
It is suitable for retail investors who:

Are looking for very safe investment option and,

Have at least 5 year investment horizon.


Dynamic debt funds funds do not have any restriction with respect to security types or

maturity profiles that they invest in.


It is suitable for retail investors who:

Are comfortable taking some level of risk for an extra return and,
Want a fund manager to actively manage their debt portfolio.

Fixed maturity plans (FMPs) are closed-end funds that invest in debt securities with maturities
that match the term of the scheme. The debt securities are redeemed on maturity and paid to
investors. FMPs are issued for various maturity periods ranging from 3 months to 5 years. The
return of an FMP depends on the yield it earns on the underlying securities. They typically
invest only in fixed income securities like debentures of issuers with a good credit rating. FMPs
are a very good substitute of a bank Fixed deposit.
There have been few significant changes in the recent budget (July 2014) for debt funds.
According to the new tax rules on debt funds, the investor has to hold the funds for at least 36
months to qualify for the 20 per cent capital gains tax. If the debt mutual units are held for less
than 36 months, it would be taxed according to the investors tax slab. This brings debt funds at
par with bank fixed deposits in terms of taxation, if it is held for less than 36 months. However,
if your investment horizon is more than 36 months, debt funds are still an attractive option.

Fin10 from www.finqa.in | info@finqa.in |+91 96500 65242

All Rights Reserved

# 32: What are Hybrid Funds?


Mutual fund products invest in a combination of debt and equity in varying proportions.
Predominantly debt-oriented hybrids invest mostly in the debt market, but invest 5% to 35% in
equity. The objective in these funds is to generate income from the debt portfolio, without
taking on the risk of equity. A small allocation to equity provides a kicker to the overall return.
Predominantly equity-oriented hybrid funds have up to 35% in debt for income and stability. A
fund must have a minimum of 65% in equity in order to qualify for tax benefits as an equityoriented fund. Equity-oriented funds have a small allocation to debt to reduce risk from equity.
There are also Dynamic asset allocation funds which have the flexibility to invest 0% to 100% in
equity and debt depending upon the fund managers view of the market scenario. Thus, these
funds have the ability to work as a 100% debt fund or a 100% equity fund, depending upon fund
manager outlook.
Debt-oriented hybrids invest minimum of 70 to 95% in a debt portfolio. The debt component is
conservatively managed with the focus on generating regular income, which is generally paid
out in the form of periodic dividend. The allocation to equity is kept low and primarily in large
cap stocks, to enable a small increase in return, without the high risk of fluctuation in NAV.
Debt-oriented hybrids are designed to be a low risk product for an investor. These products are
suitable for traditional debt investors, who are looking for an opportunity to participate in
equity markets on a conservative basis with limited equity exposure.
Equity-oriented hybrids invest in the equity market, but invest up to 35% in debt, so that some
income is also generated. Balanced funds are designed as equity-oriented funds. Balanced
funds are suitable to those investors who seek the growth opportunity in equity investment,
but do not have a very high risk appetite. Balanced funds typically have an asset allocation of
65-80% in equity, and 20-35% in debt. The proportions in equity and debt are managed
tactically by the fund managers based on their view of the markets. In an environment
conducive to equity, balanced funds allocation to equity may be raised to 80% in order to
maximize returns. In a scenario where equity markets are incurring losses, balanced funds
minimize the extent of fall in the value of the portfolio by reducing equity exposure and
increasing debt exposure. Hence, it can be safely said that balanced funds work like parachutes
in a falling market.
Fund of Funds (FoF) is a mutual fund that invests in other mutual funds. It does not hold
securities in its portfolio, but other funds that have been chosen to match its investment
objective. These funds can be either debt or equity, depending on the objective of the FoF. A
FoF either invests in other mutual funds belonging to the same fund house or belonging to
other fund houses. FoFs belonging to various mutual fund houses are called multi-manager
FoFs, because the AMCs that manage the funds are different. A FoF is different from a mutual
Fin10 from www.finqa.in | info@finqa.in |+91 96500 65242

All Rights Reserved

fund in its investment strategies. While mutual funds use bottom-up stock selection, a FoF uses
the top-down approach in selecting funds. It looks for funds that fit the need and view that the
FoF manager has. An FoF imposes additional cost on the investor. Also, note that equity FoFs
does not enjoy the tax concessions available to equity funds.
The definition of equity funds in the Income Tax Act refers only to investment in equity shares
of domestic companies. If the fund has more than 65% equity investment and you sell your
units after one year there will be no capital gain tax. In case of a hybrid fund, you may not get
this tax advantage if the fund manager invests less than 65% in equity. Moreover, hybrid funds
may be more expensive than traditional mutual fund classes such as debt fund or equity fund.
Therefore, investment in traditional mutual fund classes maybe better than hybrid funds.
Ideally, if you work with an advisor, he/she should be able to decide on the allocation between
a debt fund and an equity fund.

# 33: What are ETFs? International funds?


Arbitrage funds?
Exchange Traded Funds (ETFs) hold a portfolio of securities that replicates an index and are
listed and traded on the stock exchange. The return and risk on ETF is directly related to the
underlying index or asset. The expense ratio of an ETF is similar to that of an index fund. ETFs
are first offered in a New Fund Offer (NFO) like all mutual funds. Units are credited to demat
account of investors and ETF is listed on the stock exchange. On-going purchase and sale is
done on the stock exchange through trading portals or stock brokers. Settlement is like a stock
trade, and debit or credit is done to the demat account. ETF prices are real-time and known at
the time of the transaction, unlike NAV which is computed end of a business day. Their value
changes on a real-time basis along with changes in the underlying index. First time equity
Investors who do not wish to seek benefits of active portfolio management strategy, and are
satisfied with the returns linked to an index, are likely to find ETFs suitable.
International funds invest in markets outside India, by holding certain foreign securities in their
portfolio. If an international fund invests at least 65% of net assets in domestic equity, and the
rest abroad, only then it will be treated as an equity-oriented fund. Therefore, international
funds that invest in equity shares overseas, will not be classified as equity-oriented funds for
purposes of taxation.
International funds offer reward with respect to portfolio diversification from exposure to
global markets. However, there may be external political events and macro-economic factors
that can cause investments to decline in value. In addition, the investment value may be
impacted by changes in exchange rates.
Fin10 from www.finqa.in | info@finqa.in |+91 96500 65242

All Rights Reserved

International funds should not be more than 10% of the portfolio. Ideally, for new investor
investment into an international fund may be restricted to 5%.
Arbitrage funds aim at taking advantage of the price differential between the cash and the
derivatives markets. Arbitrage is defined as simultaneous purchase and sale of an asset to take
advantage of difference in prices in different markets. The difference between the future and
the spot price of the same underlying is an interest element, representing the interest on the
amount invested in spot, which can be realized on a future date, when the future is sold. Funds
buy in the spot market and sell in the derivatives market, to earn the interest rate differential. A
completely hedged position makes these funds a low-risk investment proposition. They feature
lower volatility in NAV, similar to that of a liquid fund.
Arbitrage funds are classified as equity funds therefore after one year all the earnings are taxfree. If you want to invest into risk-free assets and want to park your money more than one
year, Arbitrage funds are the best option.

# 34: How can I invest in Gold?


Indians are among the largest retail buyers of gold in the world. Even lower income groups buy
gold in small quantities, using them as collateral to take loans when in need. Investment in gold
has the potential to beat inflation over a long period. It is a safe haven when economic growth
is slow and traditional asset classes such as equity and debt are under-performing.
There are various ways how you can get exposure to gold as an investment asset.
1. Gold Jewellery Buying gold in the form of jewellery has its own costs involved. The
primary one is the making charges, which can be to the tune of 10%. There are costs
associated even when you sell jewellery, especially when it is to a different jeweller. If you
are looking at holding the asset for a very long term, other options will be more cost
effective and liquid.
2. Gold bar If you buy gold bar/coins, you will have to sell it to a jeweller who will deduct
charges then. With all these costs involved, holding gold physically will not be cost
effective.
3. Gold Exchange Traded Fund This fund is a passively managed open-ended ETF, which
invests in gold bullion and instruments with gold as underlying, so as to provide investment
returns that, closely track the performance of domestic prices of Gold in the bullion
market. Thus, rising gold prices would be beneficial for the investor. Of course, the actual
returns may be just a tad lower due to the effect of expense ratio on fund management.
Fin10 from www.finqa.in | info@finqa.in |+91 96500 65242

All Rights Reserved

When an investor invests in Gold ETF, the investment is done in physical gold of 99.5%
purity. Hence, impurity risk in gold ETF is absent. Gold ETF allows investors to buy gold in
quantities as low as 1gm. This is done in the form of demat units where each unit
approximately represents the value of 1 gm of Gold. Hence, investors can even use small
amounts to invest in gold.
4. Gold mutual funds A Gold Fund of Fund invests in gold ETF. Although Gold mutual fund is
an easy option to invest in gold, but generally is more expense than Gold ETFs.
5. E-Gold They are similar to gold ETF. Its the only option where units can be converted to
physical gold. You need to open a separate demat and trading account.
The best way to invest in gold is using gold exchange traded funds. This route of investment in
gold decreases the cost and taxation, and gives safety against theft. The liquidity and divisibility
is good. The other way of investing into gold can be through gold fund. Gold savings fund gives
investors indirect way of investing in gold ETF. Even investors who do not have demat accounts
can take exposure in this way to a Gold ETF. Only drawback is the higher expense ratio for the
FoF structure.
Allocation to gold in a strategic portfolio should not be over 5-10%. It may be used primarily to
take advantage of its low correlation with other assets, and the ability to accumulate it in small
lots compared to other alternate assets. Allocation to gold goes up during times of risk and
uncertainty when gold is seen as a store of value.

# 35: What is a Systematic Investment Plan (SIP)?


Key Message: Today you will learn a strategy of growing your money gradually through SIP
mode.
In order to beat inflation you must allocate portion of your investment into equities. There is
one strategy which never fails in equity market and that is the strategy of averaging your cost of
investment. If you invest on a monthly basis in quality stock over a long period of time, you will
always make huge return. There are varieties of ways investment experts try to predict market.
However, there is no one who can be 100% confident on what will happen in future. Therefore,
you should not try to time the market. Keep it simple, invest in equity market irrespective
whether the market in going up or going down. You can achieve this by investing through
Systematic Investment Plan.
Systematic Investment Plan (SIP) is a method of investing a fixed/regular sum every month or
every quarter. With the growing everyday expenses, it becomes difficult to accumulate a
considerable sum which can be invested at one go. But with an SIP, you can start with a modest
Fin10 from www.finqa.in | info@finqa.in |+91 96500 65242

All Rights Reserved

amount of Rs. 5000 every month and this can be invested in any scheme of your choice as most
mutual funds have this facility for their schemes.
The biggest advantage of an SIP is the habit of regular, disciplined savings. Every month, like all
other EMIs, this also gets deducted from the bank account through electronic clearing service,
which is convenient. Another benefit is that when investing through SIP, it is not necessary to
time the market. Investments will be made systematically every month or quarter depending
on the option. It ensures investing in all phases of the market where more units will be
accumulated during a bearish phase and a lesser number of units in a bullish phase. This way,
the investor enjoys the benefit of rupee cost averaging under this method.
Lets say, you started an SIP in June 2013 with Rs. 10,000 every month.

Month

Amount invested (Rs.)

NAV (assumed)

Units allocated

June 2013

10,000

11.0

909.1

July 2013

10,000

11.5

869.6

Aug 2013

10,000

10.0

1,000.0

Sept 2013

10,000

14.0

714.3

Oct 2013

10,000

9.0

1,111.1

Total:

50,000

4,604.1

The average cost per unit is Rs. 50,000/4604.1 = Rs 10.9. However, if you had invested your Rs.
50,000 all at once in June 2013, you would have been allotted 4,545.4 units at the cost of Rs.
11.
Its clear that SIP, with its small investments goes a long way in helping you grow your money
and achieve your goals.
How do you go about starting an SIP? Along with the mutual fund application form, you also
provide bank details for ECS mandate. Rest all is handled by the mutual fund company.

Fin10 from www.finqa.in | info@finqa.in |+91 96500 65242

All Rights Reserved

# 36: What should one keep in mind while


choosing a good Mutual Fund?
Key Message: Best mutual fund scheme does not mean the best in returns, but the one best
suited to your risk profile and goals and the one that is good in its peer group.
The biggest mistake that mutual fund investors make is selecting mutual funds only on the basis
of performance and that too just the recent performance. There are some investors who
consider only the star ratings given by various research agencies. These star ratings can be one
of the factors to look at, but there are many other parameters that one should look into before
finalising a mutual fund portfolio.
The most important first step is to have an investment goal. A fantastic fund selection done
without having an investment goal is completely useless. You should know the reason for your
investment, how long you can be in the investment, at what stage you will re-allocate, etc.
before you make your first investment. Once you know your investment objective, you may
evaluate a fund by parameters as mentioned below.
Performance Ranking
More than the recent or long term performance of any scheme its ranking among peers is very
critical. Firstly, you must compare the mutual fund with its peer group. Secondly, you must
compare the performance of the scheme with its benchmark. You can find the information on
the benchmark in the mutual fund offer document.
The fund which has performed well in one quarter may not perform well in the next quarter.
Funds with a good long term top quartile performance are far superior to a fund scheme which
has one top position and one bottom position.
Total expense ratio
As a fund starts to do well, it should attract a lot of investors, and as its assets increase it should
keep dropping its asset management charges. Look at well managed funds with charges below
1.9% p.a. there are many. Though mutual funds total expense ratio has been capped by SEBI,
still lower the better unless we get some extraordinary return by paying higher expenses for
fund management.
Fund manager tenure and experience
Fund manager plays a very important role in the funds performance. Though it is a process
oriented approach but still fund manager is the ultimate decision maker and his experience and

Fin10 from www.finqa.in | info@finqa.in |+91 96500 65242

All Rights Reserved

view point counts a lot. You should know who the fund manager of the scheme is and what his
past track record is.
If you find that due to change in the fund manager there is considerable effect on the funds
performance which does not suit your risk appetite then you may make a decision to exit.
Scheme asset size
Less AUM in any scheme is very risky as you dont know who the investors are and what
quantum of investments they have in this particular scheme. Exit of any big investor out of any
mutual fund may impact its overall performance very badly and the remaining investors in a
scheme will have to bear the impact. In schemes with larger AUMs this risk gets minimised.
A good fund manager will automatically result in better performance and thus improve the
quartile ranking and would also generate returns better than the benchmark. High scheme
assets will help in reducing the total expense ratio of the scheme. One should review the
current selection every quarter or half yearly.

It is recommended to invest in two to three funds that match and/or complement your
investment objective. This is to avoid dependence on any one fund and avert risks of market
downturns. You can always split the pie as 60:40 with two funds and 40:30:30 in case of 3
funds.

# 37: How to make extra return on your portfolio?


Key Message: How to actively manage your portfolio through combination of various asset
classes.
Based on your risk profile you can decide a long-term allocation of your portfolio. Indicative
portfolios for these profiles may be constructed as follows:

Asset
Class
Cash

Debt

Debt

Moderately

Conservative

Cash

5%

5%

5%

5%

5%

20%

15%

10%

7.5%

5%

55%

45%

35%

20%

15%

Short term
bills
Long term

Conservative

Moderate

Moderately

Asset

Fin10 from www.finqa.in | info@finqa.in |+91 96500 65242

Aggressive

Aggressive

All Rights Reserved

debt
Gold

Equity

Equity

Real
Estate

Gold

10%

10%

10%

7.5%

5%

10%

20%

30%

30%

30%

0%

5%

10%

15%

20%

Real Estate

0%

0%

0%

15%

20%

Total

100%

100%

100%

100%

100%

Large cap
equity
Mid cap
equity

The above list indicates the long-term asset allocation. However, there may be short-term
opportunities in the market. These opportunities may be capitalized by deploying cash to the
asset class that provides short-term gain.
Risk, is inherent, in equity markets, and therefore, an investor investing in an equity fund must
be prepared for some volatility. Performance of different equity funds may vary depending
upon their portfolio composition. Risk inherent in equity funds can be managed through
portfolio construction strategy. This may be accomplished through the extent of diversification,
market segment selection, and fund management style. For example, an equity fund may be
well-diversified in order to control risk. Similarly, large cap funds are considered less risky, as
compared to mid-cap and small-cap funds. Large cap funds are relatively low risk, low return
investments (compared to other equity funds), as large companies tend to be well established
in their businesses with stable growth and earnings. The smaller companies tend to exhibit
higher growth on earnings, depending on the business opportunity and their ability to grow.
However, the risk is relatively higher as smaller companies tend to also feature a higher risk of
inability to withstand downturns and lower liquidity in the stock market. These funds offer a
higher-risk and higher-return variation to large cap funds. Risk and return of equity funds also
varies depending upon market scenario. Large cap funds out-perform mid cap funds in market
fall and recovery, while mid-cap funds out-perform large cap funds in upswing, and
momentum. While Large-cap funds out-performed during the market fall of 2009 and 2010 and
the recovery in 2012, mid cap funds had significantly out-performed large caps, during the
upward momentum phase of 2006 till 2008. Flexi-cap or vari-cap funds tend to exploit such
opportunities. They switch between large and midcap stocks based on the fund managers view
of which style might outperform the benchmarks.
Fin10 from www.finqa.in | info@finqa.in |+91 96500 65242

All Rights Reserved

In case of a debt, the value of debt held in a portfolio, changes with changes in interest rates. In
a falling interest rate scenario, when investors in most fixed income products face a reduced
rate of interest income, long term debt funds gain the most. And in case of a rising interest
rates scenario, long term bond funds fall the most. The extent of change in market prices of
debt securities is linked to the average tenor of the portfolio Higher the tenor, greater the
impact of changes in interest rates and lower the tenor, lower the impact of changes in the
interest rates.
Allocation to gold may be used primarily to take advantage of its low correlation with other
assets. Allocation to gold goes up during times of risk and uncertainty when gold is seen as a
store of value.
Real estate investment should always be a long-term investment and should not be used for
capitalizing any short-term opportunity.

This kind of analytics requires in-depth understanding of both macro and micro economic
variables. In case you do not have a strong finance background, you must initially work with an
advisor who can help you capitalize on any short-term market opportunity.

# 38: How to invest your lump sum money into


Equity market?
If you had invested in Equity mutual fund in early 2008, even after 5 years postelection rally you would have just made sub 5% p.a. return. Learn how to avoid
situations like these.
There is an old saying invest in equity when markets are low, invest in debt when
markets are high. The practical challenge is that you will never get to know that the
market is high or low. As markets are volatile and can go up or down very soon, there
is always risk of losing a big chunk of your investment. Take a case where you want
to invest 10 lacs in Equity Mutual funds and suddenly market crashes for next 2
months. In such a situation, a systematic transfer plan (STP) of mutual fund can
effectively be used to reduce risk.
Systematic Transfer plan is suitable for investors who want to invest lump sum
money in equity schemes in order to avail the potential for higher growth. Firstly,
invest a lump sum amount in a debt-oriented scheme. Specify a desired amount to
Fin10 from www.finqa.in | info@finqa.in |+91 96500 65242

All Rights Reserved

be transferred to any equity schemes of the same asset management company. For
example, you want to invest Rs 10 lac in equity funds and you have selected ICICI Pru
focused bluechip fund and ICICI pru discovery fund. Then you park you money first in
ICICI pru liquid fund and provide instruction to transfer Rs10,000/week each to
selected ICICI Pru focused bluechip fund and ICICI pru discovery fund. Over a period
of one year, your money will be invested into selected schemes. This method of
investing will protect you from investing at high of the market and the investment
cost will be spread across 1 year.
With an STP, you choose a particular amount to be transferred from one mutual fund
scheme to another of your choice. You can go for a weekly, monthly or a quarterly
transfer plan, depending on your needs.
If you are looking at gradually exposing yourself to equities or reducing exposure
over a period of time, then STPs are a good option.

# 39: Myths of Investing directly in the stock


markets
Direct equity investment should be made only if your risk appetite allows. In todays email we
will share some of the myths of direct equity investment.
If you have ever tried to make your own investment decisions over a period of time, chances
are you spent most of your time wondering what you should do next. You also experienced
how time-consuming and distracting that can be. The emphasis here is on keeping things
simple. Before we proceed on stock investing, here are few myths you must be aware of
regarding stock investing.
Myth No.1: The stock market is a form of gambling
In the most basic terms, a share of common stock entitles the owner of that share to a fraction
of what is left over after all other stakeholders in a business have been paid. So, the firm takes
in revenue from customers in return for the firms product, and with that revenue pays for raw
materials, employee wages, energy, supplies, and pays interest on borrowed funds. Whatever is
left over, if anything, belongs to the holders of the firms stock, who are essentially the owners
of the firm. Rise is the share price entirely depends upon the performance of the company i.e.
after meeting all the expenses what is leftover.

Fin10 from www.finqa.in | info@finqa.in |+91 96500 65242

All Rights Reserved

Myth No.2: Stock market predictions are the key to successful investing
The best way to make money in the stock market is to avoid approaches that rely on market
predictions. You must have your opinions about where the market is heading, but always invest
as though the market is going higher. Over the long run, you will be better off than if you had
jumped in and out of the market. The best policy is to only invest money that you can afford to
be patient with if the market stalls or backtracks.
Myth No.3: What goes up must come down
Once a good performing stock has been identified, dont wait for a pullback in price before
taking your position. In the long run, this will cost you more in profits than it saves in losses.
Myth No.4: What goes down must come back up
It is impossible to reap big profits from the stock market unless you are willing to buy and hold
onto stocks that are making new all-time highs in price. Further, stocks that are at new price
highs tend to do better than those making new price lows. If you are to succeed in the stock
market, you simply must eradicate from your mind the appeal of buying declining stocks!
It is best to avoid stocks that are declining in price, even if they have financial measures (low PE,
low PB) that appear to make them good values.

An exit plan must be identified for every investment before the investment is made. This plan
should cover all possible outcomes of the trade, both profit and loss.
Investing directly into equity without sufficient knowledge can be very risky. If you are a new
investor, ideally work with a financial advisor and in parallel build your knowledge with respect
to the selected stocks.

# 40: What is a Demat account / how does it


work?
Key Message: We will learn about what is demat account and why you really need it.
A Demat account is one that allows you to buy, sell as well as transact without the need of any
paperwork. If one has to save money or make cheque payments, then he/she needs to open a
bank account. Similarly, one needs to open a Demat account if he/she wants to buy or sell
stocks. Thus, Demat account is similar to a bank account where in the actual money is being
Fin10 from www.finqa.in | info@finqa.in |+91 96500 65242

All Rights Reserved

replaced by shares. In order to open a Demat account, one needs to approach the Depository
Participants [DPs] In India, a Demat account is a type of banking account that dematerializes
paper-based physical stock shares. The Demat account is used to avoid holding of physical
shares: the shares are bought as well as sold through a stock broker. In this case, the advantage
is that one does not need any physical evidence for possessing these shares. All the things are
taken care of by the DP.
Steps involved in Dematerializing existing physical shares

Fill the Demat request form or DRF ( this can be obtained from a depository
participant)
Deface the share certificate/s which you want to dematerialize by writing across
surrendered for dematerialization

Submit the DRF & share certificate/s to the DP


DP would then forward them to the issuer or their R&T Agent Subsequent to dematerialization,

your depository account with DP, would be credited with the dematerialized securities.
Benefits of Demat

A safe, secure and convenient way for holding securities;


Immediate transfer of securities;
No stamp duty on the transfer of securities;
Elimination of risks that are associated with physical certificates like bad delivery, fake
securities, thefts, delays, etc.;
Reduction in paperwork;
Reduction in transaction cost;
No odd lot problem (even one share can be sold);
Nomination facility;
Change in address recorded with DP gets registered with all companies in which
investor holds securities electronically;
The transfer of securities is done by DP eliminating correspondence with companies;
Automatic credit into DeMat account of shares, resulting from
bonus/split/consolidation/merger etc.
Facility of holding investments in equity as well as debt instruments in a single
account.

Required documents for a Demat Account


The documentation required to open a demat account depend upon to your relationship with
the institution. The DPs also ask for a DP-client agreement to be made on a non-judicial stamp
paper. Following is a broad list of documents required:

Application form

A cancelled check, preferably MICR Proof of Identification


Fin10 from www.finqa.in | info@finqa.in |+91 96500 65242

All Rights Reserved

Proof of Address

PAN card (mandatory)

Recent photographs (one or more)


In the application form there is a sheet in which your broker or sub-broker will mention the
brokerage percentage. The brokerage varies from 0.10% to 0.75%. If you plan to trade

frequently you may negotiate with your broker/sub-broker on the brokerage percentage.

# 41: Income and tax slab


The Gross Total Income of an individual comprises of the income under all the various heads.
After arriving at the gross total income, adjustments are made for set-off and carry forward
provisions as applicable. Section VI-A, of the Income Tax Act, specifies the deductions allowed
from the total income of an individual. These deductions are provided under sections 80C to
80U of the Income Tax Act. Such deductions are allowed only against specific expenditures and
payments made during the financial year. Deductions allowable under the Income Tax Act
cannot exceed the total income of the individual.
Rates of Taxation After arriving at the net income liable to tax, after taking into account all
deductions allowable, one needs to apply the respective tax rate to arrive at the income tax
payable. Rates of taxation are different for different types of assesses and are declared each
year in the Finance Bill which is further approved and passed as the Finance Act. Income tax
rates are structured slab wise, based on lower tax for lower income and higher rate of tax for
highest of income. Rates applicable to financial year 2014-15 are as follows:
Income Level / Slabs

Income Tax Rate

Where the total income does not exceed Rs. 2,00,000/-

Nil

Where the total income exceeds Rs. 2,50,000/- but does not

10% of amount by which the total income exceeds Rs.

exceed Rs. 5,00,000/-

2,50,000/-.

Where the total income exceeds Rs. 5,00,000/- but does not

Rs. 25,000/- + 20% of the amount by which the total income

exceed Rs. 10,00,000/-

exceeds Rs. 5,00,000/-

Where the total income exceeds Rs. 10,00,000/-

Rs. 125,000/- + 30% of the amount by which the total income


exceeds Rs. 10,00,000/-

Tax rebate of Rs 2,000 for income levels less than Rs 5,00,000. 10% surcharge for incomes
above Rs 1 crore. Education Cess: 3% of the Income-tax.
Fin10 from www.finqa.in | info@finqa.in |+91 96500 65242

All Rights Reserved

For example, if you earn Rs 10 lacs, then you would calculate your tax liability as: Rs. 25,000 +
20% X Rs. 5,00,000 = 1,25,000 + Education Cess 3% = Rs 128,750.
If you have a simple Form 16 from your employer and no other income, you can file your taxes
yourself athttps://incometaxindiaefiling.gov.in/. All the above calculation will automatically be
done when you fill the information. However, if your income sources for a particular year are
complicated, for example trading loss in shares, capital gain from the sale of your house etc.
then you can engage a tax filling professional.

# 42: Understand various income heads


An individual may have income from several categories, in a previous year. Income of an
individual that is chargeable to tax has to be classified under one of the following heads of
income:
Income from salary: Salary income refers to remuneration received from the employer for
services rendered by the employee. In order to be treated as income from salary, it is essential
for an employer-employee relationship to exist. Salary is taxed in the year it is due or received,
whichever is earlier. Components of salary income include wages, annuity, pension, gratuity,
retirement benefits, fees, commissions and perquisites, profits in lieu of salary, advances and
allowance. Elements of salary income, which are exempt from tax, but only up to a specific
limit, are taxed to the extent that they exceed the exemption available.
Income from house property: Income under this head is taxable if the assesse is the owner of a
property consisting of building or land appurtenant thereto and is not used by him for his
business or professional purpose. Income earned from a house property is chargeable to tax in
the hands of the owner/ deemed owner under the head Income from house property. It is
therefore essential that the assesse should be the owner of the property. If a property is
transferred to a spouse or minor child, without adequate compensation, it is not valid. The
transferor is the deemed owner and income from property is taxable in his hands. The income
that is taxed will be computed as:
Gross Annual Value of the property Less Municipal Taxes and Unrealized rent =Net Annual
Value Less Standard deductions and/or interest payable =Income from House Property
Profits and gains from business or profession: The income charged to tax under this head, is
not the income from business or profession, but, the profit or losses from business or a
profession, after deducting allowed expenses. Income from business or profession carried on by
an individual in a previous year, after adjusting for expenses, is chargeable to tax. Income under
this head includes interest, commission, salary, bonus received or accruing, or profits earned
Fin10 from www.finqa.in | info@finqa.in |+91 96500 65242

All Rights Reserved

from the business. The value of any benefits or perquisites arising out of income or profession
is also charged under this head. From the income thus computed, expenses are deducted. The
income tax act specifically indicates the expenses that are expressly deductable, generally
deductable and expenses that are not allowed to be deducted.
Capital gains: A capital asset may have changed in value over time, leading to a gain or loss
when it is sold. Any profit or gain from sale of a capital asset is subject to tax. The profits or
gains from the transfer of a capital asset is referred to as capital gain and is chargeable to tax.
Capital gains are charged to tax in the year of transfer. Capital asset includes movable or
immovable property but excludes personal assets except jewellery, paintings, sculptures or
works of art. Capital gains accrue when there is a valid transfer. This can be through sale,
exchange, relinquishment of right, or a compulsory acquisition under law.
The above mentioned are broad categories how income is categorized. Each income source
requires an in-depth understanding of various elements of taxation.

# 43: What is salary and what does salary include?


Salary is the remuneration received by or accruing to an individual, periodically, for service
rendered as a result of an express or implied contract. Definition of Salaries includes
(i)

Wages

(ii)

Annuity or pension

(iii)

Gratuity

(iv)

Fees, Commission, perquisites or profits in lieu of salary

(v)

Advance of Salary

(vi)

Amount transferred from unrecognized provident fund to recognized provident fund

(vii)
limit

Contribution of employer to a Recognized Provident Fund in excess of the prescribed

(viii)

Leave Encashment

(ix)

Compensation as a result of variation in Service contract etc.


Fin10 from www.finqa.in | info@finqa.in |+91 96500 65242

All Rights Reserved

There are also allowances given regularly in addition to salary for meeting specific requirements
of the employees. As a general rule, all allowances are to be included in the total income unless
specifically exempted. Exemption in respect of following allowances is allowable to the extent
mentioned against each:
House Rent Allowance: Provided that expenditure on rent is actually incurred, exemption
available shall be the least of the following:
HRA received.
Rent paid less 10% of salary.
40% of Salary (50% in case of Mumbai, Chennai, Kolkata, Delhi) Salary here means Basic +
Dearness Allowance, if dearness allowance is provided by the terms of employment.
Leave Travel Allowance: The amount actually incurred on performance of travel on leave to

any place in India by the shortest route to that place is exempt. This is subject to a maximum of
the air economy fare or AC 1st Class fare (if journey is performed by mode other than air) by
such route, provided that the exemption shall be available only in respect of two journeys
performed in a block of 4 calendar years.
Certain allowances given by the employer to the employee are exempt u/s 10(14). All these
exempt allowance are detailed in Rule 2BB of Income-tax Rules and are briefly given below:
For the purpose of Section 10(14)(i), following allowances are exempt, subject to actual
expenses incurred:

Allowance granted to meet cost of travel on tour or on transfer.


Allowance granted on tour or journey in connection with transfer to meet the daily charges
incurred by the employee.
Allowance granted to meet conveyance expenses incurred in performance of duty,
provided no free conveyance is provided.
Allowance granted to meet expenses incurred on a helper engaged for performance of
official duty.
Academic, research or training allowance granted in educational or research institutions.
Allowance granted to meet expenditure on purchase/ maintenance of uniform for

performance of official duty.


Under Section 10(14)(ii), the following allowances have been prescribed as exempt.

Type of Allowance

Amount exempt

Special Compensatory Allowance for hilly areas or high

Rs.800 common for various areas of North East, Hilly areas of

Fin10 from www.finqa.in | info@finqa.in |+91 96500 65242

All Rights Reserved

altitude allowance or climate allowance.

UP, HP. & J&K and Rs. 7000 per month for Siachen area of
J&K and Rs.300 common for all places at a height of 1000
mts or more other than the above places.

Border area allowance or remote area allowance or a


difficult area allowance or disturbed area allowance.

Various amounts ranging from Rs.200 per month to Rs.1300


per month are exempt for various areas specified in Rule
2BB.

Tribal area/Schedule area/Agency area allowance available in


MP, Assam, UP., Karnataka, West Bengal, Bihar, Orissa,

Rs.200 per month.

Tamilnadu, Tripura
Any allowance granted to an employee working in any
transport system to meet his personal expenditure during

70% of such allowance upto a maximum of Rs.6000 per

duty performed in the course of running of such transport

month.

from one place to another place.


Children education allowance
Allowance granted to meet hostel expenditure on
employees child.

Rs.100 per month per child upto a maximum 2 children

Rs.300 per month per child upto a maximum two children.

Compensatory field area allowance available in various areas


of Arunachal Pradesh, Manipur Sikkim, Nagaland, H.P., U.P.

Rs.2600 per month

& J&K.
Compensatory modified field area allowance available in
specified areas of Punjab, Rajasthan, Haryana, U.P., J&K, HP.,

Rs.1000 per month

West Bengal & North East.


Counter insurgency allowance to members of Armed Forces.

Rs.3900 Per month

Transport Allowance granted to an employee to meet his


expenditure for the purpose of commuting between the

Rs.800 per month

place of residence & duty.


Transport allowance granted to physically disabled employee
for the purpose of commuting between place of duty and

Rs.1600 per month

residence.
Underground allowance granted to an employee working in

Rs.800 per month

underground mines.
Special allowance in the nature of high altitude allowance

Rs. 1060 p.m. (for altitude of 9000-15000 ft.) Rs.1600 p.m.

Fin10 from www.finqa.in | info@finqa.in |+91 96500 65242

All Rights Reserved

granted to members of the armed forces

(for altitude above 15000 ft.)

Any special allowance granted to the members of the armed


forces in the nature of special compensatory highly active

Rs. 4,200/- p.m.

field area allowance


Special allowance granted to members of armed forces in
the nature of island duty allowance.(in Andaman & Nicobar
& Lakshadweep Group
of Islands)

Rs. 3,250/- p.m.

Source: Income Tax Department

# 44: Types of tax benefits on Investment products


Today you will get a broad understanding of Sec 10 of IT act exemptions and tax saving
investments. Make sure you refer to sec 80 CCE deduction for contributions towards New
Pension Scheme.
The form in which a tax benefit may be offered can vary. The following are the three types of
tax benefits that have been offered on investments.
Exemption
Certain types of income are exempted from tax. In other words, one need not pay any tax on
such income. Incomes described in section 10 of the Income Tax Act, are not included in the
calculation of total income of an individual, for the purpose of determining tax liability. Some
important types of income exempt under Section 10 are produced as under:

Section 10(1) exempts agricultural income from tax. If the net income from agriculture

exceeds Rs. 5000 and the individual has taxable non-agricultural income, then incremental
tax has to be paid on agricultural income.
Section 10(2A) exempts the income representing the share of a partner in a firm from tax.
Section 10(5) exempts from tax concession received by an employee for proceeding on
leave to any part of India, subject to prescribed conditions.

Concession received for travel to any place following retirement or termination from
service is also exempt
Fin10 from www.finqa.in | info@finqa.in |+91 96500 65242

All Rights Reserved

Any amount received as retirement dues by way of gratuity, pension, leave encashment,
retrenchment compensation and voluntary retirement compensation is exempt. This
exemption will be up to prescribed limits. Any amount above the limits will be taxed under
the head Income from salary.
Section 10(10D) exempts from tax any amount received from an insurance policy, including
bonus.
Section 10 (32) exempts from tax, the income of a minor child clubbed with that of the
parent, to the extent of the lower of Rs.1500 or actual income.
Dividends received from a company or mutual funds, on which dividend distribution tax has
already been paid, are exempt from tax in the hands of the investor.

Long-term capital gains realized from the sale of shares of a company or units of equityoriented mutual funds, are exempt from tax provided they are sold through a recognized
stock exchange and securities transaction tax (STT) has been paid as applicable.
The above list is not exhaustive. You may refer to an income tax guide for more details.

Deduction
Either the investment made, or the income received on the investment, or both, can be
deducted (usually up to a certain limit) from the taxable income. In other words, the income tax
one pays gets reduced, to the extent that some income or investment is deducted from the
amount that is subject to tax. Below mentioned are some of the most often used deductions.

Section 80 C allows for expense incurred under certain heads or investments made to be

deducted from the total income. 100% of the amount invested or Rs.1,50,000, whichever is
less, is available as deduction from total income.
1. Life insurance premium paid towards life of self, spouse or any child in case of an Individual
and members in case of a Hindu Undivided Family.
2. Payment towards a deferred annuity contract on life of self, spouse or any child in case of
an individual.
3. Contribution towards statutory provident fund, recognized provident fund, approved
superannuation fund.
4. Contribution towards Public Provident Fund Scheme, 1968 in the name of self, spouse or
any child in case of an individual or member in case of HUF.
5. Any sum deposited in a 10 year or 15 year account under the Post Office Savings Bank
(CTD) Rules, 1959.
6. Subscription to the NSC (VIII Issue).
7. Subscription to units of mutual fund Equity Linked Savings Scheme notified by the central
government.
Fin10 from www.finqa.in | info@finqa.in |+91 96500 65242

All Rights Reserved

8. Contribution by an individual to any pension fund set up by any Mutual Fund referred u/s
10(23D).
9. Subscription to any such deposit scheme of National Housing Bank (NHB), or as a
contribution to any such pension fund set up by NHB as notified by Central Government.
10. Subscription to notified deposit schemes of (a) Public sector company providing long-term
finance for purchase/construction of residential houses in India or (b) Any authority
constituted in India for the purposes of housing or planning, development or improvement
of cities, towns and villages.
11. Tuition fees (excluding any payment towards any development fees or donation or
payment of similar nature), to any university, college, school or other educational
institution situated within India for the purpose of full-time education of any two children
of individual.
12. Towards the cost of purchase or construction of a residential house property (including the
repayment of loans taken from Government, bank, LIC, NHB, specified assessees employer
etc., and also the stamp duty, registration fees and other expenses for transfer of such
house property to the assesse). The income from such house property should be
chargeable to Tax under the head Income from house property.
13. Subscription to equity shares or debentures forming part of any eligible issue of capital of
public company or any public financial institution approved by Board.
14. Term Deposit (Fixed Deposit) for 5 years or more with Scheduled Bank in accordance with
a scheme framed and notified by the Central Government.
15. Subscription to any notified bonds of National Bank for Agriculture and Rural Development
(NABARD) (applicable from the assessment year 2008-09).
16. Account under the Senior Citizen Savings Schemes Rules, 2004.
17. Five year term deposit in an account under the Post Office Time deposit Rules, 1981.
Section 80CCC allows deduction from total income for contributions made to specified
pension funds. Deduction under this section is included in under the Rs. 100000 limit

specified under section 80C. Deduction under this section is available within the overall
limit of Rs. 1,00,000 available for sections 80C, 80CCC, and 80CCD.
Section 80CCD In case of New Pension Scheme, the employee contribution up to 10% of
basic plus dearness allowance, or DA, is eligible for deduction under Section 80CCD within
the Rs 1.5 lakh limit, the employers contribution up to 10% of basic plus DA is eligible for
deduction under Section 80CCE over and above the Rs 1.5 lakh limit.
Section 80CCG pertains to Rajiv Gandhi Equity Savings Scheme, 2012 (RGESS). The scheme
allows retail investors who are investing for the first time to avail a tax benefit on 50% of
the investment made up to Rs.50, 000 directly into RGESS eligible securities.
Premium paid on health insurance policies is allowed as deduction from total income,
within the limits specified by section 80D.
Section 80DD allows deduction on expenses of maintenance of disabled persons up to a
limit of Rs. 50,000 or Rs. 1,00,000 depending upon the severity of disability.
Fin10 from www.finqa.in | info@finqa.in |+91 96500 65242

All Rights Reserved

Section 80DDB allows deduction of the expenditure on medical treatment for specified
diseases such as Parkinson, up to a limit of Rs.60,000 for senior citizens and Rs.40,000 for
others.
Section 80E allows the entire interest paid on education loan as deduction from the
assessment year relevant to the previous year in which the assesse begins paying interest
and seven subsequent years.
Section 80G allows all assessees to claim deduction up to specified limits for contributions
made to charitable organizations.
Section 80GG provides deduction to an assesse not receiving HRA for rent paid by him up
to specified limits.
Section 80U allows deductions for persons with disabilities of Rs. 50,000 for normal
disability and Rs. 1,00,000 for severe disability.

The above list is not exhaustive. You may refer to an income tax guide for more details.

# 45: Taxation of Capital Gains


Today you will learn how capital gains are categories and what is the tax treatment for each
category.
Capital gains are categorized as short-term and long-term gains based on the period, for which
they were held before transfer. This is because the tax treatment differs based on the type of
asset and the type of capital gains. If the asset has been held for period not exceeding 36
months, immediately before transfer, the gains, if any, is considered short-term in nature. For
units of equity mutual funds, shares of companies whether quoted or unquoted and quoted
securities holding period of not more than 12 months is considered for short-term capital gains.
Rates of Taxation Short-term capital gains (STCG) for equity shares and equity-oriented funds
are taxed at 15%. In case of all other securities and funds, STCG is taxed at the applicable rate of
tax. STCG is added to the taxable income and taxed at the marginal rate applicable to the
investors level of taxable income. Long-term capital gains (LTCG) are exempt from tax in the
case of equity shares and equity-oriented funds. In the case of all other securities and funds
they are taxed at 20% after indexation. The rates of tax have to be increased by surcharges and
cess where applicable.
Type of Mutual Fund scheme

Equity mutual fund/shares

Long Term Capital Gains

Short Term Capital Gains

Units held for more than 12

Units held for less than 12 months:

months: Nil

15%+ 10% Surcharge + 3% Cess

Fin10 from www.finqa.in | info@finqa.in |+91 96500 65242

All Rights Reserved

Non-equity schemes

Units held for more than 36

Units held for more than 12 months:

months: 20% with

Tax rate as per tax bracket + 10%

indexation*

Surcharge + 3% Cess

Exemptions in case of a long term capital gain on sale of residential house

Section 54 exempts long-term capital gains from sale of residential house from tax to the
extent that the gains have been invested in another residential property. The other
residential property should have been purchased within six months of sale of the first
residential house.
Section 54EC, exempts long-term capital gains on the transfer of any capital asset if they
are invested in bonds specified for this exemption. This exemption is available up to a limit
of Rs. 50 lakhs per financial year.

In case you have an investment horizon of more than five years, you should definitely invest
some portion of your portfolio in equity due to tax free advantage after one year. This will
provide growth to your portfolio on a tax-efficient basis.

# 46: How can I invest safely for a short span?


We may be saving for goals such as buying a car in 6 months, vacation in 1 years, childs school
admission fee in next 2 years etc. Savings for these short term goals can also be done in a better
way. Learn how in this lesson.
What do you do if your time horizon is more than 3 years?
If your time horizon is more than 3 years and you want a safe investment choice, investing your
money in a debt fund can be a tax efficient option in comparison with a Fixed Deposit due to
indexation benefit. But what is indexation?
If investors held an investment for the long term, they could claim that the increase in the price
of the investment was from the effect of inflation, and that no real gains have been made. An
investor making a long term capital gain can adjust the cost of acquisition for inflation. This
process is called indexation. Indexation plays an important role on Long Term Capital Gain Tax
when an investor redeems the debt fund or switches from one debt scheme to another.
Indexed cost of a long term capital asset = Cost of acquisition x (Index in the year of sale/Index
in the year of purchase)

Fin10 from www.finqa.in | info@finqa.in |+91 96500 65242

All Rights Reserved

Lets say you have mutual fund units at Rs 10 in the year 2010 and sold them at Rs 15 in the
year 2015. If the cost of inflation index for 2015 was 450 and for 2010 it was 375, what is the
indexed cost? Answer: The cost of acquisition adjusted for inflation is = 10 x (450 /375) = 12. His
indexed capital gain is 15 12 = Rs 3. The long capital gain has come down from Rs 5 to Rs 2
due to indexation.
What do you do if your time horizon is between 1-3 years?
Post budget 2014, indexation benefit is available to debt funds only after 36 months of
investment. Therefore, taxation on an FD or on a debt fund is now similar i.e. both are taxable
as per your tax bracket. In such a situation an arbitrage fund can be used. Arbitrage funds are
the panacea for low risk taking investors. These funds capitalize on the market inefficiencies
and generate profits for the investors. These funds tax treatment is at par with equity funds i.e.
after one year of investment all capital gains are tax-free.
What do you do if your time horizon is less than 1 years?
If your time horizon is less than 1 year and you want a safe investment choice, investing your
money in a Fixed Deposit is the best option due to flexibility and ease of investment.
Indexation benefit puts debt funds better than fixed deposits if you are investing for more than
36 months. However, a close substitute arbitrage fund may be used as safe investment if you
want to invest between 1-3 years. Fixed deposit now is a better option for investment less than
1 year because you can lock in the desired rate of return.

# 47: Set off and Carry forward of Losses


Set off refers to reducing the income of a year, with expenses or losses, as may be allowed by
the income tax act. Losses can be carried forward and set off against income from the same
head in a specified number of years after the previous year in which it was incurred. Losses
incurred in a previous year can be set off against income earned under the same head or other
heads of income, subject to rules. Losses can also be carried forward, to be set-off against
income from subsequent years.
Losses from house property can be set-off against income from other house properties or
other income for a period of 8 years.
Losses from business or profession can be set off against income from the same head or
other heads of income. Losses can be carried forward for a period of 8 years.
Losses from speculative business can be set-off against profits under the same type.
Investors, who make a capital loss from their investments, are allowed to carry forward the loss

for eight years from the date of the loss. Investors can deduct the loss that is carried forward,
from the capital gains made in the subsequent years reducing the capital gains tax. This act of
Fin10 from www.finqa.in | info@finqa.in |+91 96500 65242

All Rights Reserved

reducing the capital gains by deducting the capital loss is called set off. The rules for set off are
as under:

Loss arising from a short-term capital asset can be set off against the gains arising from the
sale of a long or short-term capital asset.

Loss arising from a long-term capital asset can be set off only against the gains from the
sale of long-term assets.
Any loss under the head capital gains, either short or long term, can be set off only against
income from the same head (capital gains).
Short or long term capital losses cannot be set-off against any other source of income.

If you do not have relevant background, it is better to work with a tax consultant in case you
have any of the above mentioned capital gains. However, this lesson explains you the basic rule
that even a tax consultant has to follow.

# 48: Should I select growth or dividend option in


case of a mutual fund?
We can have an objective view between growth and dividend option, only after considering the
time horizon. Unless you know for how long you want to invest the money, you cannot decide
which option is better.
Investment horizon more than 5 years
An equity investment would be preferred if your time horizon is more than 5 years. And you
must choose growth option for equity funds. This is because you can make compounding work
for you. If you receive regular dividends, you may end up using that money for non-priority
things.
Investment horizon between 3-5 years
A combination of debt and equity will work best. You must choose growth option in case of an
equity fund. In case of a debt fund, again you must choose growth option which will give you
indexation benefit.
Investment horizon between 1-3 years
Post budget 2014, indexation benefit is available to debt funds only after 36 months of
investment. In such a situation an arbitrage fund can be used. Arbitrage funds are the panacea
for low risk taking investors. These funds capitalize on the market inefficiencies and generate
Fin10 from www.finqa.in | info@finqa.in |+91 96500 65242

All Rights Reserved

profits for the investors. These funds tax treatment is at par with equity funds i.e. after one
year of investment all capital gains are tax-free.

Investment horizon less than 1 year


If your time horizon is less than 1 year and you want a safe investment choice, investing your
money in a Fixed Deposit is the best option due to flexibility and ease of investment. However,
if you are on highest tax bracket, you must chose ultra-short term debt fund with dividend
option. Dividends are completely exempt from tax in the hands of the investor. They are
however subject to additional tax on dividends which is paid directly by the mutual fund before
it distributes dividends to the investor. In case of dividends distributed by mutual funds, it is
called Dividend Distribution Tax (DDT). The rate of DDT is 28.325%. The tax on dividends is paid
directly by the company of the mutual fund which declares the dividend. The impact on the
investor is that the return would be lower to the extent of the tax so paid. Dividend option in
case of a Debt fund would make sense if you are on the highest tax bracket.

Tax Implication on Dividend received

Dividend Distribution Tax (Payable by

by Unit holders

the Scheme)

Equity Oriented Schemes

Unit holder not required to pay taxes

Nil

Debt Oriented Schemes

Unit holder not required to pay taxes

28.325%

Type of Mutual Fund scheme

# 49: Tax Saving Infrastructure Bonds vs.


Infrastructure Bonds
There is often confusion between tax-saving infrastructure bonds and tax-free bonds. We will explain
the differences between the two.
Tax-saving infrastructure bonds are a good option in the fixed income category. These are issued by
infrastructure companies approved by the government and they offer a decent rate of interest plus tax
benefits.
Investment up to Rs 20,000 in these bonds is eligible for income tax deduction under Section 80 CCF of
the Income-Tax Act. This is over and above the Rs 1,00,000 deduction available under Section 80C.
These are long-term secured bonds which mature in 10-15 years.
Everyone should take benefit of tax-saving infra bonds especially those who are on higher income tax
brackets.

Fin10 from www.finqa.in | info@finqa.in |+91 96500 65242

All Rights Reserved

Tax-free bonds are mostly issued by government enterprises and pay a fixed coupon rate (interest rate).
As the proceeds from the bonds are invested in infrastructure projects, they have a long-term maturity
of typically 10, 15 or 20 years.
The income by way of interest on tax-free bonds is fully exempted from income tax. The interest earned
from these bonds does not form part of your total income. There is no deduction of tax at source (TDS)
from the interest, which accrues to the bondholders. But remember that no tax deduction will be
available for the invested amount.
Tax free bonds vs. bank fixed deposits (FDs): The interest earned on bank FDs and other normal bonds
are added to the income of the investor and taxed as per the income-tax slabs. As interest earned from
tax-free bonds are not taxed, investors in higher tax brackets mostly earn a better post-tax return than
from FDs. But remember, the bank FDs score over tax-free bonds in terms of liquidity as these bonds
have a longer maturity tenure.
Tax-free bonds are suitable for investors looking for a steady source of income annually and can afford
to lock-in their capital for the long term.

# 50: How can an NRI open bank account in India?


Each year there are lot of Indians who leave India for better work opportunities. As per the income tax
law, a person residing outside India for more than one hundred and eighty-two days during the course
of the preceding financial year is classified as Non Resident Individual. The period of stay is counted in
number of days for each financial year beginning from 1st April to 31st March (known as previous year
under the Income-tax Act).
There is another condition that may change the status of an individual to NRI. If a person is not in India
for 60 days or more during the previous year and he/she is not in India for 365 days or more during the 4
years prior to the previous year is also classified as an NRI.
Most, if not all have left family behind whom they would like to take care of. Again, most if not all, may
look to retire back in India when the time comes. All this may require an NRI to look at various
investment opportunities that India offer.
All investments made by NRIs have to be in local currency, that is, the rupee. An NRI needs to first open
one of the three bank accounts-non-resident external rupee (NRE) account, non-resident ordinary rupee
(NRO) account or foreign currency non-resident account (FCNR)-with an Indian bank.
An NRE account is a rupee account from which money can be sent back to the country of your
residence. The account can be opened with money from abroad or local funds. An NRO account is a nonrepatriable rupee account. An FCNR account is similar to the NRE account, except for the fact that the
funds are held in a foreign currency.

Fin10 from www.finqa.in | info@finqa.in |+91 96500 65242

All Rights Reserved

The amount that is invested can be directly debited from an NRE/NRO account or received by inward
remittances through normal banking channels. An NRI needs to give a rupee cheque or draft from his
NRE/NRO account. He can also send a rupee cheque/draft issued by an exchange house abroad drawn
on its correspondent bank in India.
If the investment is made through cheques or drafts, the investor should attach with the application
form a foreign inward remittance certificate (FIRC) or a letter issued by the bank confirming the source
of funds. FIRC is a proof of payment received by the individual from outside the country in a foreign
currency. It is issued by the bank where you have the account to receive the funds.
Other know-your-customer documents such as Permanent Account Number and address proof are also
to be submitted, just as in case of resident investors.
Taxes on interest earned
Interest earned on a NRO account, as well as on the credit balances of this account, is taxed based on
the account holders tax bracket. However, the interest accrued on a NRE account is wholly exempted
from income tax, as well as wealth tax that would otherwise be charged on the credit balances of the
account. Also, cash gifts to this account do not attract taxes.
Transfers
Funds can be transferred from a NRE to a NRO account, but funds transfer from a NRO to a NRE is not
permissible. Once a transfer to a NRO from a NRE account has been made, the funds are considered as
non-repatriation, and as such, they cannot be transferred back.

# 51: NRI equity investments in India?


NRIs may invest in the Indian stock market freely. NRIs can invest in Indian stock markets under the
portfolio investment scheme (PIS) of the Reserve Bank of India (RBI). Under this scheme, an NRI has to
open an NRE/NRO account with an RBI-authorized Indian bank. An individual can open only one PIS
account for buying and selling stocks.
Aggregate investment by NRIs cannot exceed 10% of the paid-up capital in an Indian company and a PIS
account helps the RBI ensure that the NRI holding in an Indian company does not cross that limit. Each
transaction through a PIS account is reported to the RBI.
The next step is to open a demat account and a trading account with a Sebi-registered brokerage firm.
An NRI cannot transact in India except through a stock broker.
Regulations regarding NRI Trading/investments in shares:

Intraday trades are not allowed for NRI clients; clients can trade only on Delivery basis.

Fin10 from www.finqa.in | info@finqa.in |+91 96500 65242

All Rights Reserved

All contract notes of either Buy or Sell has to be reported to Authorized Dealer (PIS Banker) within
24 hours to transactions. This is done by the broker.
Every sale transactions will be credited to client Banks account Net of tax. As per current laws for
long term capital gains, Tax rate is nil & for short-term capital gain, tax rate is 15.45%.
Subscription to IPO
Shares issued through initial public offerings (IPOs) are not covered under the PIS. In case of IPOs, it is
the responsibility of the issuing company to inform the RBI the number of shares it is allotting to NRIs.

However, NRIs need NRE/NRO accounts to subscribe to IPOs. The shares acquired through IPOs can also
be sold without a PIS account. However, NRIs must furnish their bank details, besides the date of
allotment and cost of acquisition of the shares to calculate the tax on any gains they may have made.
If directly investing in the market is not your cup of tea, then Mutual Funds provide a very attractive
alternative. These while broadly delivering the advantages of the equity market also obviate the pitfalls
associated with it.

# 52: NRI Mutual Fund investments in India ?


All investments made by NRIs have to be in local currency, that is, the rupee. Mutual funds in India are
not allowed to accept investments in foreign currency. For investing in Indian mutual funds, therefore,
an NRI needs to open one of the three bank accounts-non-resident external rupee (NRE) account, nonresident ordinary rupee (NRO) account or foreign currency non-resident account (FCNR)-with an Indian
bank.
Mutual funds allow a power of attorney (PoA) holder to keep track of your money and react to market
movements that at times may call for additional purchases, switches or redemptions even as you are
away. All that the PoA holder needs to do is to submit the original PoA or an attested copy of it to the
fund house. The PoA should have signatures of both the NRI and the PoA holder. The PoA holders
signature will be verified for processing any transaction.
Similarly, an NRI can make a resident Indian his/her nominee in the mutual fund scheme. An NRI can
also be the nominee for investments made by a local resident. Fund houses also allow an NRI to have a
joint holding with a resident Indian or another NRI in a scheme.
How to redeem your mutual fund units?
Redemption proceeds are either paid through cheques or directly credited to the investors bank
account. All earnings will be payable in rupees.
As mentioned earlier, investments made through inward remittances or from NRE/FCNR accounts are
fully repatriable. Hence, earnings made by redeeming the units or through dividends are fully
repatriable.

Fin10 from www.finqa.in | info@finqa.in |+91 96500 65242

All Rights Reserved

However, in case of investments made through NRO accounts, only the capital appreciation is
repatriable, not the principal amount.
What about taxes?
While tax liabilities of an NRI investing in India are the same as that of a resident investor, tax is
deducted at source in case of the former. Whether an NRI is subject to double taxation-once in India and
again in the country of their residence depends on the country of residence. If the Indian government
has an avoidance of double taxation treaty (ADTT) with that country, the NRI will be spared from paying
tax twice.
NRI earnings from investments in India is taxed at the rate given below:

Equity Mutual Funds/Shares

Debt Funds

Investments held for less than 12 months-

Investments held for less than 36 months-

15%

As per tax slab

Long-term Capital Gains Tax

Nil

20% with indexation

Dividend Distribution Tax

Nil

28.33%

Short-term Capital Gains Tax

# 53: What are the Income Tax rules on sale of a


house?
Acquiring a house is relatively easier than sale. Unlike equity or debt market, real estate market may not
be always liquid and we may not have ready buyers. Therefore, we should pay particular attention when
it comes to investing in real estate.
The sale of a house could be for moving into a bigger house or sale of an investment in the form of
house. As per the current income-tax rules, long-term capital gains on sale of a property held for three
years, attracts 20 per cent tax. Long-term capital gain occurs if the house is held for atleast 36 months.
Exemptions are granted under certain conditions.
Capital gain exemption on residential house property: As per the existing tax provisions, long-term
capital gain arising from sale of a capital asset is exempt under Section 54/54F if invested in purchase or
construction of a house property, subject to certain conditions. To get the captain gains exemption, the
assesse needs to purchase the new residential house within a period of one year before or two years
after transfer of the original house. For under-construction properties, the construction needs to be
completed within three years from the date of transfer of the original house. Investment for getting
capital gains benefit should be made in one residential house property situated in India, not abroad.

Fin10 from www.finqa.in | info@finqa.in |+91 96500 65242

All Rights Reserved

Investment in bonds for availing long-term capital gains exemption: A tax payer who wishes to claim
the exemption from long-term capital gains has to invest the amount in the capital gains bonds within
six months from the date of sale or before the filing of returns, whichever is earlier. This benefit is
available under Section 54- EC of the Income Tax Act 1961 up to a limit of Rs. 50 lakh in a single financial
year. The total of exemption in respect to long-term capital gains for investment in capital gains bond
shall be restricted to Rs. 50 lakh.
In case you sell your house within 36 months of possession, short-term capital gain tax will be applied
and you pay taxes as per your marginal tax bracket.

# 54: 7 incomes you shouldnt forget to declare


Generally, many of us who are employed will get a Form 16 from our employer and we usually refer to
details in that form to file our returns. However, there are few other heads of income which we should
report (taxable or non-taxable) while filing our tax returns.
Employers would give us our Form 16 based on the declaration and proofs we submit. Most of us would
declare and submit proofs for house rent allowance (HRA), leave travel allowance (LTA) and our savings
under sections 80C (PPF, EPF, insurance, etc.)
There are a few more sources of income which one must disclose. The disclosure can be done either to
our employer (so that they are taken care of in Form 16) or while filing our returns.
Some of them are:
1. Interest earned from savings bank account: This interest is tax free up to Rs. 10,000. Any interest
earned above that is taxable and should be declared.
2. Interest earned from fixed deposits: This is taxable as per ones income tax slab. Most of the time
banks deduct 10 per cent TDS when the interest accrued is more than Rs. 10,000 (unless one submits
Form 15 G/H). However, the actual tax liability will be more or less, depending upon the tax bracket one
falls under after all incomes and deductions are claimed.
3. Interest earned from recurring deposits: This interest is taxable as per ones income tax slab. Banks
do not cut any TDS on interest earned on recurring deposits and, hence, it becomes even more
important to declare this source of income.
4. Cash gifts: Cash gifts of over Rs. 50,000 should be declared as they are taxable (unless for specific
occasions like marriage)
5. Capital gains/losses: Any capital gains/losses made from trading equities, selling mutual funds, gold,
etc. should be declared even though they may be non-taxable (e.g. for equities, long-term capital tax is
nil). Similarly, any losses should be declared as these help in offsetting gains for subsequent years.
6. Exempt income: Exempt income (e.g. interest earned on PPF/EPF accounts) should be declared for
auditing purposes only. This is a tax-free income.
7. Dividend income: Dividend income is tax free in the hands of the investor. However, this should be
declared while filing income tax returns.

Fin10 from www.finqa.in | info@finqa.in |+91 96500 65242

All Rights Reserved

# 55: Steps in insurance planning


Insurance is a basic form of risk management that removes the risks associated with loss of
income, reduction in income or an unplanned and unexpected charge on income, which will
upset the personal financial situation of an individual. Insurance planning is specific to the
individual and their situation. The steps in insurance planning include identifying the protection
needs and quantifying them, buying the type of insurance that suits the requirement and
setting in place a review of insurance needs periodically.
Identify insurance need: Insurance is primarily a tool for protection from financial loss.
Identifying insurance needs therefore requires identifying all those situations that can result in
a loss of income or an unexpected charge on income. Insurance needs can be broadly
categorized as:

Income replacement needs in the event of risk to the life or earning ability of an asset,
which includes the life of an individual as an asset generating income. Life insurance,
insurance for the maintenance and replacement of plant and machinery, annuities, are all
examples of insurance products that meet this need.
Income protection needs which protect the available income from an unexpected charge.
Health insurance and motor insurance are examples of insurance products that will take
over such expenses if they occur, and thereby protect the income from a large outflow.
Asset protection needs which include the need to protect assets created, from theft or

destruction. Household insurance is one such product.


The type of insurance required depends upon the age and stage in the life of the individual.
Insurance implies a cost and buying insurance that is not required is a wasteful use of income. A
young individual without any dependents may probably need a personal accident insurance
policy that will give him an income in the event of him being incapacitated in an accident, more
than a life insurance policy. The life insurance policy will replace his income in the event of his
death, but since he has no dependents it may not be as relevant at this stage in his life. For an
individual with dependents, the primary need is income replacement to support his family in
the event of his death and therefore a life insurance policy that does this will be more relevant.
The personal accident insurance is usually available as an addition to the life insurance policy.
Estimate the amount of insurance required: The purpose of insurance is to compensate the
financial loss suffered from a specified event. It is not to profit or gain from it. The amount of
insurance required must be calculated giving due consideration to factors such as the future
Fin10 from www.finqa.in | info@finqa.in |+91 96500 65242

All Rights Reserved

value of the costs being sought to be replaced, the period for which protection is required and,
the ability to bear the cost of insurance. Under-insurance will imply that the beneficiary who is
likely to suffer the loss is retaining a portion of the risk with them. Over-insurance would imply
that unnecessary costs are being incurred.
Evaluate the type of policies available for their costs and features: Insurance products can be
differentiated on the basis of their features such as premium payment, nature of cover
provided, structuring of benefits and the like. A product should be chosen based on the
features that are applicable to the individual, and not merely on the basis of multiplicity of
features. The cost associated with the insurance is an important parameter while evaluating
insurance products. Insurance is a long-term commitment and exiting midway is difficult and
has financial implications. It is therefore essential to consider the suitability of the product,
features and cost before signing on.
Evaluate insurance needs periodically since needs keep changing: Every change in the lifecycle
of the individual will warrant a review of the adequacy and coverage provided by insurance.
These include change in status from single to married, having children, approaching retirement.
Similarly, changes in financial situations and commitment such as income levels, purchase of
home, all trigger an insurance review.

Insurance should be bought to cover various risks and that is why you pay the premium.
Insurance should never be used as a tool to make money.

# 56: What are the different types of insurance


products?
Insurance products available to an individual to manage the risk in their personal financial
situation can be categorized as life insurance and non- life insurance.
Life insurance products provide cover for the life of the insured. Non-life insurance or general
insurance includes all the other types of insurance products such as health insurance, motor
insurance, household insurance, travel insurance and the like. Insurance Development and
Regulatory Authority (IRDA) is the regulator of the insurance business in India. All products and
their specifications have to adhere to the guidelines issued by IRDA.
Life insurance products can be defined by the benefits that they provide to the insured. The
insured would get the following benefits from life insurance products:

Fin10 from www.finqa.in | info@finqa.in |+91 96500 65242

All Rights Reserved

Death cover: Where the benefit is paid only on the death of the insured within a specified
period. If death does not occur, then no benefit will be paid.
Survival benefits: Where the benefit is paid when the insured survives a specified period.
Elements of Life Insurance Product
Insured: This refers to the person whose life is being insured and can be individuals, minors

or joint lives. If the life being insured is different from the person buying the insurance
policy, such a buyer is called the proposer or policy holder and such person should have an
insurable interest in the individual being covered.
Term of the contract: This is the period during which the insurance cover will be available
to the insured. In some cases, the insurance company may specify an upper age limit at
which the term of the policy would end.
Sum assured: This is the amount being insured. Some insurance policies may specify a
minimum or maximum sum assured. Some products feature sum assured as a multiple of
premiums paid. The insurance contract may also specify situations, if any, when the sum
assured will change. For example, if a term insurance is taken to cover outstanding
mortgage payments, the sum assured will decrease as the outstanding loan decreases.
Payment of sum assured: The payment of sum assured will be on the occurrence of a
specific event such as death or expiry of the term of the policy. The mode of payment of
the sum assured, whether in lump sum or as installments will be specified in the contract.

Premium payable: This will depend upon the sum assured and the term of the policy. The
mode of payment of premium, such as monthly, quarterly, half-yearly or annually will be
included in contract. Some policies involve the payment of a single premium at the start.
Non-payment of premium within the grace period allowed will make the policy lapse. The
policy can be revived during the reinstatement period by paying the pending premiums

and penalty.
Nomination is the right of a policy holder to identify the person(s) entitled to receive the
policy money in the event of the policy becoming a claim by death. The nomination can be

made at the time of taking the policy or subsequently at any time and can also be changed
any number of times.
Life insurance should ideally be bought to provide cover for the life. However, life insurance
investment policies (ULIPs) in India are positioned as safe investment options. This is absolutely
untrue. An insurance company also invests in stock market, make more money and share some
of the profits with you. Just imagine stock market has delivered 17% p.a. for the last 10 years,
how much return have you made on your ULIP policy? Think again before you invest in a Life
insurance investment linked product. Buy a term plan and save rest of the money in an equity
linked mutual funds, you will definitely make more return and yet be sufficiently covered for
your life.

Fin10 from www.finqa.in | info@finqa.in |+91 96500 65242

All Rights Reserved

Non-Life insurance provides risk cover against loss or destruction of assets created and to
provide for unexpected, large expenses that can affect the financial situation of the individual.
Elements of Non-Life Insurance Product
Sum insured is the amount specified in the policy which represents the insurers maximum

liability for claims made during the policy period. The minimum and maximum sum may be
specified by the insurer.
Term of the insurance is typically 1 year. In some cases, such as health policies, the term
may be two years.
Premium payable is a function of the sum insured and the assessed risk. The risk will be

determined based on the cover being sought such as age, gender and health history for
medical cover, cubic capacity of the vehicle, place of registration and age of the vehicle,
among other factors. Premium is typically paid at the inception of the policy.
Deductible is the portion of the claim that is met by the insured.

No claim bonus is the benefit of lower premiums enjoyed in subsequent years for each
year of no claims being made.

Lot of young people think that nothing bad can happen to them. However, accidents do happen
and they can happen to you as well. Why not prepare yourself well in advance and be
sufficiently covered with respect to medical expenses, temporary or permanent disablement
and household insurance.

# 57: What is bonus in case of a Life-insurance?


Understanding of bonus is directly linked with understanding how Life insurance products
operate. Bonus is an amount that is added to the sum assured, announced periodically as a
percentage of the sum assured. It is paid out along with the maturity value or on the
occurrence of the insured event.
Guaranteed bonus: Guaranteed bonus is paid for the first few years of the policy period, say,
five years and is paid as a percentage of the sum assured. It forms part of the benefits of the
policy and is received at the end of the term.
Revisionary bonus: This is based on the performance of the insurance company and is declared
for policy holders at the discretion of the insurer. Revisionary bonuses are declared after the
completion of the guaranteed bonus period.
If the premium due on a policy is not paid, then the policy lapses and no claim is payable on a
lapsed policy. However a policy that has been in force for at least three years for traditional
Fin10 from www.finqa.in | info@finqa.in |+91 96500 65242

All Rights Reserved

policies, on which full premiums have been paid, may acquire a cash value or surrender value.
This value is returned to the policy holder. Surrender value is usually paid out as a percentage
(30% to 35%) of the premium paid minus the first years premium. Along with this the insurers
may also pay an amount based on the current value of the assets held against the policy. In
case of Unit linked plans, the policy as a lock-in of five years and the surrender value is paid only
at the end of the term. The insurance companies offer the service of letting customers know
the surrender value of a given insurance policy.
The other option when there is a lapse in premium is to make the policy paid-up. This means
the sum assured is proportionately reduced to the same proportion that the number of
premiums paid bears to total premiums due. For example, Mr. Perfect had taken a 25-year life
insurance policy with a sum assured of Rs. 1,000,000 with the premium being paid in a halfyearly mode. After paying the premium for five years, Mr. Perfect is unable to continue paying
the premium payment. What happens to the policy? The sum assured would be Rs.1,000,000 if
premium is paid for 25 years. On half yearly basis, that would be 50 premium payments. Since
10 premiums have been paid over a period of 5 years, the sum assured will be readjusted to,
1,000,000 x 10/50 = 200,000.

So now you know, how much money you make on your insurance policy is at the discretion of
an insurance company. Its not that insurance company does not earn, but they can very well
decide that your returns may not be great.

# 58: Types of Life Insurance Policies


Traditional life insurance policies can be categorized based on the benefit patterns. The
payment of benefits from the policy at death or maturity is used to differentiate the policies.
Term Insurance
Term insurance is a pure risk cover product. It pays a death benefit only if the policy holder dies
during the period for which one is insured. Term insurance generally offers the cheapest form
of life insurance. Term life insurance provides for life insurance coverage for a specified term of
years for a specified premium. The policy does not accumulate cash value. It is generally
considered pure insurance, where the premium buys protection in the event of death and
nothing else. Term insurance premiums are typically low because it only covers the risk of death
Fin10 from www.finqa.in | info@finqa.in |+91 96500 65242

All Rights Reserved

and there is no investment component in it. The three key factors to be considered in term
insurance are:

Sum assured (protection or death benefit),


Premium to be paid (cost to the insured), and

Length of coverage (term).


Various insurance companies sell term insurance with many different combinations of these
three parameters. The term can be for one or more years. The premium can remain level or
increase. A policy holder insures his life for a specified term. If he dies before that specified
term is over, his estate or named beneficiaries receive a pay-out. If he survives the term, there

is no maturity benefit.
Endowment
Endowment is a level premium plan with a savings feature. At maturity, a lump sum is paid out
equal to the sum assured plus any accrued bonus. If death occurs during the term of the policy
then the total amount of insurance and any bonus accrued are paid out. There are a number of
products in the market that offer flexibility in choosing the term of the policy; you can choose
the term from 5 to 30 years. Endowment policies are quite popular for their survival benefits.
The benefits are enhanced by guaranteed and reversionary bonus that is declared on policies.
Some policies pay compounded revisionary bonuses, where the bonus amount is added to the
sum assured every time it is declared, and subsequent bonus is computed on the enhanced
sum assured.
Money Back Insurance policies are a type of endowment policies that covers life and also
assures the return of a certain per cent of the sum assured as cash payment at regular intervals.
It is a savings plan with the added advantage of life cover and regular cash inflow. Since this is
generally a participating plan the sum assured is paid along with the accrued bonuses. The rate
of return on the policies is quite low.
Whole Life insurance
Whole Life insurance provides life insurance cover for the entire life of the insured person or up
to a specified age. Premium paid is fixed through the entire period. There are variations to the
whole life policy provided in the market such as shorter premium payment periods and return
of premium option. The primary advantages of whole life are guaranteed death benefits;
guaranteed cash values, fixed and known annual premiums. The primary disadvantages of
whole life are premium inflexibility, and the internal rate of return in the policy may not be
competitive with other savings alternatives. Whole Life insurance is mainly devised to create an
estate for the heirs of the policy holders.
Unit Linked Insurance Plans (ULIP) ULIP is an insurance product that combines protection and
investment by allowing the policy holder to earn market-linked returns by investing a portion of
Fin10 from www.finqa.in | info@finqa.in |+91 96500 65242

All Rights Reserved

the premium money in various proportions in the equity and debt markets. The returns on
ULIPs are linked to the performances of the markets. The premium is bifurcated into the
premium used for providing the life cover and the rest is invested in the fund or mixture of
funds chosen by the policy holder. Since the fund chosen has an underlying investment either
in equity or debt or a combination of the two the fund value will reflect the performance of
the underlying asset classes. Each fund has its own risk and return profile based on the asset
class that the fund has invested in. The policyholder is also offered the option of choosing the
fund mix based on his desired asset allocation. Different insurers have different names for
these funds to reflect their return and risk characteristics. Investors are also given the option to
switch between funds.
ULIPs may offer a single premium option where a lump sum premium is paid once. They may
also feature limited premium payment period where the premium is paid only for a portion of
the term of the policy. The sum assured will be a multiple of the annual premium. Depending
upon the sum assured selected, a portion of the premium will be apportioned towards
providing the risk cover and the remaining is invested in the fund of choice. The allocation rate
refers to the portion of the premium that is invested. This rate tends to be low in the initial
couple of years when the charges are high and subsequently rises. In a unit-linked plan
investors also have the option to make additional premium payments in the form of top-ups
which again gets invested in the funds. The ULIP provides both death and maturity benefits to
the holder. At the time of maturity of the plan, the policy holder will receive the value of the
fund as on that date. The value of the fund will be the number of units standing to the credit of
the policy holder multiplied by the net asset value of the fund as on the day. In the event the
policy holder dies during the term of the policy, the beneficiary will receive either the sum
assured, the higher of the fund value and the sum assured or the sum assured and the value of
the fund, depending on the terms of the policy. The policy may have guaranteed bonus
especially in the initial years. The additions to the benefits may also be in the form of loyalty
bonus at the end of the term.

When an insurance agent positions an insurance based investment product as risk free, he/she
is just trying to close a sale by creating false impression.

# 59: Riders on a Life Insurance Policy


Riders are add-ons to the basic insurance policy to supplement the insurance cover provided. One can
also combine a set of riders and append it to the main policy. Some of the popular riders that are added
to a basic insurance policy are:

Fin10 from www.finqa.in | info@finqa.in |+91 96500 65242

All Rights Reserved

Double sum assured rider, which provides twice the amount insured in case the death
happens due to the specific reason such as accidental death while the policy is in force.
Critical illness rider, which provides a sum that could be double the sum assured on
diagnosis of a life-threatening illness.
Accident or disability rider, which enables the insured to receive a periodic payout if
temporarily disabled, for a limited period of time.
Waiver of premium rider, which is triggered if there is a disability or loss of income that
makes it difficult to pay the premium.
Guaranteed insurability option rider, which enables enhancing the insurance cover without
further medical examination.

The premiums will obviously be higher, depending on the rider or a combination of them
chosen. When combined, these riders provide the flexibility to customize a policy to ones
needs. IRDA regulations lay down limits on the benefit of riders that can be availed as follows:

The premium paid for all health and critical illness riders in case of a term or group policy
should not exceed 100% of the premium paid on the base policy.
In case of all other riders it should not exceed 30% of the premium paid on the base policy.

The benefit from each rider cannot exceed the basic sum assured.
You must weigh whether having a rider on the insurance policy is better or buying a separate

policy for the rider makes more sense. Rider has its own cost and must be evaluated in the
context of the purpose of buying life insurance.

# 60: Types of Non-Life Insurance Products


Property insurance provides protection against most risks to property such as fire,
theft etc. Property insurance generally means insuring the structure and the contents
of the building against natural and man-made disasters. Willful destruction of
property, Loss/ damage due to wear and tear and Art and antiques are typically not
covered by property insurance.
Health Insurance policy reimburse the medical expenses incurred for the policy holder and
identified family members who are covered under the policy. This policy provides for
reimbursement of hospitalization or domiciliary treatment expenses for illness or accidental
injury up to the sum insured under the policy. The expenses that can be claimed, such as
consultation fees, medicine and treatment costs, room costs, are specified in the policy and
sub-limits may be fixed for each head. Claim is typically allowed only for In-patient (patients
who are admitted in a hospital for treatment that requires at least overnight or 24 hours of stay
in hospital) treatments and domiciliary treatments (patients can be treated at home when they
are not in a condition to be moved to the hospital), according to the terms of the policy. Preexisting illnesses may be excluded from cover for a fixed period when insurance is being taken
for the first time or if it is being renewed after a lapse. Health policies provide cashless facility
Fin10 from www.finqa.in | info@finqa.in |+91 96500 65242

All Rights Reserved

too where the bills are directly settled with the hospital and the insured is not required to pay
upfront up to the sum approved for this facility. There is also the option to take a family floater
policy that will cover multiple family members under the same policy up to the sum insured.
The premium payable on the policy is a function of the sum insured, age and medical history of
the insured, among others. Premiums may be adjusted for continued health cover and record
of no-claim. Portability of health policies has been introduced under which the benefits of noclaim, bonus and time-bound exclusions for existing conditions can be transferred, if the
insured chooses to switch the insurance company. To benefit from portability, the previous
policy should have been maintained without a break. It is a good practice to maintain one
health policy even if your employer offers a health plan.
Motor Insurance indemnifies the insured in the event of accident caused by or arising out of
the use of the motor vehicle anywhere in India against all sums including claimants cost and
expenses which the insured shall become legally liable to pay in respect of (i) death or bodily
injury to any person, (ii) damage to the property other than property belonging to the insured
or held in trust or custody or control of the insured. The insurance of motor vehicles against
damage is not made compulsory but the insurance of third party liability arising out of the use
of motor vehicles in public places is made compulsory. No motor vehicle can be used in a public
place without such insurance.
Personal Accident Insurance provides that if the insured shall sustain any bodily injury resulting
solely and directly from accident caused by external violent and visible means, then the
company shall pay to the insured or his legal personal representative, as the case may be, the
sum or sums set forth, in the policy. Following types of disablement are covered under this
policy: (i) Permanent total disablement (ii) Permanent Partial disablement (iii) Temporary total
disablement
Critical illness insurance provides for a lump sum benefit to be paid if the named insured
contracts certain specified diseases such as cancer, heart attack, stroke, kidney failure or
multiple sclerosis. It differs from life insurance in that there is no payment on death.
Reimbursement is usually subject to a minimum survival period of 30 days. The lump sum
payment under the critical illness policy can be used in whatever way the claimant chooses. It
could be used for example, for income, or for repaying a mortgage. Today, these are available
either with life insurance policies or as standalone policies and as many as 30 illnesses can be
covered.
Travel Insurance provides medical, financial and other assistance in case of an emergency
during international travel. The cover will typically be provided for instances such as medical
help required, delay in baggage clearance, accident and any additional cover required. The
cover will be in the form of reimbursement up to the maximum mentioned in the policy. Travel
insurance may be mandatory for travel to some European countries.
Liability Insurance provides indemnity in respect of damages payable under law for personal
injury to third parties or damage to their property. This legal liability may arise under common
law on the basis of negligence or under statutory law on no fault basis i.e. when there is no
Fin10 from www.finqa.in | info@finqa.in |+91 96500 65242

All Rights Reserved

negligence.

# 61: How do you calculate your life insurance


needs?
The amount of life insurance cover required depends upon the economic value that can be
attached to human life. This is called the human life value (HLV). This is the value that insurance
needs to compensate for, if there is a loss to the life or disability which results in a reduction in
the ability to generate income.
Human Life Value Approach
The human life value is the present value of the persons future earnings. This is a method of
calculating the amount of life insurance a family will need based on the financial loss that family
would incur if the insured were to pass away today. It is usually calculated by taking into
account a number of factors including but not limited to the insured individuals age, gender,
planned retirement age, occupation, annual wage, employment benefits, as well as the
personal and financial information of the spouse and/ or dependent children.
Information Required The information required to calculate the same is as follows:
The number of years the individual is likely to earn (Retirement age less present age)

Average annual earnings during the earning years


Amount of personal expenses like taxes, personal costs, insurance premium which is

deducted from annual income.


Example Sid earns Rs.8 lacs p.a. He is 28 years old and would like to retire at age 60. Calculate
the insurance he needs as per Human Life Value if inflation is seen at 6% and return on
investment at 10%.
Method of calculation The HLV is the present value of the expected income over the working
life of the individual that is available for the dependents. That is Human Life Value and would
be the amount of insurance required after considering the future value of assets already
available that will contribute to this value.
The present value function in excel can be used to calculate the HLV.
Sid needs to have an insurance cover of Rs. 1,52,75,741. If Sid were to buy this cover in the
form of a ULIP, the policy would be very expensive. Ideally a term plan that too bought online
will turn out to be a cheaper option.

Fin10 from www.finqa.in | info@finqa.in |+91 96500 65242

All Rights Reserved

# 62: What type of Insurance do you need?


You often have a family member who is an LIC agent trying to sell you an insurance policy. You
land up buying these policies irrespective whether you understand the features of the policy or
not. If you were to die today, do you think this policy will be able to fulfill the needs of your
family?

Before you decide how much insurance, you first need to understand why you need insurance.
Most people buy insurance to save on tax and to get some residual retirement income. Infact,
its not uncommon to find people with multiple LIC policies. Very few of them have the most
essential and basic form of insurance, popularly known as term insurance.
An insurance policy is not meant for you. Its meant for your loved ones and to provide them
with a lump sum of money in case something was to happen to you. Its not very popular on
account of 2 reasons:
Insurance agents dont like to sell them because the premiums on these are very low and
hence so are their commissions
People think its a waste of money because you dont get anything back at the end of the
term
Dont fall into this trap. Get a term insurance policy if you really care for your family.

Now coming to how much of term insurance you should ideally have.
Legally, you can avail 20 times your gross salary as term insurance. So if you are earning Rs 10
lakhs as gross annual salary, you can get a term insurance plan of Rs 2 crores. But before you
buy a term policy you must know how much insurance you really need. Ideally you should
perform a detailed analysis to ensure you buy adequate insurance cover.

# 63: Retirement planning is not for the old


Lot of people in young age think that retirement planning is for those people who are retired or
close to retirement. But reality is quite opposite to this notion.
Retirement Planning is about ensuring that there is adequate income to meet the expenses in
the retirement stage of an individuals lifecycle. The primary income at this stage will be a
pension drawn from the employer or the passive income drawn from the retirement corpus
Fin10 from www.finqa.in | info@finqa.in |+91 96500 65242

All Rights Reserved

that has been created during the employed period of the individuals life or a combination of
the two. To be able to be adequately provided with income in retirement, a portion of the
current income has to be saved and invested over the working life of the individual to create
the corpus required. Retirement planning involves making an estimate of the expenses in
retirement and the income required to meet it, calculating the corpus required to generate the
income, assessing the current financial situation to determine the savings that can be made for
retirement and identifying the products in which the savings made will be invested so that
required corpus is created and in which the corpus will be invested to generate the required
income in retirement.
There are two distinct stages in retirement: the accumulation stage and the distribution stage.
The accumulation stage is the stage at which the saving and investment for the retirement
corpus is made. Ideally, the retirement savings should start as early as possible so that smaller
contribution made can grow to the corpus required with the benefit from compounding. The
other benefit of starting to save early (in age) is that the amount which needs to be contributed
periodically to reach the retirement corpus is lesser. It may be necessary to start low and
increase the savings as the income grows and the ability to save increases. Investments made in
the accumulation stage should be growth-oriented since there is a long investment horizon for
the short-term volatility in return to smoothen out. There is generally a greater ability to take
risk and the portfolio should be invested to earn higher returns. The distribution stage of
retirement is when the corpus created in the accumulation stage is employed to generate the
income required to meet expenses in retirement. Investment made at this stage is incomeoriented primarily. The ability to take risks is lower since any erosion in the capital created
cannot be replaced.
Retirement planning involves the following steps:

Compute the retirement corpus required based on the estimation of expenses in


retirement or income at retirement.
Determine the periodic savings required to accumulate the retirement corpus
Analyze the current financial situation to determine the savings possible
Set in place a long-term savings plan based on the expected income
Identify the investment products in which the savings will be invested
Monitor the performance of the investment and the growth of the retirement corpus
periodically.
Review the adequacy of the retirement corpus whenever there is a change in personal
situation that has an impact on income or expenses.
Make mid-course corrections, if required.
Rebalance the portfolio to reflect the current stage in the retirement plan.
Fin10 from www.finqa.in | info@finqa.in |+91 96500 65242

All Rights Reserved

A good retirement plan provides adequate corpus for the sunset years without compromising
on the standard of living of the person. It also involves smart selection of retirement products
to not only save for retirement but also help save on taxes.

# 64: How to determine your retirement expenses


? Part I
Typically, the expenses required by the person on retirement is taken as a percentage of the
expenses of that person just before retirement the assumption being that ones post
retirement expenses will not include certain expenses such as transport, home loan EMI. On the
other hand the expenses incurred on other heads such as health, is likely to be higher.
The process involves preparing a list of pre and post-retirement expenses, and arriving at the
total expense list. The expense so calculated has to be adjusted for inflation over the period of
time left to retirement to arrive at the expense in retirement. For example, Ankur has a
monthly expense of Rs. 1,00,000 of which 60% is for household expenses. He is 30 years old and
expects to retire at the age of 60. He expects to incur additional expenses of Rs. 10,000 pm at
current prices for discretionary expenses in retirement.
Impact of Inflation Inflation is a general rise in prices of goods and services over a period of
time. Over time, as the cost of goods and services increase, the value of one unit of money will
go down and the same amount of money will not be able to purchase as much with that unit of
money as he could have purchased earlier i.e. last month or last year. Inflation eats away the
purchasing power of money over time. If inflation is seen at 8%, what is the expense that has to
be met by retirement income?
Monthly expense

Rs 1,00,000

Proportion of Household Expenses

60%

Current Household Expenses

Rs 60,000

Additional Discretionary Expenses

Rs 10,000

Total Expenses in Retirement at


Current Prices

Rs 70,000

Age

30 years

Retirement age

60 years

Fin10 from www.finqa.in | info@finqa.in |+91 96500 65242

All Rights Reserved

Years of retirement

30

Rate of Inflation

8%

Expenses at the Time of Retirement


(per month)

Rs 7,04,386

60-30

70,000 X (1+8%)^30

Now you may want to look at your current expenses, apply the above calculation to figure out
what would be your expense in the first month of your retirement.

# 65: How to determine your retirement


investments ? Part II
The monthly expense in the first month of retirement can be estimated using your current expense and
future expectations. Once this has been done, the next step in retirement planning is to calculate the
investment that will generate the income required in retirement. This investment is also known as
retirement corpus.
We calculated yesterday, first month of retirement expenses as Rs 7,04,386. When Ankur turns 60 years
this will be the monthly expense. However in order to estimate overall retirement investments, we need
to have three more variables; Inflation, expected rate of return and life expectancy.

Inflation
Inflation impacts retirement planning in two ways:
1. At the time of estimating first month retirement expense, the value of the current expenses has to
be adjusted for inflation to arrive at the cost of the expense at the time of retirement. For instance,
if consumer goods prices rise 8% a year over the next 30 years, items that cost Rs 100 today would
cost Rs 216 in 10 years, Rs 466 in 20 years and Rs 1,006 in 30 years.
2. Over the retirement years, the income required would not be constant but would go up due to
inflation. The corpus created to fund income during retirement will have to consider the escalation
in cost of living during the period in which pension is drawn. For example, in the previous example
Rs. 7,04,386 is the monthly expense at the time of retirement. However, this will not remain the
same throughout the retirement period but will increase over time, depending upon inflation. The
increase in expenses has to be considered while calculating the retirement corpus else there is a
risk of the retirement being under-funded.

Fin10 from www.finqa.in | info@finqa.in |+91 96500 65242

All Rights Reserved

Expected Rate of Return


The investment created to generate the retirement income has to consider that this investment will be
invested to earn a return which will contribute towards the retirement income that the investment has
to provide. The investment has to be in place at the start of retirement can be lower to the extent that
these funds will generate a return through the retirement period. Higher the rate of return that the
funds are expected to earn, lower will be the required investment. However, a higher return will come
with a higher risk and the ability to take risk with the savings in the retirement stage will be low. The
rate of inflation and the expected rate of return on investments act in opposite ways on the amount of
retirement corpus required. While the rate of inflation pushes up the expenses and therefore the
amount of retirement savings required funding the income in retirement, the return that the investment
will generate will reduce the savings required. The real or effective rate of return that the investment
will generate will then be the expected return adjusted for inflation.

Inflation Adjusted Rate or real rate of return is the periodic rate of return on an investment after
adjustment for inflation.
Inflation Adjusted Rate = (1 + Expected Rate of Return)/(1 + Inflation) -1
Lets assume expected rate of return during retirement period as 10% and Inflation as 8%. Inflation
adjusted rate would be: (1+10%)/(1+8%)-1 = 1.85%
Meaning, the portfolio during the retirement period will grow @1.85% per annum.

Life expectancy
Life expectancy is the expected number of years of life remaining at a given age. Lets say Ankur is
expected to live till 85 years. Ankurs retirement age is 60 years and life expectancy is 85 years,
therefore he is expected to fund his retirement for 25 years.
Now we use MS Excel to estimate what is the amount of retirement assets Ankur would need when he
turns 60 years of age.

Ankur would need an investment worth Rs 16.8 crores when he retires at an age of 60 years.
In case you are not familiar with the calculation, would highly recommend sharing your details so that
we can calculate the amount for you.

Fin10 from www.finqa.in | info@finqa.in |+91 96500 65242

All Rights Reserved

# 66: What is the amount you should be saving


now for your retirement?
Once you are aware what is the retirement corpus/investment you need when you retire, the
retirement saving required is estimated. The savings for your retirement have to be created over the
working years of the individual. The amount of saving required will depend upon the investment
required, the time available to accumulate the savings and the return that will be generated by the
investment avenue into which the savings being made will be channelized.
We calculated yesterday, when Ankur turns 60 he would need investments worth Rs 16.8 crores. We are
also aware that he is currently 30 years and assume the expected rate of return to be 10% p.a.
Therefore, we have 30 years to fund the requirements for his retirement investments. So how much
Ankur must be saving on a monthly basis to meet his goal.

Ankur needs to invest Rs 74,000 per month for next 30 years to maintain his current standard of living.
In case Ankur already has savings worth Rs 10 lacs, how will that impact his monthly savings amount:

In that case Ankur will be expected to save 65,530 per month for next 30 years.
In case you are not familiar with the calculation, would highly recommend sharing your details so that
we can calculate the amount for you.
The monthly saving required is therefore a function of the investment required, the period available for
saving and the return that the investments will earn. Each individual makes a trade-off between these
three factors depending upon their situation. The higher return will come at a higher risk and Ankur
should be able to find investments that will generate the yield he requires and with a risk that he is
comfortable with. Making unrealistic estimations of the return that can be earned will result in the
corpus growing at a lower rate, and Ankur finding that his retirement savings is not adequate when he is
ready to retire. Ankur can also choose to reduce his income requirement in retirement so that the
retirement corpus required will come down and therefore the savings that he has to make too will
reduce. But while he may be able to cut down on his discretionary expenses in retirement, reducing his
expected expenses unrealistically will mean that he may not be able to live comfortably and will

Fin10 from www.finqa.in | info@finqa.in |+91 96500 65242

All Rights Reserved

probably find his retirement corpus inadequate. This is too big a risk to take in retirement and a better
option for Ankur would be to find additional ways to save during his earning years or to postpone his
retirement.

# 67: What are the kinds of retirement products?


Retirement products are used to invest the savings to create the retirement corpus in the accumulation
stage and to generate regular income in the distribution stage. The retirement stage will define the type
of retirement product suitable to the investor. In the accumulation stage, the focus of the product
should be on growth while it will be income in the distribution stage.
Retirement products may be classified as Defined Benefit (DB) plans or Defined Contribution (DC) plans.
In a defined benefit plan, the pension amount or value of retirement benefit is known beforehand. Or,
the formula based on which the pension amount would be calculated, is known. A contribution is made
by the employee, employer or both towards the benefits, which is invested to generate returns. The
investor is assured of the pension amount as defined, irrespective of the returns that is generated by the
pension fund. In a defined contribution plan, the pension amount is not known beforehand. Also, there
is no fixed formula based on which it would be determined. In a DC, a defined amount is contributed by
employee, employer or both, towards a pension fund over a period, and is invested. The final corpus of
the pension fund determines the retirement benefit, which can be a lump sum or an annuity paid by
investing the fund, or both.
Conventional retirement products have been DB plans. The biggest risk a DB plan runs is that it could
become under-funded or unable to make the defined payments, if the portfolio in which the
contributions are invested does not generate the returns required to make the payments over the
retirement period. New pension products are DC plans, which are more flexible. Since the pension
depends upon the returns made on the investment, the individual usually gets some say in the way the
amount is invested. There can be multiple options available like equity focused (high risk, high
returns), debt focused (low risk, low returns) or anything in-between (medium risk, medium returns).
The allocation of funds to various options and periodic re-allocation is usually done by the individual.
The retirement corpus of the individual depends on both the value of the contributions made and the
returns generated through their investment. A portion of this (usually around one-third of the fund
value) can be withdrawn as a lump sum. The remaining portion is used to buy an annuity that provides a
regular income through the retirement years.
In next few lessons, we will discuss retirement products in details.

Fin10 from www.finqa.in | info@finqa.in |+91 96500 65242

All Rights Reserved

# 68: What are the various mandatory retirement


saving products?
Saving for the retirement can be done through mandatory saving avenues such as the employee
provident fund or through voluntary savings schemes such as the PPF, Mutual fund schemes, and
retirement products from insurance companies or a combination of the two.
Mandatory Saving Avenues
The Employees Provident Fund and Miscellaneous Provisions require employers covered under the act
to offer the following schemes to the workers.
Employee Provident Fund Scheme
Under this scheme, the employee and the employer each contribute 12% (10% in some special cases) of
the basic emoluments to the employees EPF account. Employees can contribute over and above the
statutory rate of 12%. However, employers contribution is fixed at 12%. Contributions are credited to
individual employee accounts. Each year the government declares the rate of interest applicable to the
scheme. Annual statements which give information on the opening balance, contributions during the
year, interest credited at the end of the period, withdrawals if any and closing balance is sent to each
account holder through his/her employer. Partial withdrawals are allowed to members for specified
purposes, including housing and marriage. The balance in the employees account is paid out as a lump
sum on retirement, or total and permanent disablement or death in service or early retirement.
Employee Pension Scheme
This scheme is run by the Employees Provident Fund Organization (EPFO) and is guaranteed by the
government. 8.33% from the employers contribution of 12% to the Employee Provident Fund is
diverted to the EPS. The scheme provides pensions for life to members or their family upon retirement
at the age of 58. The central government also contributes one and one-sixth percent of the wages.
Gratuity
The Payment of Gratuity Act, 1972 requires gratuity to be paid to an employee at the time of
termination of employment where the employee has been in continuous service for not less than five
years. The condition of completion of five years continuous service is not applicable where termination
is due to death or disablement. The benefit is paid in lump-sum. The minimum benefit is approximately
two weeks last drawn salary for each year of service with an upper limit of Rs. 350,000. Gratuity
received up to the limit of Rs. 350,000 is exempted from tax under the Income Tax Act. The exemption
is, however, not available for payment of gratuity when the employee is still in service.
Superannuation Benefit
Employers provide superannuation plans to augment the benefits available by contributing to a
superannuation fund. The company has to appoint trustees to administer the scheme and get the

Fin10 from www.finqa.in | info@finqa.in |+91 96500 65242

All Rights Reserved

scheme approved by the Commissioner of Income Tax. A company can offer a group superannuation
scheme in two ways:

Through the constitution of a trust fund where fund managers are appointed by the trustees to
manage the fund.
Through investment in a superannuation scheme from a life insurance company.

On retirement the employee is allowed to take one third of the accumulation in his account as
commutation. Commutation refers to the exercise of the facility of taking a portion of the annuity
corpus in a lump sum. The balance in the corpus is used to purchase an annuity. Apart from LIC, all other
life insurance companies allow its customers to purchase annuity from any annuity provider. Income Tax
rules restrict the employers contribution, whether to the PF or superannuation fund or a combination
of both, to 27% of the employees earnings.

# 69: What is a NPS under voluntary retirement


saving option?
The mandatory retirement benefits are available only to employees of covered establishments. This
leaves people in other establishments, self-employed persons and others out of the coverage of the
benefits. Retirement plans are available that an individual can subscribe to independently. Such
schemes can be used to save for retirement by people who do not have mandatory cover and also by
persons who are covered to augment their retirement benefits. Voluntary retirement schemes include
the New Pension Scheme, Public Provident fund, Schemes of mutual funds and insurance companies.
New Pension Scheme (NPS)
The NPS is a defined contribution scheme launched in May 2009 by the government of India for all
citizens on a voluntary basis. The contributions made by an individual to the fund are managed to create
a retirement corpus. Any Indian citizen not less than 18 years of age and not more than 55 years of age
can join the NPS.
There are two types of accounts that the NPS offers Tier I account where the contribution cannot be
withdrawn and a Tier II account where the contribution can be withdrawn at any time. To have a Tier II
account the individual must have a Tier I account. The NPS offered only the Tier I accounts to begin with.
Investment Fund Options
The funds contributed will be managed according to the investment mix selected by the contributor. The
portfolio thus selected will be created and managed by the fund manager selected at the time of
registering from among the approved fund managers. The fund options available are equity or E, credit
risk-bearing fixed income instruments or C and pure fixed investment products or G. A subscriber can
choose to invest the entire corpus in C or G. However investment in E is capped at 50 per cent. Any
combination of the funds can be chosen to apportion the corpus within the limits specified. Investors

Fin10 from www.finqa.in | info@finqa.in |+91 96500 65242

All Rights Reserved

who cannot make the choice between options can opt for the default option. The default option, called
auto choice lifecycle fund, will see the investment mix change according to the age of the subscriber.
At present, the equity investment will include investment in index funds that track the BSE Sensitive
Index or the S&P CNX Nifty index. The C segment includes liquid funds, corporate debt instruments,
fixed deposits, public sector bonds, municipal bonds and infrastructure bonds. The pure fixed
investment instruments include state and central government securities. Asset
Allocation in the Life Cycle Choice
Between the age group of 18 years and 36 years the funds will be allocated as 50% in E, 30% in C and
20% in G. After 36 years, the ratio of investment in E and C will decrease annually, while the proportion
of G rises till it reaches 10% in E and C and 80% in G at the age of 55.
On reaching the fixed retirement age of 60 years, the contributor has to use at least 40% of the
accumulated corpus to buy an annuity. The remaining funds can be withdrawn as a lump sum either at
once or in a phased manner before 70 years. Any balance in the account on reaching 70 years of age will
be paid-out as a lump sum. The annuity selected can be one that pays survivor pension to the spouse. A
subscriber can also choose to invest more than 40% in an annuity. Exit before turning 60 years is
possible through an option to withdraw 20 per cent of the accumulated savings and compulsorily buy an
annuity with the remaining 80 per cent. In the event of the death of the subscriber, the entire corpus
standing in the account will be paid-out to the nominee(s) as a lump sum. The NPS follows the EET
regime for taxation. This means that the contribution made to the NPS will receive tax benefits in the
form of deduction from taxable income, the gains made in a year will not be taxed in the year in which it
is earned but the amount will be taxed at the time of withdrawal.
Employees opting for NPS through corporate scheme can help lessen tax burden by saving in NPS upto
10% of their basic salary. This investment is another avenue over & above those of Sec 80C investments
to secure retirement well in advance. Some portion of your retirement investment should definitely be
invested in NPS. If you are young apart from investing into NPS, you should also make investments into
equity since equity portion of NPS is restricted to maximum 50%.

# 70: What are pension plans offered by insurance


companies?
Insurance companies offer individual pension plans both as traditional plans and as unit-linked plans. In
traditional plans, the sum assured is the corpus (along with additions and bonus, whether guaranteed or
otherwise) and is fixed at the time of taking the policy. In a unit-linked pension plan the corpus would be
the value of the fund. The value of the fund in turn, would depend upon the type of fund (combination
of equity and debt in various proportions) chosen by the policy holder. The pension plans are usually
structured as deferred annuity plans where the policy holder contributes premium over the term of the
policy and the corpus so created is used to buy an annuity on vesting. Depending upon the type of

Fin10 from www.finqa.in | info@finqa.in |+91 96500 65242

All Rights Reserved

annuity purchased, the annuitant receives a pension over their life time. Apart from the elements of an
insurance contract such as the sum assured, premium details, the minimum and maximum age at entry
into the plan, insurance companies specify the minimum and maximum age at vesting. Vesting refers to
the age at which the corpus has to be used to buy the annuity. The policy holder has to decide the sum
assured and the vesting age that is required. Based on these factors and the age of the policyholder the
applicable premium will be decided in a traditional plan. In a unit-linked plan, the premium that the
policy holder can pay is decided first and is invested in the fund of choice to create the corpus. Unlike a
traditional plan, the value of the fund on maturity is not known in a unit-linked plan.
Death and Maturity Benefits
If the policy holder dies during the term of the policy when the corpus is being accumulated, the death
benefit would depend upon the terms of the policy. If the pension plan offers life cover, the death
benefits in case of a traditional policy will be the sum assured. In case of unit-linked pension plans, the
death benefit will be the higher of the sum assured and fund value, only the fund value or fund value
and sum assured, depending upon the terms of the policy. If life cover is not offered then the insurance
company will repay the premiums paid with interest. The maturity benefit will be the sum assured along
with benefits such as bonuses in case of traditional plans and the value of the fund in case of a unitlinked plan. One-third or one-fourth of the corpus may be withdrawn as a tax free lump sum.
Commutation refers to the exercise of the facility of taking a portion of the annuity corpus in a lump
sum. The balance has to be used to buy an annuity which will provide the pension. The guidelines issued
by IRDA now require that the annuity be bought from the same insurer with whom the accumulation of
the corpus was done. If death occurs during the period when annuity is being drawn, then the benefit
will depend upon the type of annuity that has been bought. This can either be cessation of pension,
continuation of pension to spouse, repayment of annuity purchase price or a combination. If the
nominee is the spouse, then the death benefits may be taken as a lump sum or in combination with an
annuity. If the nominee is not the spouse, then the benefits are paid out as a lump sum. The premiums
paid towards pension plans are eligible for benefits of deduction under Section 80C and 80CCC.
Similarly, the amount received on commutation and other benefits received on maturity or death is also
subject to exemption under the Income Tax Act.

# 71: What are pension plans offered by mutual


funds?
Investment in designated pension plans from mutual funds allows accumulating a corpus for retirement
Mutual funds are ideal for long-term financial planning. But unfortunately only two asset management
companies, UTI and Franklin Templeton, have till date been allowed to launch funds whose purpose is to
help people plan for retirement. These invest up to 40% money in equities and the rest in debt. Plus,
investments in both are eligible for tax exemption up to Rs 1.5 lakh under Section 80C of the Income Tax
Act. Both levy an exit load to discourage people from quitting early.

Fin10 from www.finqa.in | info@finqa.in |+91 96500 65242

All Rights Reserved

Pension schemes work just like other mutual fund schemes. Funds contributed by different investors are
pooled together and invested in a portfolio of securities. The investors share of the pool is determined
by the number of units held. The fund will define the minimum investment. The investment objective of
the fund would be to build a retirement corpus for the investor over a long period of time, by using a
combination of equity and debt securities. These funds differ in the exit options available to investors.
Since the corpus is being accumulated for retirement, mutual funds penalize withdrawals from the fund
before the assumed retirement age of 58 years. After the specified age, the funds accumulated can be
freely withdrawn. Investors can choose to remain invested and receive a periodic payment in the form
of dividend or redemption while the rest of the funds continue to remain invested and grow. Or the
corpus can be withdrawn to buy an annuity. The returns from mutual fund schemes, including pension
plans, cannot be assured according to SEBIs guidelines. Investments made into these funds are eligible
for deduction from total income under section 80C of the Income Tax Act. At the time of redemption
they will be subject to capital gains tax applicable to debt-oriented funds. The products described above
are designed primarily for accumulation of the retirement corpus.
You can invest in these plans, if you are keen on a significant equity component.

# 72: If you are already retired, what are your


options for creating pension during retirement?
We will cover various investment options during your retirement phase
Senior Citizens Saving Scheme (SCSS)
This scheme has been announced by the Government of India for Indian citizens who are 60 years of age
or more. It is a 5-year deposit on which interest is paid on a quarterly basis. On maturity, the tenor can
be extended by another 3 years. The maximum amount that can be invested by an individual, singly or
jointly with another holder, is Rs.15 lakh. The age restriction applies only to the first holder of the
account. The second holder has to be the spouse and can be younger than 60 years of age. Deposits
have to be in lots (multiples) of Rs.1000. SCSS is also eligible for tax deduction under Section 80C of the
Income Tax Act. SCSS deposits can be opened by post offices or with designated banks. Only resident
individuals can invest. The deposits are not transferable. Pre-mature withdrawal is possible after the
completion of one year but with penalties. Interest is paid on a quarterly basis, at 9.2%. Interest is paid
out on the last dates of the calendar quarters of March 31, June 30, September 30 and December 31,
every year. Interest is reset each year.
Interest can be paid out or accumulated to be paid at maturity. The reinvestment of interest into the
account also earns interest. Tax is deductible at source on interest paid on SCSS deposits, if the amount
of interest exceeds Rs. 5000. Declaration in Form 15H (senior citizens aged 65 or more) or Form 15G

Fin10 from www.finqa.in | info@finqa.in |+91 96500 65242

All Rights Reserved

(others) can be made to ensure payment of interest without TDS. If a person is above 55 years and has
received retirement benefits under a voluntary retirement scheme, such retirement proceeds can also
be invested in the SCSS by such persons. Retiring personnel from defense services are eligible to invest,
without any age limit, subject to other conditions. SCSS deposits cannot be pledged or offered as loan
collateral.
Post Office Monthly Income Scheme (MIS)
One of the more popular schemes of the post office the MIS offers a monthly interest at 8.4% p.a. for
deposits made with the POSB, from the second month after the deposit is made. Minimum investment
is Rs.1000 and maximum Rs.3 lakhs. The tenor of the scheme is six years. MIS accounts can be opened
jointly by a maximum of two depositors, in which case the maximum amount permitted is Rs. 6 lakhs. A
depositor can have multiple accounts but the aggregate amount across all MIS accounts with the post
office cannot exceed the maximum prescribed limits. The interest payable on the scheme is reset each
year with reference to market rates. Interest is subject to tax.
Monthly income plans (MIPs)
MIPs of mutual funds seek to provide a regular monthly income (without any guarantee to do so) by
creating a portfolio that invests pre-dominantly in debt instruments. Childrens education plans
designed for older children whose educational goals are not too far away, invest pre-dominantly in debt
markets to protect the capital, and a small proportion in equity to provide the benefit of growth. The
overall portfolio tends to feature a low level of risk. Debt-oriented hybrids are designed to be a low risk
product for an investor who likes to earn the short term debt market return enhanced by a small
equity component that does not significantly add to the risk of the portfolio. Investors can structure the
income from the mutual fund either as dividends received from the scheme or by setting in place a
systematic withdrawal of units. Dividend received is exempt from tax in the hands of the investor. In
case of withdrawal or redemption of units, the gains made will be taxed as short-term or long-term
capital gains.
Bank Deposits & Other Deposits
Bank deposits and other deposits will generate the regular income that will be required to meet
expenses. Since bank deposits are seen as risk-free institutions, there is a high degree of safety in the
investment and the returns committed. The drawback is that deposit products available have a
maximum tenor of 5 to 7 years after which the funds have to be re-invested at the rates prevailing in the
future. This may be lower or higher than current rates and this causes uncertainty in the retirement
income of the individual. Deposits, other than those with banks and financial institutions, have a high
degree of risk of default and must form a small part of the portfolio. They are also highly illiquid. Interest
received on deposits is taxed at the marginal rate of tax applicable to the individual.
Debentures and Bonds
Debentures, bonds and other debt instruments of various issuers can be used to generate a portion of
the periodic income that will form part of the retiral income. The choice of the bonds should be made
with care to reduce the risk of default by the issuer. The interest earned will depend upon the market

Fin10 from www.finqa.in | info@finqa.in |+91 96500 65242

All Rights Reserved

rates at the time of issue and the default risk associated with the borrower. The interest is typically paid
annually or semi-annually and is taxed in the hands of the investor.
A combination of monthly income plan (mutual funds), Post office scheme, Senior citizen scheme and
corporate bonds may be a suitable strategy for retirees. The proportion in each of the above categories
may vary depending upon your risk appetite and existing investments.

# 73: So many retirement products but which one


should I buy?
The physical and financial assets that are accumulated in the income earning years are used to generate
a passive income to support expenses in retirement. Planning early and executing the plan in a way that
considers changing situations ensures that the financial security required for a comfortable retirement is
achieved.

Income increases in the accumulation stage and then starts declining


In the distribution stage the individual requires the retirement corpus that has been accumulated to
generate the income required to meet expenses in retirement. The investments must be primarily
income-oriented and must feature a lower risk. This is because the individual will not be able to take the
risk of losing the corpus that has been accumulated.
The income needs, and risk and return requirements in retirement can be further divided into two
stages: the first 10 to 15 years of retirement where the retiree is expected to be active, healthy and may

Fin10 from www.finqa.in | info@finqa.in |+91 96500 65242

All Rights Reserved

have a higher lifestyle expenses on travel and such. They may also be supplementing retirement income
with income from some employment. At this stage in life, some exposure to equity can be taken
primarily to protect the investment corpus from higher than expected inflation and higher than
expected expenses. In the second stage of retirement, the level of activity and lifestyle expenses
significantly reduces. At this stage, the investor prefers safe income generating investment avenues. The
retirement corpus should be distributed among different types of income generating investments with
different features. It is good for the retiral income to be generated from multiple sources rather than
just one. The features of risk, return, liquidity, flexibility and taxability will be different in the different
investment avenues.
Summary of various retirement products:

Criteria

Minimum
Contribution

Cost

PPF

Rs 500

n/a

Unit linked pension


plans (Insurance)

NPS

Rs 18,000 Rs 24,000

Rs 6000

Depends upon the

0.00010% on net

company

assets

Pension funds (Mutual


fund)

Rs 5000 lump sum or Rs


500 through SIP

1.5-2% of investments

Stopping
contribution
without incurring

Possible

Not before specified


time

Possible

Possible

loss

Deduction u/s
80CCE i.e. 10% of
Tax benefit

Deduction u/s 80C

Deduction u/s 80C

employees basic
salary over and

Deduction u/s 80C

above deduction
u/s 80C

Upto 20% of the


corpus before 60.
Lock-in is for 15
Lock-in and

years. Partial

withdrawal

withdrawal from
7th year.

Its mandatory to buy


annuity from insurer
offering ULIP

At 60, can
withdraw 60% of
investment; the
remaining has to

Restriction on withdrawal.
Lock-in period varies across
companies.

be put into an
annuity scheme.

Return

Fixed return
(decided by

Market linked

Market linked

Fin10 from www.finqa.in | info@finqa.in |+91 96500 65242

Market linked

All Rights Reserved

government)

Tax on
withdrawal

Penalty for
discontinuation

No

On maturity 1/3rd is tax

Rs 50 while
restarting the
account

Yes

free; pension is taxable

0.5% annually

Rs 100 to restart
dormant account

Yes, taxes as debt fund

No

Two retirement products should definitely be part of your portfolio during the accumulation stage; NPS
and PPF. There are few reasons why we recommend having these products as part of your portfolio:
NPS: Under direct regulatory oversight by government agency (PFRDA) the scheme offers option of
extra saving through tax benefit over and above 80C (employer sponsored). The returns are also linked
to the market that will help to grow your savings more than return on debt.
PPF: Although PPF will always provide you return similar to a fixed deposit return but it is tax free.

Retirement planning requires periodic review to make sure that the estimates for income and expenses
in retirement are relevant or need to be changed. Typically, every time there is a significant change in
current income and lifestyle or expenses that are likely to continue into retirement, it is necessary to
make changes to the plan. The change in income or expense will imply a change in the retirement
corpus being accumulated and the periodic savings and investments that have to be made to achieve
the new target.
The performance of the investments that have been made also needs to be monitored to ensure that
the retirement savings are growing as expected. Unless a periodic check is made, under-performance
will not be detected early enough and making corrections may become difficult. The allocation of the
savings being made to various types of investment will depend upon the age and stage of the individual.
For example, in the accumulation stage, the focus will be on growth-oriented products. As the individual
comes closer to retirement the portfolio has to be periodically rebalanced to shift away to less risky
assets so that the corpus accumulated over the years is protected from a fall in value when it has to be
available for generating the retirement income.

# 74: Buying your first Home


Everyone aspires to buy a home at some stage of their life. Lot of people believe that real estate prices
will never go down. This is just a myth, real estate prices also go down but the decline is gradual.
Therefore, buying first time home should not be based purely on emotions.

Fin10 from www.finqa.in | info@finqa.in |+91 96500 65242

All Rights Reserved

A house, besides being a long-term asset, makes you eligible for significant tax breaks as well. You can
claim deduction up to Rs 1.5 lakh for repayment of home loan principal under the overall limit of Section
80C of the Income Tax Act. Moreover, you can claim an additional deduction of up to Rs 1.5 lakh under
Section 24B for interest payment once you get possession and occupy the house. Also, a joint loan
means each of you can claim both these deductions individually, thus optimizing your tax savings.
One of the key parameters defining your home purchase budget is how much do you currently have and
how much you would borrow from bank as home loan. These days most of the houses in metros would
come around a crore plus. In smaller towns this budget may place you very well in luxury segment.
Irrespective of the category of home buyer, first time home purchase is usually a stretch on personal
finance. On one side, you would exhaust all your savings and on the other side you would take a home
loan that will pump up your monthly expenses. Therefore, you should run your own numbers before
committing yourself for first time home purchase.
One of the factor most of the first-time home buyers are usually concerned with is the amount of loan.
Usually 46 times of your annual take home is considered as the eligible home loan amount with
maximum 80% of home purchase value. However, there are many other factors that can impact your
home loan eligibility. It is best to check with your home loan provider for more details.
The other important factor is the amount of EMI towards home loan. Here are three scenarios if you
plan to take a home loan of Rs 1.5 crore @ 10.5 % p.a.

Scenario 1

Scenario 2

Scenario 3

Tenure

30 years

20 years

15 years

EMI

1.37 lacs

1.50 lacs

1.66 lacs

3.44 crore

2.1 crore

1.48 crore

Interest over
lifetime of the loan

Ideally you should pay off your loan as soon as possible and should go with the least tenure. However, if
you are an experienced investor and have confidence of generating return more than the interest cost,
you may select the longest tenure and invest the differential (1.50 lacs 1.37 lacs = Rs 13,000) that will
grow your portfolio more than the extra interest expense.

There are many factors that impact decision of buying a house. Take your time and do your research
before you commit yourself to one of the dearest and long-term asset of your lifetime.

Fin10 from www.finqa.in | info@finqa.in |+91 96500 65242

All Rights Reserved

# 75: How can Real estate help you during your


retirement?
The house in which you stay should ideally be kept for next generation. However, in case you have
second home or another property, that real estate can be used to provide extra income during your
retirement days.
Rental income from real estate property held provide a source of income that is adjusted for inflation.
The income also has the advantages of being periodic and known in advance to plan and use. Rental
yield earned depends upon the price at which the property was purchased. If the property that was
purchased at a lower price earlier on is now generating good rental income, it adds to the financial
security of the individual. Real estate provides appreciation in value as well as income. However, in the
distribution stage of retirement, its ability to generate good rental yields is what is important.
The drawback of real estate comes from the low liquidity that it has. If there is an emergency and funds
are required immediately, it will be difficult to liquidate the investment fast, though it is possible to get a
loan against the property. Real estate can be one more asset class for generating income in retirement
provided the investor has adequate investments in financial assets that are liquid and provide regular
income.
Reverse Mortgage
The self-occupied home can also become a source of income in the extreme situation of the retirement
corpus being inadequate to fund a comfortable retirement. The reverse mortgage scheme is offered by
housing finance companies and banks. The important features of Reverse Mortgage are summarized
below:
1. In a typical mortgage, you borrow money in lump sum right at the beginning and then pay it back over
a period of time using Equated Monthly Installments (EMIs). In reverse mortgage, you pledge a property
you already own (with no existing loan outstanding against it). The bank, in turn, gives you a series of
cash-flows for a fixed tenure.
2. Eligibility Criteria

Indian citizen of 60 years or more,


Married couples will be eligible as joint borrowers for joint assistance. In such cases, the age criteria
for the couple would be at the discretion of the RML lender, subject to at least one of them being
above 60 years of age and the other not below 55 years of age.
Should be the owner of a residential property (house or flat) located in India, with clear title
indicating the prospective borrowers ownership of the property.

Fin10 from www.finqa.in | info@finqa.in |+91 96500 65242

All Rights Reserved

The residential property should be free from any encumbrances.


The residual life of the property should be at least 20 years. There is no minimum period of
ownership of property required.
The prospective borrower(s) should use that residential property as permanent primary residence.
3. The amount of loan available under RML depends on the age of the borrower, appraised value of the
house and the prevalent interest rates of the lending institution.

4. A reverse mortgage loan cannot be availed against commercial property.


5. The maximum monthly payments under RML have been capped at Rs.50,000. The maximum lump
sum payment shall be restricted to 50% of the total eligible amount of loan subject to a cap of Rs. 15
lakhs, to be used for medical treatment for self, spouse and dependants, if any. The balance loan
amount would be eligible for periodic payments.
6. All receipts under RML shall be exempt from income tax under Section 10(43) of the Income-tax Act,
1961.
7. The rate of interest and the nature of interest (fixed or floating) will be decided by the lender.
8. The maximum tenure of an RML will be 20 years.
9. An RML will become due and payable only when the last surviving borrower dies or permanently
moves out of the house. An RML will be settled by proceeds obtained from sale of the house property
mortgaged. After the final settlement, the remaining amount (if any) will be given to the borrower or
his/her heirs/beneficiary. However, the borrower or his/her heirs may repay the loan from other
resources without bringing the property to sale.
10. The borrower will remain the owner of the house property and need not service the loan during
his/her lifetime as long as the property is used as primary residence. Periodic payments under RML will
cease after the conclusion of the loan tenure. Interest will accrue until repayment.
11. The Reverse Mortgage loan can be prepaid at any time during the currency of the loan. On clearance
of all the dues, all the title deeds will be returned by the lender.
12. The borrower can opt for the frequency of EMI pay out (a monthly, quarterly, annual or lump sum
payments) at any point, as per his discretion.

First you create a house in which you live, atleast for couple of years. Then you may aspire to move into
a bigger house. In case your financial situation is quite comfortable, you may create an additional real
estate property, which can be used to provide rental income during your retirement days. In case you

Fin10 from www.finqa.in | info@finqa.in |+91 96500 65242

All Rights Reserved

just have one property in which you stay, the possibility of having rental income is very low. Only in
extreme financial constraint situation you should evaluate Reverse Mortgage option.

# 76: Alternative to pension plans?


A lot of investors think that retirement pension plans are the only way to go; and if they do not invest in
these products today, then they will miss out on something. Without investing into a pension plan, you
can create your own pension.
There are two types of pension plan; Deferred Annuity Plans and Immediate Annuity Plans. In Deferred
Annuity Plans you first pay premiums for X number of years. Once you retire, then you start getting
pension income. For example, LIC Jeevan Tarang, LIC Jeevan Nidhi , Bajaj Allianz Swarna Raksha ROC ,
New Pension Scheme (NPS).
Immediate Annuity Plans start paying you the annuity right from day one once you make a lump sum
payment. So if a person wants a monthly pension and has huge lump sum money, he can buy an
immediate annuity plan and start getting pension. Its a simple product which is not so much popular in
India like deferred annuity plans. For example, LIC Jeevan Akshay , ICICI Pru Immediate Annuity , HDFC
Immediate Annuity .
In case of pension plans up to one third of the maturity amount, which can be withdrawn, is treated as
tax free in the hands of the individual. The pension, from the remaining two-thirds amount, is taxed
according to the marginal rate of tax.
ULIPs and other similar products charge high costs for initial years without giving clarity to customers.
These annuity plans also have high allocation charges many times and customers do not know about it
and cant do much later when he acknowledges it! So why do you want to pay high fees for these
products?
During the accumulation phase (income earning period), you can create your own pension by combining
mutual funds, real estate, PPF and NPS. When the time comes for retirement, you can shift your mutual
fund corpus into secure investment vehicles such as Post office scheme, senior citizen savings plan, bank
deposit etc. You can create your own pension through interest income from PO schemes/bank deposits,
rental income from real estate and pension income from NPS.

Fin10 from www.finqa.in | info@finqa.in |+91 96500 65242

All Rights Reserved

# 77: 10 things to do before you Retire


Do not put off today what you cannot afford to do tomorrow. In spite of the world wide pension crisis
and a growing acceptance that we must plan and save for our retirement, the harsh reality is we are
actually not saving enough. Research reports reveal that only 15 per cent of the individuals are saving
sufficiently for their retired life. Here are a few tips on things to do before you retire so that your retired
life is more comfortable and enjoyable.
1. Get rid of all your debts
If you are taking a housing loan, personal loan, car loan or any other loan make sure that you repay
them on or before your retirement. You need to choose the term of the loan in accordance with your
retirement age. You can truly enjoy your retired life when you have 100 per cent financial freedom, not
when you have to repay your loans.
2. Protect your emergency fund
Emergency expenses can happen any time. But the possibility goes up as we grow older. So we need to
enhance the emergency reserve year on year, based on the inflation and change in your expense levels.
An Emergency fund will give you a sense of security and also you need not touch your other investments
during an emergency where you need to pay pre-closure penalty. Also dont forget to refill the
emergency fund once you met an expense out of emergency fund.
3. Establish a retirement budget
You need to visualize your retired life well in advance and need to create a budget for your retirement.
E.g. You will not be going to office so expenses on transport and clothes may come down. Also you will
have more time to spend. You may need to spend more on leisure travel and health care.
4. Examine your cash flow
Take a close look at your cash inflow as well as outflow. Is there going to be any income after
retirement? Like rent, royalty, etc. Will there be any unwanted outflow during your retired life? Like
paying life insurance, or SIP. At times during your beginning of the career, you could have taken a policy
where you need to pay premium up to the age of 60. But now you may plan to retire at 55 itself. So you
need to realign your existing policy and other investments in sync with your retirement age.
5. Grow your retirement corpus
Find out how much of a corpus you will need to have when you retire so that you have complete
financial freedom. E.g. Rs 1 lakh a month sounds enough today but will it be enough 20 years down the
line to meet your requirements ? A professional financial planner can help you determine the right
corpus amount.
6. Develop a withdrawal strategy
How are you planning to withdraw your cash outflow during retirement from the retirement corpus?
Are you going to withdraw monthly, quarterly, half yearly or annually? Through systematic withdrawal

Fin10 from www.finqa.in | info@finqa.in |+91 96500 65242

All Rights Reserved

plan in mutual funds or by way of dividend or interest. All these will have a great impact on the corpus
you need to accumulate. So you need to decide in advance.
7. Minimize taxes
Your retirement corpus and retirement income need to be tax efficient. You need to pay taxes for the
interest accrued irrespective of that you withdraw the interest or reinvest under a cumulative option.
But you need to pay income tax only when you withdraw from the mutual funds. Careful selection of
investment vehicle can reduce your tax during the retired life.
8. Get sufficient mediclaim coverage
The moment you retire, your employer will stop covering you under the group mediclaim. So you need
to plan for your individual medical cover well in advance. At old age the medical expenses are inevitable
and will only increase. If you have not planned it properly the all your retirement plans can go haywire.
9. Consider inflation adjusted annuities
The monthly income you need when you retire is not going to be the same even after 5 years of your
retirement. Inflation will increase your retirement expenses year after year. So year after year your
retirement income needs to go up.
10. Oversee estate planning
How your fixed assets and financial assets need to be distributed to your legal heirs? Create a Will. You
can avoid creating relationship problems to your next generation because of your left out wealth.
A lot of us tend to think that just because we have bought a few LIC Policies or invested in a few mutual
funds, we will be fine. However, unless you are aware of what and how much you need for your
retirement goals, your current investments will probably not be enough.

# 78: What should I do with my Bonus?


Everyone gets a bonus at year end or whenever a company closes its financial year. Depending upon
your performance throughout the year, you are paid a percentage of your salary as bonus.
Here are few suggestions what you can do with your year-end bonus:
Treat yourself. Budget a portion of your bonus and reward yourself with some short-term gratification.
Pay off credit cards. Using your bonus to pay off a credit card may be one of the most productive moves
you can make with it.
Reduce your home loan. Look at this as the best of both worlds you get to spend the money by
putting it toward your homes equity, but still get some ongoing benefit from that money in the form of
interest savings.

Fin10 from www.finqa.in | info@finqa.in |+91 96500 65242

All Rights Reserved

Make payment for IT tax saving schemes. If you havent reached the contribution limits then put your
bonus into action for tax savings.
We could go on with quite a few suggestions but everyone situation is different.
Whatever you do, please make sure that you just do not leave it lying around in your savings account.
Yes, it might provide you with an ego boost and give you a very good feeling but its not the best
solution for your money. A typical bank pays between 4-6% annual interest. The amount you earn is also
taxable so it provides you with a very low return on your money.
E.g. If you get a bonus of Rs 5 lakhs and leave it lying in your account for a year, at the end of next year,
it will be worth Rs 5.14 lakhs after taxes. If inflation is 7%, your money is now worth less than what it
was worth last year.
So yes, dont waste your bonus but make best use of it either spend it to reduce your liabilities or
invest it.

# 79: Should I buy a flat or rent one?


Real estate investments have caught everyones attention in the past decade and everyone with 5-10
lacs salary has tried to buy Rs 40 lacs flat. Buying a house has always been considered as a good decision
by our forefathers. However, time has changed and now the theory of our forefathers does not hold
good. Real estate as an asset class gave lackluster performance from 1994-2004 and from 2009 onwards
the prices of real estate have declined by 30%.
So that leaves us with question whether you should buy a home or rent one. Like any other asset class
real estate has also delivered returns based on the economic cycle. If you have funds and the real estate
market is down, you can evaluate buying a house at a good bargain. If you want to buy for investment
its better to wait and analyze the economic data. If there are signals of economic recovery you may buy
real estate for investment purpose.
Interest on home mortgages is deductible, which sounds good but is usually overrated. Yes, its
deductible but the deductibility doesnt offset the fact that you are paying someone interest. Its an
expense, and you are worse off because of it. It may be worthwhile, all things considered, but its still an
expense.
When you rent, the landlord picks up the taxes, insurance, maintenance and sometimes utilities. If you
buy, plan on replacing the water taps one year, the flooring during another year, etc. Hope that you
dont need to replace all of them the same year.

Fin10 from www.finqa.in | info@finqa.in |+91 96500 65242

All Rights Reserved

Most people thinking about buying compare monthly payments to rent, which is a good starting point.
However, some of that monthly payment goes to principal. Its like saving. To put buying on a level
playing field with renting, look at just the part of the monthly payment that will go to interest.
Transaction costs are large in housing. Real estate agents charge one to two percent commission on
sales, which will make moving expensive. You can sell the house yourself, but keep in mind that its a lot
of work and your house may not be exposed to as many buyers, reducing the price you can get for it.
This argues against buying unless you are confident you want to stay in the house for several years,
preferably even longer.
Renters should keep in mind that they do not control their housing destiny. If the landlord decides to sell
the property, youll be looking for a new home. The landlord can also raise the rent at the end of the
lease. The landlord can also decide not to rent to you, though thats rare for people who are well
behaved.
One of the benefits of owning a house is the ability to do with it what you want. When your daughter
wants her bedroom walls red, you can be the cool parents and change the color of the wall. You can
build a bar counter in the living room and have toilets in any color of the rainbow.
Owning a house gives you some flexibility, but also requires flexibility. When you get a bonus from work,
you can upgrade your housing by adding a hot tub. Renters dont have that option. When you lose your
job, you can defer replacing the carpet.
At the end, we recommend running the numbers as best you can, then asking yourself if buying or
renting is worth the cost.

# 80: Why you dont want to be Amitabh


Bachchan?
We are referring to the character played by Mr. Amitabh Bachchan in a movie, Baghban.
Baghban film is a tale of a family where the parents (Amitabh and Hema) sacrifice everything to nurture
their four children, only to be left uncared for when they grow old. Amitabh and his wife Hema have
raised a family of four sons who are nicely settled in their respective professions. Their life is filled with
love and prosperity, and all this has been possible because parents spent all their income including
their provident fund and gratuity to secure future of their kids. But after retirement, life suddenly
changed. None of the four sons was ready to take their parents responsibility.
An elderly couple wish their children to care for them in their old age. But their children see and treat
them as a burden. Unfortunately but this is true in the era we live in. Gone are those days when parents

Fin10 from www.finqa.in | info@finqa.in |+91 96500 65242

All Rights Reserved

were considered next to God and taking care of old parents was considered as a duty and not as a
burden.
In your retirement years neither you may have windfall gains by being the author of best-selling novel
nor you may have an adopted son who would consider you as God. In reality, your retirement years will
be defined by how much savings you have made during your earning years. The physical and financial
assets that are accumulated in the income earning years are used to generate a passive income to
support expenses in retirement. Planning early and executing the plan in a way that considers changing
situations ensures that the financial security required for a comfortable retirement is achieved.
While you are saving for your retirement, you must never assume that the same money is available for
any purpose. The money saved should be strictly marked for retirement purpose. Even for your
childrens education or their professional purpose, you should never withdraw the saved money.
Retirement planning also requires periodic review to make sure that the estimates for income and
expenses in retirement are relevant or need to be changed. Typically, every time there is a significant
change in current income and lifestyle or expenses that are likely to continue into retirement, it is
necessary to make changes to the plan. The change in income or expense will imply a change in the
retirement corpus being accumulated and the periodic savings and investments that have to be made to
achieve the new target.
The allocation of the savings being made to various types of investment will depend upon the age and
stage of the individual. As the individual comes closer to retirement the portfolio has to be periodically
rebalanced to shift away to less risky assets so that the corpus accumulated over the years is protected
from a fall in value when it has to be available for generating the retirement income.
Having sufficient savings for your retirement years will not only make you comfortable but also make
your kids comfortable. Dont have high expectations from your childrens that they would take care of
you, however if your kids do take care of you in your retirement years, consider yourself blessed.

# 81: How can you not become victim of misselling?


The way a bank or insurance company sells in India is much like a trap with companies, regulators,
agents and staff colluding to defraud you of the hard earned money. The process is usually as follows:
you have a bank that you have banked for years. As soon as there is any credit (salary, bonus etc.) in
your account, the trigger is generated and sent to your account relationship manager. As soon as a
trigger is sent to your Relationship Manager, he would call you to fix an appointment. This manager
pitches a product to you. The pitch is verbal. The merits of buying this product are explained with great
enthusiasm. However, the same merits will not appear in the product brochure. The excuse will be that

Fin10 from www.finqa.in | info@finqa.in |+91 96500 65242

All Rights Reserved

the brochure is not updated. Words like guaranteed, surety, historical returns and mutual fund with
free insurance are commonly used. Next day your RM will follow up again and probably even pressurize
to decide quickly. You will get convinced because after all you are dealing with your own bank. You
would have bought something for which you either have no knowledge or half-baked knowledge.
One of the two things will happen after few months. Either you will find out that the product you bought
is not the one that was described to you. Or you wait for few years to see that market is performing well
but the product has not delivered any return. When you approach your bank with your concerns, you
will find a different manager who would be your new relationship manager. He will either say talk to the
relevant department or you should return back the policy and invest in a better scheme with the same
company. You will land up sending emails to 100 different departments or get sold for the new product
with new features.
These scams are very common irrespective whether you are dealing with a government agency or a
private company. All these relationship managers are sales agents and do not have any knowledge of
the product. They are assigned targets and they are simply chasing those targets. Neither do they have
any understanding of your financial objectives nor do they care.
A rampant practice among banks is to cross-sell insurance and mutual funds of their respective
insurance and asset management companies to customers who approach them for a loan. Customers
taking a home loan are sold mortgage redemption insurance or term insurance policies of their
insurance subsidiaries. Similarly, when a customer wants to get a locker in a bank branch, he is made to
invest in a fixed deposit. Bank personnel selling mutual funds or insurance plans are usually not even
certified by AMFI and IRDA to sell the respective products.
How can you avoid becoming a victim of mis-selling?
You must have your own basic understanding of the financial product.
You must understand how the financial product helps you achieve your financial objective.
Do not accept facts as suggested by the agent, investigate.
Any time is a good time to start investments so take your time to perform due-diligence.
Please read offer document before signing it.
Make sure the agent is registered with relevant regulator, ask for licence details.
Ideally deal with a fee-based SEBI registered investment advisor.
If you dont care about yourself, no one will care about you. So take charge of your personal finance and
avoid becoming victim of financial product mis-selling.

Fin10 from www.finqa.in | info@finqa.in |+91 96500 65242

All Rights Reserved

# 82: Do I need a professional to help me with my


finances?
If you do your own investing, have you ever wondered whether you should turn things over to a
professional advisor?
The need for critical self-evaluation is vital when determining whether to hire a financial planner. The
following questions should help you sort out whether you need an advisor:
Do you have a fair knowledge of investments?
Do you enjoy reading about investments and doing research?
Do you have expertise in investments? Do you have the time to monitor, evaluate them and make
periodic changes to your portfolio?
If you answered yes to the above questions, you may not need an advisor or financial planner. If you
feel inert in your financial life, a little professional guidance can go a long way. Frankly, going it alone
doesnt seem to work well for most investors.

Your CA may be a tax expert but may not be a financial planner. A mutual fund/Insurance
company/Bank agent will only sell a product which may or may not be suitable to your objective.
All of the above get paid when they sell you products and they earn commission from the company
whose products they sell to you. Mostly, they are individuals and are affiliated to one or two companies
and usually push products from that company. An insurance agent from Company X will not, and cannot
sell you products from Company Y even though company Ys products maybe cheaper and better suited
for your needs. This often leads to conflict of interest which usually ends up going against you.
A financial planner on the other hand is paid directly by you for the advisory service he/she provides.
They are independent and not affiliated to any company and they take a broader view of your financial
requirements and suggest a plan accordingly. With a Financial Planner, there is no conflict of interest.
There are various reasons why a financial planner is best to handle your financial planning. Financial
planning is way more than selling a particular insurance plan or a mutual fund scheme. It is about
understanding your financial goals, your financial needs and your aspirations. Chartered Accountant
focusing on tax planning can do your taxes but not financial planning. A Chartered Accountant,
insurance agent, mutual fund agent etc. may be well equipped in their area of expertise but not in
financial planning. They may not be aware to structure your portfolio for a dream you want to realize
after 20 years, say marriage of your daughter. Financial planner designs plans especially for the purpose
of solving your personal finance problems. It is similar to who would you go if you are not well, a doctor
right, not a chemist.

Fin10 from www.finqa.in | info@finqa.in |+91 96500 65242

All Rights Reserved

Our recommendation is that you do some independent research on your own, on financial planning
before finalizing on a Financial Planner. Stop relying on your CA/agents to give you the right advice,
because even if they are honest, they may not have the right skills to give you a complete Financial Plan.

# 83: Can I take care of my financial matters


myself and my family in the future ?
Yes, you can.
Taking control of the finances and making plan to achieve our goals are the first steps towards mastering
the financial planning process. A robust financial planning ensures a sustainable and secured financial
life. Though taking the first steps to plan our finances itself is a good start, the following tips would
ensure that we reap maximum benefits from the entire financial planning process.
1. Stop leakage first: Correct your past financial mistakes
Each one of us makes some mistakes in our financial decisions. Mistakes like buying wrong insurance
products, too many insurance policies, buying a home we cannot afford, credit abuse, taking loans at
higher lending rates or a bad investment are few common financial mistakes we often commit. It is
important that we learn to accept our past financial mistakes and commit ourselves to correct these
mistakes. These past mistakes would always come back to haunt us if we fail to take corrective actions
today.
2. Understand that every financial decision has an impact on other aspects of your financial life
Financial decisions should never be taken in isolation. Isolated financial decisions would often have
considerable impact on other aspects of our financial life. For example, in the case of tax planning, if
every investment decision is made with the primary motive of tax saving only, it will negatively affect
the return we receive on the investments. Similarly, every investment when being liquidated should be
assessed to know the tax implications. A holistic approach towards financial planning maximizes the
benefits of financial planning.
3. Know that goal setting is a tradeoff: Prioritize realistically
Though financial planning would help to set our financial life in order, it cannot cast a magic spell to
make all your dreams into reality. It is important that we prioritize the goals we want to achieve.
Prioritizing goals also means trade-off. We would have to let go some of our dreams which we feel are
less important than others. Once we prioritize the goals, we have to be realistic while translating our
goals to monetary terms. It is not advisable to expect unrealistic returns on the investments. The
external factors like inflation, interest rate changes and several other macro-economic factors would
have a major say in determining the net return from the investments.
4. Dont sit on your financial plan, implement immediately: Costs of delay are huge
Getting a financial plan and recommendations on financial products through expert advice is job half
done until put into action. We often postpone implementing our financial plan to later dates. The
success of a well-planned financial road map depends on how soon we start walking on it. The early

Fin10 from www.finqa.in | info@finqa.in |+91 96500 65242

All Rights Reserved

starters always have a huge advantage of power of compounding and a flexibility of saving lesser
amounts. If we want to accumulate Rs.50,000 in 5 months at a 10 per cent annual rate compounded
monthly, we have to save Rs.9,754 per month. However if we have a 10 month period, we would be
saving only Rs.4776 a month. Starting today, even with baby steps, is the most critical success factor for
your financially secured future.
Apart from the four points mentioned above, managing and monitoring you plan at regular intervals is
important. The financial situation of every individual is dynamic in nature. Our goals and priorities
change from time to time. Our personal life situations would also have an impact on financial life. We
must review our financial plan periodically in order to ensure that it is in tune with the current events in
our life and on course to meet our long term goals.
Any financial decision that you make is sure to impact your future. It therefore becomes easier when
you take the help of an expert as managing money is a time taking process and requires good know-how
of current situations.

# 84: Higher education is expensive, why not plan


for it !
Before you start your studies, you must ensure that you have the necessary funds in place for the
duration of the programme. Most funding sources have strict deadlines, often several months before
your course formally begins.
Studying for higher education is expensive. Hence planning is essential to ensure that adequate funds
are at disposal when needed. Education today is the key to professional success and is treated as an
investment by parents who are willing to run that extra mile to give their child the best possible
education. In recent years, education has been globalised which has increased awareness of the best
institutes worldwide or even the fact that under grad, grad education can be done aboard. This has
resulted in more and more Indian students going to various countries for education; be it for grad
courses or post grad courses. Undertaking a course at these schools is a substantial investment in time
and money. One of your key decisions will be how to finance your studies and estimate the costs
involved. Below mentioned are few tips before you even apply for various programs.
Think ahead
Before you start your studies, you must ensure that you have the necessary funds in place for the
duration of the programme. Most funding sources have strict deadlines, often several months before
your course formally begins.
Use your funds first
Provide as much personal funding as possible through any investments, savings or sponsorship you have
access to, before approaching any external funding organizations. Much of the external support
available will depend on the course you wish to apply for, your mode of study and your nationality or
place of residence.

Fin10 from www.finqa.in | info@finqa.in |+91 96500 65242

All Rights Reserved

Part-time students fees only


There are only a limited number of scholarships available to part-time students who are in full-time
employment. You will be expected to cover the living costs yourself, normally through employment.
Start-up costs
You must have enough funds to cover at least eight weeks of living expenses when arriving to start the
course (more if you need to cover the cost of the duration of your study period). Your expenses are
highest at the start of the course when you need to pay an accommodation deposit, buy a travel card for
transportation, buy your first course materials and so on.
Check for programme-specific funding
Check for any course-specific internal scholarships offered by the selected University/School. Many
universities offer financial assistance to international students primarily based on merit and rarely on
poor financial background. The amount and type of assistance offered varies for different universities,
departments and level of study. Assistance of funds is more likely for graduate studies and less in
undergraduate courses. Also funds are more likely to be available in fields like engineering, physical and
biological sciences, rather than humanities, social sciences and management.
So how do you go about saving for your education funding requirements. Firstly, if your goal is couple of
years away (>5 years), you must get equity exposure through mutual funds . This will grow your portfolio
more than the inflation. In case your education goal starts in less than 5 years, you should allocate more
towards debt oriented instruments. Most of the government schemes such as PPF, NSCs etc. should be
avoided because the maturity date of these instruments is more than 5 years. The easiest option is
opening a fixed deposit with your bank, however that is not a tax efficient way. You should have a
combination of ultra-short term and short term funds. Investing in ultra-short term and short term funds
give you a benefit of indexation (inflation adjusted returns) and your portfolio can grow tax-efficient.
You can avoid financial crisis while you are at a school/university and even post that, provided you plan
and save for your education goal.

# 85: True returns for the Investors


There is a relatively new concept called advisors alpha, which describes the value investment advisers
bring to a client relationship. Most of this is self-serving for the investment industry, but there are some
important points surrounding this concept. Individual investors strive for the highest returns possible
but do not necessarily think about process, efficiency, or discipline. These are all areas in which a
professional investment adviser can help.
Process

Fin10 from www.finqa.in | info@finqa.in |+91 96500 65242

All Rights Reserved

Setting up an asset allocation plan, or getting the right mix of risk and return, is important. However,
maintaining this balance over time is equally important. Regular rebalancing and having a process in
place to invest methodically through time are rare among individual investors.
Tax efficiency
Asset location, tax swapping, holding out for long-term capital gains, and methodically selecting which
tax lots of a stock to sell can all have real economic value by controlling the amount lost to taxes.
Discipline
The discipline to stay in the market, rather than selling out at market bottoms, is probably the most
important role of a good investment adviser. Any market timing can be damaging, but abandoning
discipline at market extremes (peaks and valleys) can be all but impossible to recover from.
Individual investors can go to a low-cost mutual fund provider, construct a balanced portfolio, and
rebalance annually or with cash flows that they dollar-cost average into the portfolio. This approach is
simple, efficient, and should provide competitive returns. However, the vast majority will be tempted by
hot tips and media stories that encourage more active trading.
Working with a professional may not only produce market-beating returns net of fees and taxes but also
help avoid many of the behavioral traps individual investors fall into. Professionals have a wider
knowledge of portfolio construction, customary fee levels, and tax-minimization techniques and often
bring much-needed discipline to the investment process.

# 86: 5 steps to the perfect financial plan


Taking control of the finances and making plan to achieve our goals are the first steps towards mastering
the financial planning process. A robust financial planning ensures a sustainable and secured financial
life. Though taking the first steps to plan our finances itself is a good start, the following tips would
ensure that we reap maximum benefits from the entire financial planning process.
1. Stop leakage first: Correct your past financial mistakes
Each one of us makes some mistakes in our financial decisions. Mistakes like buying wrong insurance
products, too many insurance policies, buying a home we cannot afford, credit abuse, taking loans at
higher lending rates or a bad investment are few common financial mistakes we often commit. It is
important that we learn to accept our past financial mistakes and commit ourselves to correct these
mistakes. These past mistakes would always come back to haunt us if we fail to take corrective actions
today.

Fin10 from www.finqa.in | info@finqa.in |+91 96500 65242

All Rights Reserved

2. Understand that every financial decision has an impact on other aspects of your financial life
Financial decisions should never be taken in isolation. Isolated financial decisions would often have
considerable impact on other aspects of our financial life. For example, in the case of tax planning, if
every investment decision is made with the primary motive of tax saving only, it will negatively affect
the return we receive on the investments. Similarly, every investment when being liquidated should be
assessed to know the tax implications. A holistic approach towards financial planning maximizes the
benefits of financial planning.
3. Know that goal setting is a tradeoff: Prioritize realistically
Though financial planning would help to set our financial life in order, it cannot cast a magic spell to
make all your dreams into reality. It is important that we prioritize the goals we want to achieve.
Prioritizing goals also means trade-off. We would have to let go some of our dreams which we feel are
less important than others. Once we prioritize the goals, we have to be realistic while translating our
goals to monetary terms. It is not advisable to expect unrealistic returns on the investments. The
external factors like inflation, interest rate changes and several other macro-economic factors would
have a major say in determining the net return from the investments.
4. Go for a credible expert
We generally rely on neighborhood agents, friends or family for financial advice. Internet has also
emerged as a source for financial information. It is important that we always assess the credibility of the
source before taking financial advice. As mentioned earlier, financial mistakes would always have a
major influence in our future well-being. Choose your advisor wisely to make sure that you always make
well informed financial decisions.
5. Dont sit on your financial plan, implement immediately: Costs of delay are huge
Getting a financial plan and recommendations on financial products through expert advice is job half
done until put into action. We often postpone implementing our financial plan to later dates. The
success of a well-planned financial road map depends on how soon we start walking on it. The early
starters always have a huge advantage of power of compounding and a flexibility of saving lesser
amounts. If we want to accumulate Rs.50,000 in 5 months at a 10 per cent annual rate compounded
monthly, we have to save Rs.9,754 per month. However if we have a 10 month period, we would be
saving only Rs.4776 a month. Starting today, even with baby steps, is the most critical success factor for
your financially secured future.
Apart from the five points mentioned above, managing and monitoring you plan at regular intervals is
important. The financial situation of every individual is dynamic in nature. Our goals and priorities
change from time to time. Our personal life situations would also have an impact on financial life. We
must review our financial plan periodically in order to ensure that it is in tune with the current events in
our life and on course to meet our long term goals.

# 87: Which should you use Credit or Debit


Cards?
Gone are the days when you would have to carry wads of cash in your wallet. Debit cards and credit
cards are probably the most convenient way to carry money without having to carry any money. They
look similar, are accepted universally, and make for light travel. But carrying a credit card is like getting a
small loan. In other words, you basically ask the bank to lend you the money you dont have, but

Fin10 from www.finqa.in | info@finqa.in |+91 96500 65242

All Rights Reserved

promise to repay in a month when you receive the credit card bill. A debit card, on the other hand,
withdraws directly from your bank account, i.e. draws on existing funds.
If you are disciplined with respect to the payments and your cash flow supports your expenditure, a
credit card is a preferred option. You get 45 days interest free loan plus bonus points. However, you will
easily be trapped if you dont make the complete payment before due date. Another benefit of using a
credit card is that you get a snapshot of your expenses at the end of each month. You dont have to
maintain the expenses separately but the Credit Card Company will send you the statement which will
have the expense amount, nature of expense and date when the expense was incurred. Few companies
also offer analysis of expenditure.
However, use of debit card is preferred if you are not disciplined with respect to making payments. The
reason could be lack of funds or just carelessness. In that situation, use of debit card is the most
preferred option since the money is taken from your bank account.
Know your behavioral patterns when it comes to spending, it will help you make the right decision.

# 88: 6 things you should NOT try and save


money on
Human beings are funny. We dont mind spending Rs 5000 for a meal at a fancy restaurant but will
haggle with the vegetable vendor for Rs 5 !!
Similarly, while we tend to splurge on certain areas, we also try and save on stuff we should not be
doing.
Here are some things which you should get of the best quality of, and why:
1. Mattresses: Think about it. We spend almost 30% of our lives sleeping on one. Yet, we dont buy
the most comfortable or optimal one. A bad back will be more expensive in the long run than a
good mattress
2. Shoes: Our feet are the most under appreciated part of our bodies and they take the maximum
stress. Show your feet some love and buy comfortable and reliable pair(s) of shoes. Yes, if you are
thinking of running or walking, investing in a proper set of trainers will save you a lot of injuries in
the long term
3. Underwear: If they arent comfortable, they will make your life miserable. Enough said
4. Water Purifier: Do we really need to say why clean and pure water is important for your family. Get
the best that is out there.
5. Toothbrushes: Poor dental health is not just about painful teeth. If you have bad teeth, you cant
chew food properly, which often leads to indigestion, which in turn leads to weight

Fin10 from www.finqa.in | info@finqa.in |+91 96500 65242

All Rights Reserved

gain/constipation and a host of other problems. So yes, brush after every meal or atleast twice a
day. And change your toothbrushes regularly before they fray out.
6. Fruits: Fresh seasonal fruits are a great source of healthy calories and fibre along with being tasty. If
you think fruits are expensive, try and think of that last drink or meal you had at a restaurant which
left you hungover and bloated the next morning. Suddenly, they wont seem very expensive.
Aim to plan your life financially so that you can live life to the fullest, enjoy doing what you do and make
the most of your money.

# 89: Should You Hire A Financial Planner Or


Wealth Manager?
Whether its your bank, insurance agent, mutual fund agent or some guy you knew in college hassling
you through LinkedIn, theres no shortage of people offering to help you plan your financial future, the
challenge is figuring out which type of financial advisor you should seek.
Further complicating matters, theres often a disconnect in what people describe themselves as doing.
But whats really important is how they get paid. .
Heres how to tell the difference between the two:
Wealth Manager
The term Wealth Manager can be used to describe either insurance salespeople or mutual fund agents
that get paid on a commission. These types can be found at insurance companies, acting as independent
agents, advisors at large banks that prefer to be paid on a transactional basis. This person will be paid to
get you to do some kind of financial product agree to buy or sell a mutual fund or agree to a long term
contract with an insurance company. Generally theyll be paid a sales commission from the company
selling the product or a brokerage commission from their firm in the case of a mutual fund.
Generally Wealth Managers acting as salespeople and not held to a fiduciary standard, meaning that
they arent compelled to put their clients interests above their own, but are instead held to a lower
suitability standard. Obviously wealth managers have certain conflicts of interest and may be inclined
to sell clients investments purely for the purpose of generating a commission.
Financial Planner

Fin10 from www.finqa.in | info@finqa.in |+91 96500 65242

All Rights Reserved

True financial planners are held to a fiduciary standard, and generally hold the CFP credential. Often,
insurance advisors or wealth managers will position themselves as a financial planner to stir up business
they will eventually lead to commissions or assets under management.
A financial planner usually describes an advisor who, for an hourly or project based fee, helps their
clients develop a written financial plan that they will execute elsewhere.
If a true financial planner, theyre among the most unbiased sources of advice. Because they are paid a
fee (as opposed to commission or fee-based), they tend to be more interested in working with less
wealthy clients. In fact solutions offered by a financial planner may not be for wealthy. A wealthy person
will be more interested in a wealth management service and not so much in a financial plan.
Now you know the difference between the two. Do your due diligence and make the right decision.

# 90: When and why to rebalance your Portfolio?


Investment decisions often throw a challenge of risk vs. return. In the pursuit of maximizing returns,
most of us often end up ignoring the risk associated with it. As a result, you dont have an optimal riskreward portfolio. By optimizing the risk, you can ensure that the portfolio being created is in sync with
your risk appetite and stay invested for a longer period of time.
However, returns from individual assets in a portfolio vary over time and might deviate from your
original target asset allocation. In order to restore asset allocation to optimal risk levels, individuals
should consider rebalancing their portfolios periodically based on changes in market conditions and risktolerance levels.
Let us examine why, when and how one should rebalance a portfolio and also assess the factors which
prompt for rebalancing in the current market conditions.
Why rebalance your portfolio?
Over a period of time, some asset classes may provide higher returns than others while some give
suboptimal results. In order to recapture the original risk-return characteristics, your portfolio should be
rebalanced periodically.
For example, let us consider an individual who invested Rs. 6,000 in equities and Rs. 4,000 in debt a year
ago. And now, the debt funds are valued at Rs. 6,000 and the equity funds are valued at Rs. 5,500. A
target allocation of 60 per cent in equity and 40 per cent in debt is no longer good enough. As per this
example, the investor is holding 52 per cent in debt and 48 per cent in equities. In order to regain the
original composition of his portfolio, he should sell a part of his debt investments and invest more in
equities.
How to rebalance your portfolio?

Fin10 from www.finqa.in | info@finqa.in |+91 96500 65242

All Rights Reserved

While some argue that by rebalancing a portfolio, one is essentially selling the winners and buying the
losers. However, it is also important to note that the markets are volatile by nature and it is hard to tell
the winning picks from the rest. By withdrawing at the right time, you can also ensure that you maximize
the returns from your investments and exit a pick before it falls from its high price. Common strategies
here include time-based rebalancing or threshold rebalancing.
While time-based rebalancing emphasizes on change in portfolio allocation at predefined intervals,
threshold rebalancing emphasizes on percentage of deviation. The rule of threshold rebalancing reads:
Rebalance the portfolio once the current allocation deviates from the target allocation by 10 per cent.
When you should rebalance your portfolio?
It is important to note that rebalancing a portfolio is associated with costs in the form of transaction
costs, to execute the process, and capital gains, if applicable. Instead of reviewing your portfolio more
often, limiting the frequency of rebalancing to once or twice a year is a cost efficient and recommended
practice.
Frequent rebalancing of your portfolio would result in higher transaction costs and taxes as well as
impact the net return from your investments. Portfolio rebalancing decisions should be driven by
change in its composition, macro-economic changes and/or changes in ones risk appetite.
Conclusion
If we consider the current scenario of Indian markets, the debt markets have performed well during the
recent times on the back of interest rate cuts implemented by the apex bank, Reserve Bank of India. The
equity markets remained highly volatile and provided suboptimal returns during this time. When
translated to your portfolio, this means that debt funds have dominated equity funds in terms of the
returns they provided. Also, this increases debt exposure above the optimal level.
However, the current state of the rupee may not bode well with the debt market. The debt markets
might not perform as well in the current year as they did last year. Equity markets, on the other hand,
remain deeply undervalued. The undervalued equity market stands out as an investment opportunity
which can provide good returns in the future. Therefore, by rebalancing your portfolio at this time you
will be ensuring that you are selling your debt funds at a higher price and buying equity funds at a lower
price.
While rebalancing your portfolio, consider the tax aspects of long-term and short-term capital gains.
Also make a note of the commissions associated with these investments. If you are investing through
mutual funds, know the exit load criteria of the funds you hold. If you are investing through SIPs, check
whether all the investments made are in line with the exit load conditions of the fund.

# 91: Why your credit score is very important ?


Fin10 from www.finqa.in | info@finqa.in |+91 96500 65242

All Rights Reserved

In these times when each of us is increasingly dependent on credit cards whether to buy household
appliances or furniture or simply to splurge on some luxuries it becomes very necessary to know what a
credit score is.
Just like you use a thermometer to check body temperature when ill or a speedometer used to calculate
the distance travelled over time, a credit score is used to analyze your credit health. One of the
important aspects of being financially healthy is checking what your credit health report says. A credit
score is a 3 digit number ranging between 300 900 representing your credit and payment history.
And, yes why exactly is it important for you to know about your credit score is because it plays a
significant role when you are looking to buy a car on credit or get a home loan or even apply for a new
credit card. It shows how well you have maintained your credit payment in the past and thereby gives a
picture of your management of credit in the future.
A low credit score translates into having to pay higher interest rates on loans or at times being denied to
avail a loan or even a new credit card. Building and maintaining a good credit score is an important skill
that will benefit you when you need to avail loans. A score above 700 usually suggests good credit
management.
A credit score tells the lenders what level of risk they would be taking by loaning money to you and that
is why when you apply for a credit card or any loan your credit score is checked.
Factors Determining Credit Score
Payment History: This is the record youve established of how promptly or not promptly you have been
paying back your credit. This negatively impacts your credit score in terms of late payments, defaults on
your EMIs.
Credit Usage: It is the ratio of your credit card debt to your credit limits. It checks as to how much of
your available credit you use on a monthly basis. If an individual often uses their entire credit limit then
they are higher chances that he/she finds it difficult to repay back.
Duration of that Account: With a longer credit history more information is available about your
spending habits. This includes the average time since the accounts were opened by account type and
the account activity.
New Card Inquiries: Every time you make an inquiry about getting a new credit card, your credit score is
affected. Minimize the frequency of applying for unnecessary credit by often requesting for new credit
cards.
Credit Mix: This refers to a mix of revolving credit like credit card and installment loans like auto loan or
home loan. A history of borrowing using different types of credit (for example, credit card, home
mortgage, and car loan) increases your score.
In India credit scores are calculated by credit information companies such as CIBIL, Equifax and Experian.
You can get a copy of your credit report from these agencies by paying a nominal charge.

Fin10 from www.finqa.in | info@finqa.in |+91 96500 65242

All Rights Reserved

# 92: Which Loans should you pay off first?


Loans are important part of modern day personal finance. Most of us rely on loans for funding our
higher education, new car or home etc. Though loans boost our purchasing power, over reliance on debt
often leads to financial stress. One important question that we often encounter is: When should I close
my loan? Exit strategy from existing loans plays an important role in minimizing the interest burden.
Prioritizing loan repayments ensures that the loans get cleared in a systematic way to boost the
available monthly surplus. The loan repayments should be prioritized in the following order:

Priority 1: Personal loans


Personal loans top the priority list when it comes to paying off existing debt. Personal loans are
unsecured loans which are advanced on the basis of the borrowers credit history and ability to repay
the loan from the available income sources. Being an unsecured loan, Personal loans are often offered
at a higher interest rate. Higher interest rate necessarily means higher EMI payments. Though the
repayment charges for personal loans are also on a higher side, it is always advisable to close this high
interest debt once an individual has enough surpluses.

Priority 2: Unproductive loans


The loan instruments like gold loans, loan against property, loan against fixed deposits and insurance
policies, loan against PF and auto loan do not attract any tax benefits. Such loans should be paid off
based on the interest burden. The interest rate on gold loans and loan against property are dependent
on margin between pledged value and loan amount. If an individual opts for 50 per cent of the value of
the gold as loan then he or she is expected to get a better rate compared to opting for 80 90 per cent
of the value as loan. These loans hold a lesser interest rate compared to personal loans. Loans against
fixed deposits, insurance and PF attract lower interest rate than the gold loans and loans against
property.

Priority 3: Educational loan


The increasing educational expenses have aided in the increased demand for educational loans.
Educational loans should be given second least priority before closing off the existing debts. The reason
behind it would be the tax savings one can enjoy on the educational loans. One can claim tax benefit on
the interest payments being towards educational loan availed from approved institutions. So essentially
the interest payments can be offset by the tax benefit and hence one is advised to pay off educational
debt only after paying off other debts.

Priority 4: Home loan


Home loans are the most common form of debt among the Indians. One can avail tax benefits on both
principal repayment and interest payments on the home loan. This tax advantage makes the home loan
the last debt an individual should pay off. The exit strategy for home loan also differs based on the
tenure and type of house. Generally in the initial years, majority of the EMI payments account for
interest payments and during the last few years of loan tenure they account for principal repayments. It

Fin10 from www.finqa.in | info@finqa.in |+91 96500 65242

All Rights Reserved

is advisable to consider prepayment during the first half of the loan tenure. If an individual has two
existing home loans, only interest payments on second home loan, which is not self-occupied, are tax
deductible. However, there is no cap on this deduction. So considering the tax benefits associated with
them, home loans should be paid off after servicing all the other existing debts.
Though the above mentioned priority list give an outline of debt servicing, sometimes you may find an
investment which pays you higher interest rate than the interest rate being paid on the existing debt. As
with any financial decision, make sure you analyze the pros and cons of whether to opt for an
investment or to pay off the existing loan. Exiting a loan is an important decision that should be made
using the merit based reasoning (ROI, opportunity cost) than emotional reasoning (debt free life).

# 93: The 3 starters to Managing Debt Effectively


Im buried in debt;
My Debts going out of control;
Totally stressed out, my debt is too high
Dont want to find yourself talking this language just make sure you follow these 3 starters to be on
track and keep yourself in the pink of financial health.
1. Control your expenses, check your spending habits:
Ahhhh! We know you are saying that you already know this and so we might as well wrap up here. But,
frankly knowing this and not following it is equal to not knowing, so we might as well repeat. The ABC of
personal finance is Always Be Careful with your spending. You must not spend more than you earn;
therefore it becomes extremely important to check that your spending habits are aligned with your
income.

Money saved is money earned. Simple! You start saving early and cut down on expenses that
stretch your purse strings, you are pretty much already on track then.
Guard against impulse buying - That unplanned indulging is your biggest budget spoiler. Of
course, its not easy to keep yourself away from temptation but writing a list before going out
shopping could be a starting point to self discipline thus enabling you to save yourself a significant
amount of money every year.
Be realistic; as much as we would want to live with Alice in her Wonderland lets remember that
Pandoras Box is the actual reality! You are the right judge to decide what to lay your hands on and
what not to. So driving yourself into a make-believe mode that your purchasing decision is justified
emotionally will start creating the dent into your pocket sooner than you think. Nevertheless, you

Fin10 from www.finqa.in | info@finqa.in |+91 96500 65242

All Rights Reserved

totally deserve to live your dreams so if you want something really badly then write your goal and
chalk out your plan in reaching there.
Never borrow money to buy expensive luxury items you can do without. Frankly, this would be the
most ridiculous act you could do. This is definitely a BIG NO!
Save regularly- Yes, this is another adage that will never change! If you have surplus cash, invest in
liquid funds. Invest regularly as per your risk appetite for wealth creation and make sure you
consult the right person before taking these decisions.
2. Borrow prudently:
Borrowing money can help you do things that you otherwise couldnt do. However, you can get into
financial difficulty if you borrow too much, too frequently. Focus on investment debt like home loans
and education loans. A house appreciates in value in the long run and extra educational qualifications
propel your career to greater heights with better paying jobs. Limit your consumption debt; do not look
for easy access to cash for short term unplanned buying. Pay outstanding credit card bills before the due
date and never borrow cash against your credit card.
3. Create opportunities for an extra income:
Where there is a will, there is a way. However, in case managing your debts are going out of hand and
you have not found a way to clear them off, its time that you carve that way yourself. You must try and
think of options to generate extra income.
Whether taking a part time/ freelancer job position or renting a part of your home or turning your
hobby into your source of income; open your eyes wide and look for other avenues to generate a
second income stream, even if it is a small one.
You deserve to be worry-free with regards to your debts and that sure can be achieved via a financial
plan. So that you may bid ciao to your debt faster.

# 94: How to avoid the cycle of Bad Debt?


Amita Shahs way of managing debt
She wanted to buy a house?
She took a loan.
Wanted to redo her home interiors?
Took another loan.
Wanted to buy a car?
And a third loan.
Now, she wanted to close the first loan. Took another loan to close it. The second loan was giving her
sleepless nights. She was looking for another option to help her close that. And the vicious cycle began
While she was earning 15lacs per annum her expense had reached 20lacs per annum!! She never
understood the red signal earlier and so before she realized she was sunk in debt and obviously sunk in
despair. Eventually all the aspects of her life were severely impacted.
A common sight is that people often take multiple loans to fulfill their aspirations and then need more
loans to close the previous ones. However, a wise step is to clear your previous debts before applying

Fin10 from www.finqa.in | info@finqa.in |+91 96500 65242

All Rights Reserved

for new loans. Be it when taking a personal loan for renovating your house to give it a stylish look or
paying through credit cards for expensive house hold items or clothes that you actually cant afford,
these kinds of debts are to be avoided as much as possible. It usually tends to happen that the amount
you pay in interest, late fees and any other penalties might actually exceed the value of the product or
service you purchased. Paying EMIs for multiple loans can be daunting, resulting in the inability to pay
current outstanding dues in time. You finally find yourself losing out on your savings and forced to take
irrational decisions leading to the distress sale of valuable assets like house and jewellery to repay debts.
Taking a loan to clear previous debts is a poor strategy as this puts you in a dire situation where you will
struggle to clear all your debts through your lifetime.
If you find yourself in a similar situation, the first thing is to take external help from a qualified person to
help you sort out things.

# 95: Things you should know about PPF


Interested in investing into Public Provident Fund (PPF) but not sure where to start? Read on.
How do I open a PPF account? What should I keep in mind when opening my PPF account?
You can open a PPR account in selected nationalized banks, selected private banks or post office. Fill in
the form, attach a photograph, state your PAN Number, and youre done. Once your formalities are
completed, you will receive a pass book which will record all your PPF transactions. At any point in your
life, you are allowed to have only 1 PPF account in your name. You can also have an account in the name
of a minor child of whom you are the parent / guardian. However that will be the childs account, you
will simply be the guardian. You can never have a joint account. If at any time it is seen that you have
more than 1 account in your own name, the second account will be deactivated, and only your principal
will be returned to you. If you have a General provident Fund account, or an Employees Provident Fund
account, you can still have a PPF account there is no restriction.
Can an NRI open a PPF account?
Non Resident Indians are not eligible to open an account under the PPF Scheme. However Provided
that if a resident who subsequently becomes a Non Resident during the currency of the maturity period
prescribed under the PPF scheme may continue to subscribe to the Fund till its maturity, on a Non
Repatriation Basis. So if you open it as an RI, and during the 15 year tenure become an NRI, you can
continue to invest, but on a non-repatriable basis.
When is the best time to invest in PPF account?
The best time to invest is between the 1st and the 5th of any month, preferably April each year. Interest
is calculated for the calendar month on the lowest balance at credit of your account, between the close
of the 5th day and the end of the month, and is credited at the end of every year.

Fin10 from www.finqa.in | info@finqa.in |+91 96500 65242

All Rights Reserved

Is a Loan against PPF account allowed?


Yes loan facility is available against a PPF account. The first loan can be taken in the third year of opening
the account i.e., if the account is opened during the year 2010-11, the first loan can be taken during the
year 2012-2013. The loan amount will be restricted to 25% of the balance including interest for the year
2010-11 in the account as on 31/3/2011. The loan must be repaid in a maximum of 36 EMIs. You can
take a second loan against your PPF account before the end of your sixth financial year, but your second
loan can be taken only once your first loan is fully settled.
Are withdrawals from PPF account allowed?
Any time after the expiry of the 5th year from the date that the initial subscription is made, you become
eligible to withdraw an amount of not more than 50% of the previous years balance or of the 4th year
immediately preceding the year of withdrawal, whichever is less. If you have taken any loan on your PPF,
this also gets factored in and reduces your balance. You cannot make more than a single withdrawal in
the year. You need to apply with Form C for any withdrawals.
What happens after PPF account matures?
You have 3 choices. Either you can withdraw your maturity amount, or you can extend your account by a
5 year block, as many times as you want and make fresh contributions, or you can extend the account
without making any further contributions, and continue to earn interest on it every year. If you decide to
withdraw your money, your maturity value is exempt from tax. If you decide to extend your account and
continue making fresh contributions, you can extend it for a block of 5 years at a time, as many times as
you want, you can also make withdrawals from the account, up to 60% of the account balance that was
there at the beginning of the extended period. Just remember, if you choose to extend your account,
submit the necessary documentation for extension before one year passes from the maturity date. If
you choose to extend your account without making any fresh contributions, you can do so. In this case,
any amount can be withdrawn without any restriction; however you can only withdraw once per year.
The balance will continue to earn interest till it is withdrawn.

# 96: From whom to buy mutual funds


As an investor, you are spoilt for choice not only when it comes to the number and type of schemes
available but also for the path you pick to invest in schemes. Broadly, there are five different channels
through which you can investdirectly into a mutual fund either via their online/mobile platform or in
physical form, through an independent financial adviser (IFA), through your bank, through your online
broker or your online fund adviser and through your wealth management firm.

Criteria

Mode

Independent Financial

Corporate entities

Banks

Online

Direct plans

An IFA will not just sell

These are essentially

Any bank with

Many broking

The new direct plan

you a mutual fund

the wealth

which you have a

firms and banks

option enables you

scheme but will take

management and

savings account or

with broking

to buy your scheme

Advisor

Fin10 from www.finqa.in | info@finqa.in |+91 96500 65242

All Rights Reserved

you through a process

financial planning

any other

arms have shifted

directly from the

of financial planning to

firms which offer a

meaningful

a lot of services

fund house and

ascertain your

service similar to

relationship such as

online.

benefit from that

financial goals,

what your IFA gives

a fixed deposit

purchase by paying

spending and

you but through a

might approach you

a lower expense.

investment needs.

more organized

for channelizing

platform.

funds into
investment
products including
mutual funds.

It is similar to
Many such firms are

Pros

Your level of trust and

now present across

Comes without any

comfort with an IFA is

big cities. This means

cost and can be

what will determine

if you move from one

accessed easily for

the success of the

city to another, you

investing in mutual

association.

wont have to change

funds.

your adviser.

You can basically


buy and redeem
funds through
your online
account with the
fund house.

investing via an
online portal,
however, in this
case you will end up
having to
consolidate your
mutual fund
portfolio yourself.

While the bank will


not charge an
additional fee,
Comes with a pricing
similar to IFAs. You
may not be able to

Cons

Many IFAs do charge

strike the same level

fees for the advice and

of trust and comfort

the financial planning

as an IFA offers.

service they extend.

Moreover, changing
faces of advisers
doesnt go down well
with all.

some may charge a


transaction fee for
executing the
purchase and this

The online portal

Direct plan is not

could go upto 2%. It

doesnt provide

useful if you need

is not a mandatory

any advice which

guidance on which

charge that you

all mutual funds

mutual funds to

have to pay and is

to select.

buy.

based more on the


size of the
transaction and the
relationship and
hence can be
negotiated.

You need to be aware of the investment adviser guidelines which SEBI has put in place. These guidelines
segregate an adviser, who advices on your investment choice, from an agent who simply sells a fund.
Your adviser can earn only from the fee she charges you for the investment advice, whereas, an agent
represents the product company (mutual fund in this case) and hence, earns commission. In other

Fin10 from www.finqa.in | info@finqa.in |+91 96500 65242

All Rights Reserved

words, the adviser is motivated to give you the best product as you are paying her and the agent may be
motivated to sell the product which earns her the maximum commission. Knowing this difference is
important. After this, whether you buy funds from an agent or an adviser depends more on whether you
need advice or simply want to execute. It also depends on the relationship you have with your agent; if
the trust is established you may be willing to continue buying from her despite the lack of the word
adviser as part of her credentials.

# 97: How to Open Post Office Monthly Income


Scheme
Though the postal system is finding few takers with the advent of email and other forms of
communication, the guarantees offered by post office small savings, make them still one of the most
sought after location for financial products.
Once you have selected the post office to open the POMIS account, you will first need to open a post
office savings account to link the monthly pay-out from your MIS account and you will need the
following documents:
An account opening form, which the post office will provide.
Two passport size photographs.
Address and identity proof such as copy of passport, PAN (permanent account number) card or
declaration in form No 60 or 61 as per the Income Tax Act 1961, driving license, voters identity card
or ration card.
Carry original identity proof for verification at the time of account opening.
Choose a nominee and get a witness signature to complete the formalities to get started.
How to Operate the Account?
You need a pay-in slip with the initial account opening sum to be credited into your account.

The savings and MIS account have passbooks with rules applicable to the accounts stated in them.

# 98: How to buy National Savings Certificate


NSC offers assured returns and tax benefits. If planned well, one can create a regular income stream
using this instrument.

Fin10 from www.finqa.in | info@finqa.in |+91 96500 65242

All Rights Reserved

Once you have decided on the sum that you wish to invest:
You need to fill the NSC application form available at the post office
Carry original identity proof for verification at the time of buying

You can buy the certificate with cash, cheque or demand draft drawn in favour of the postmaster
of the post office from where the NSC is being bought
Choose a nominee and get a witness signature to complete the formalities when buying the
certificate
Eligibility: You need to be a Resident Indian to buy these certificates
Entry age: No age is specified for account opening
Investments: Minimum of Rs 100 per annum. Certificates are available in denominations of Rs 100, Rs
500, Rs 1,000, Rs 5,000 and Rs 10,000
Interest: 8.6 per cent compounded half yearly on 5-year tenure and 8.9 per cent compounded half
yearly on 10-year tenure
Tenure: 5 and 10 years
Account holding categories: Individual, Joint and Minor through the guardian
Risks associated with loss or mutilation of certificate exists, for which a duplicate certificate can be
issued on furnishing an indemnity bond in a format prescribed by the post office.

# 99: Want to invest in bonds but dont know


where to start?
Corporate bonds
The process of investing in corporate bonds is as simple as investing in any other instrument. When a
company issues bonds, you will have to fill out an application form and submit it to any branch of the
issuing company with the application fee and required documents. These documents may include a copy
of your PAN card, address proof, identity proof, etc. If you have a demat account, you can fill out the
details in the form, and the bonds will get credited to it. In case you do not have a demat account, you
can choose to receive the bonds in a physical format.
Government bonds
Government bonds, unlike shares, are not traded on the stock market. They are sold through official
distributors and designated branches of banks and post offices. To invest in bonds, you will need to visit
any branch designated with applications and submit a filled-up application form. After your request is
processed, you will receive a bond certificate in your name.
Infrastructure bonds

Fin10 from www.finqa.in | info@finqa.in |+91 96500 65242

All Rights Reserved

Hindu undivided families and any Indian resident who is not a minor can invest in these bonds. A person
should ideally submit only one application. Multiple applications will be aggregated based on the
permanent account number or PAN. The tax benefit will be allowed only on investment up to Rs 20,000.
The bonds can be held both in demat and physical forms.
While the process of investing in bonds is quite simple, selecting the best type of bond might be difficult,
especially if you are new to making investments in the debt market. If you are unsure of the facts or
need guidance in the process, taking professional advice can clear the picture for you.

# 100: How to buy life insurance?


Till late 90s insurance sector was dominated by LIC agents. Usually a family LIC agent would invariably
sell some or the other policy on each visit. There were no objections made for the family friend selling a
government sponsored product. Therefore, your parents landed up buying these LIC policies, one after
the other without knowing why they were even buying these policies. Neither the LIC agent was aware
of how the policy would operate nor were your parents interested in knowing. Guess what, the tradition
continues; only the generation has changed but still new generation is buying insurance products as a
tool for making money. Big mistake!
Life insurance is an important tool of risk management. Whenever you combine life insurance with
investment objective, risk management will take a backseat. Therefore, it is strongly recommended to
have sufficient risk cover before you are start evaluating investment linked insurance products. A term
insurance product is sufficient to provide risk cover at a reasonable price. A 28 year old male nonsmoker term policy for a cover of Rs 1 crore comes around Rs 10,000. Whereas the 1 crore insurance
cover through an investment linked insurance product comes at a premium of Rs 275,000. Think about
it, why not take a term plan of Rs 1 cr by paying a premium of Rs 10k p.a. and invest Rs 2.65 lacs in a
growth asset.
Most of the people feel comfortable buying a child education plan or a retirement plan from insurance
because they feel insurance company would do the needful. But guess what, in the initial few years
there would be exorbitant management charges and actually less money will get invested. Investors
who bought these policies five years back have still not received their principal back.
Therefore the first step of buying a life insurance should be in the form of a term plan. The
recommended way of buying a term policy would be online so that you save on the commissions, which
are paid to the insurance agent. Insurance company reputation is linked with the claim settlement ratio.
Claims settlement ratio is the only objective yardstick for the consumer to determine which insurance
company is preferable and more reliable. IRDAs data reveals that LIC has the best claim settlement ratio

Fin10 from www.finqa.in | info@finqa.in |+91 96500 65242

All Rights Reserved

of 97.73% among life insurers in the country. Of the 23 private life insurers, only five ICICI Prudential
Life, SBI Life, HDFC Life, Max Life and Kotak Life have a claim settlement record of over 90%.
LIC e-term plan is relatively expensive but should be one of the term plans you purchase. You may have
one private player policy and the other LIC policy.

Fin10 from www.finqa.in | info@finqa.in |+91 96500 65242

All Rights Reserved

Congratulations on making this far.


Personal finance is considered a pretty boring topic so we hope the
information above has made you slightly more aware of various
aspects.
Here another thing to think about:
If you had to build a house, would you hire an architect
If you had to get heart surgery or a root canal, would you go to a
professional doctor
If you wanted to be a musician, how long would you have to
practice for ?
If you wont do any of the above yourself, then Why would you want
to grow your money on your own, especially when you know so little
of how the market operates?
The key reason is Trust.
There is a major deficit of it in the retail financial space. Agents have
mis-sold products rampantly in the past, due to which folks have lost
money = which has translated in them not trusting anyone but trying
to do it on their own.
Thats why we started Finqa to provide a trusted, unbiased an
honest advisory service. Thats also the reason why we charge for our
advice (unlike other agents / salesmen in the market)
Start small with our free trial plan. See our performance for a year and
then decide on the future. http://finqa.in/get-started/
www.finqa.in | info@finqa.in |+91 96500 65242

Fin10 from www.finqa.in | info@finqa.in |+91 96500 65242

All Rights Reserved

And one last thing


Please feel free to share
this book with anyone
who you think could
benefit from it.
Pass it on.

Fin10 from www.finqa.in | info@finqa.in |+91 96500 65242

All Rights Reserved

Vous aimerez peut-être aussi